SlideShare une entreprise Scribd logo
1  sur  60
Télécharger pour lire hors ligne
1
Revisão Geral de
Termologia
Prof. Marcos Guimarães
Sampaio
(só prá moçadinha)
Termometria
01) (ITA – SP) O verão de 1994 foi
particularmente quente nos Estados Unidos da
América. A diferença entre a máxima
temperatura do verão e a mínima no inverno
anterior foi de 60 °C. Qual o valor dessa
diferença na escala Fahrenheit?
a) 108 °F
b) 60 °F
c) 140 °F
d) 33 °F
e) 92 °F
Resolução: ∆C/5 = ∆F/9  60/5 = ∆F/9
 ∆F = 108 °F (Resposta A)
02) (MACK – SP) Uma pessoa mediu a
temperatura de seu corpo, utilizando-se de um
termômetro graduado na escala Fahrenheit, e
encontrou o valor 97,7 o
F. Essa temperatura, na
escala Celsius, corresponde a:
a) 36,5 o
C
b) 37,0 o
C
c) 37,5 o
C
d) 38,0 o
C
e) 38,5 o
C
Resolução: (F – 32)/9 = C/5  (97,7 –
32)/9 = C/5  C = 36,5 ºC (Resposta A)
03) (UFPI - PI) O Aquecimento Global é um
fenômeno climático de larga extensão. As
previsões mais catastróficas para a região
Amazônica incluem o desaparecimento
completo da floresta se a temperatura média da
região tiver um aumento superior aos 5ºC. Com
isso a temperatura média anual da cidade de
Manaus passaria a ser de 33ºC, que lida na
escala Kelvin corresponderia a:
a) 300 K
b) 310 K
c) 290 K
d) 306 K
e) 302 K
Resolução: T(K) = C + 273  T(K) = 33
+ 273 = 306 K. (Resposta D)
04) (IFAL - AL) Uma pessoa que estava
viajando para Londres, no dia seguinte, procura
se informar a respeito das condições do tempo
no local de destino. Num canal de TV a cabo,
toma conhecimento de que a temperatura
naquela cidade e de 26,6°F, pois, na Inglaterra,
costuma-se usar termômetros graduados na
escala Fahrenheit. Baseando-se nessa
informação, o viajante chegara a seguinte
conclusão:
a) Terá que levar pesados agasalhos para
suportar uma temperatura abaixo do zero
absoluto.
b) Com uma temperatura alem dos 36°C, não
devera se preocupar com agasalhos.
c) Levara roupas leves, pois a temperatura esta
entre 27°C e 36° C.
d) Como a temperatura esta abaixo de 0°C,
providenciara roupas adequadas a um inverno
rigoroso.
e) Usará suas roupas normais, pois as
temperaturas nas duas escalas são iguais para
todos os valores.
Resolução: (F – 32)/9 = C/5  (26,6 –
32)/9 = C/5  C = – 3 ºC. (Resposta D)
05) (UFPE - PE) As escalas de temperatura
mais conhecidas são Celsius (ºC) e Fahrenheit
(ºF). Nessas escalas, o ponto de congelamento
da água corresponde a 0ºC e 32ºF, e o ponto de
ebulição corresponde a 100ºC e 212ºF.
A equivalência entre as escalas é obtida por uma
função polinomial do 1º grau, ou seja, uma
função da forma f(x) = ax + b, em que f(x) é a
temperatura em grau Fahrenheit (ºF) e x a
temperatura em grau Celsius (ºC). Se em um
determinado dia a temperatura no centro do
Recife era de 29ºC, a temperatura equivalente
em grau Fahrenheit (ºF) era de:
a) 84ºF
b) 84,02ºF
c) 84,1ºF
d) 84,12ºF
e) 84,2ºF
Resolução: (F – 32)/9 = C/5  (F –
32)/9 = 29/5  F = 84,2ºF (Resposta E)
06) (PUC – RS) Podemos caracterizar uma
escala absoluta de temperatura quando:
a) dividimos a escala em 100 partes iguais.
b) associamos o zero da escala ao estado de
energia cinética mínima das partículas de um
sistema.
c) associamos o zero da escala ao estado de
energia cinética máxima das partículas de um
sistema.
d) associamos o zero da escala ao ponto de
fusão do gelo.
e) associamos o valor 100 da escala ao ponto de
ebulição da água.
2
Resolução: Zero absoluto corresponde ao
estado de agitação mínimo das partículas o que
significa energia cinética mínima. (Resposta B)
07) Imagine dois termômetros graduados. Um
na escala Celsius e o outro na escala Fahrenheit.
Sendo assim, responda às questões seguintes:
a) Quando o termômetro Celsius acusar uma
variação de temperatura igual a 20 ºC determine
a variação de temperatura acusada por um
termômetro Fahrenheit e por um termômetro
Kelvin.
b) Quando um termômetro Celsius acusar uma
temperatura de 20 ºC, qual será a temperatura
acusada, no mesmo local por um termômetro
Fahrenheit e por um Kelvin.
Resolução:
a) ∆C/5 = ∆F/9  20/5 = ∆F/9  ∆F =
36 ºF e ∆T(K) = ∆C  ∆T(K) = 20 K.
b) (F – 32)/9 = C/5  (F – 32)/9 = 20/5
 F = 68ºF e T(K) = C + 273 = 20 + 273
= 293 K. (Respostas: a) 36 ºF e 20 K; b) 68ºF
e 293 K)
08) (UFRPE - PE) Duas escalas termométricas,
ºX e ºY, têm suas respectivas temperaturas, TX e
TY, relacionadas pela expressão 2TX – 3TY + 2 =
0. Pode-se afirmar que uma variação de
temperatura de 30 ºX corresponde, na escala Y,
a uma variação de:
a) 10 ºY
b) 20 ºY
c) 30 ºY
d) 40 ºY
e) 50 ºY
Resolução: Para cada 1ºX de variação teremos,
em ºY: 2.1 – 3TY + 2 = 0  TY = 4/3 ºY.
assim, para uma variação de 30 ºX teremos
30.4/3 = 40ºY. (Resposta: D)
09) Três afirmações são feitas a respeito de
temperatura.
I) Zero absoluto equivale à temperatura em que
ocorre o congelamento da água.
II) Podemos obter água a - 350 ºC.
III) Quando um corpo tem sua temperatura
variando 50 ºC pode-se dizer que, se a escala
fosse a Kelvin, a variação de temperatura,
numericamente, seria a mesma.
a) Apenas I é falsa
b) Apenas II é falsa
c) Apenas III é falsa
d) Apenas III é verdadeira
e) As três afirmações são falsas.
Resolução:
I- (F) O congelamento da água sob pressão
normal ocorre a 273 K.
II- (F) A menor temperatura teoricamente
possível é o 0 K ou – 273 ºC.
III- (V) Já que o intervalo entre os pontos fixos
de fusão do gelo e ebulição da água são os
mesmos.
(Resposta D)
10) (ACAFE – SC) Três blocos de metais
diferentes foram aquecidos de forma contínua e
suas temperaturas medidas, instante após
instante, por três termômetros, simultaneamente.
Um, na escala Celsius, outro na escala
Fahrenheit e o terceiro na escala Kelvin.
A figura abaixo mostra três gráficos da
temperatura versus tempo, T X t,
confeccionados a partir dos dados obtidos. As
curvas de aquecimento, em cada gráfico, foram
obtidas por termômetros diferentes.
Analisando os gráficos, é correto o que se
afirma em:
a) No gráfico C, a curva de aquecimento 1
corresponde às medidas realizadas pelo
termômetro na escala Fahrenheit e a curva de
aquecimento 2 corresponde às medidas
realizadas pelo termômetro na escala Celsius.
b) No gráfico A, a curva de aquecimento 2
corresponde às medidas realizadas pelo
termômetro na escala Celsius e a curva de
aquecimento 1 corresponde às medidas
realizadas pelo termômetro na escala Kelvin.
c) No gráfico B, a curva de aquecimento 2
corresponde às medidas realizadas pelo
termômetro na escala Fahrenheit e a curva de
aquecimento 1 corresponde às medidas
realizadas pelo termômetro na escala Celsius.
d) No gráfico B, a curva de aquecimento 1
corresponde às medidas realizadas pelo
termômetro na escala Fahrenheit e a curva de
aquecimento 2 corresponde às medidas
realizadas pelo termômetro na escala Kelvin.
e) No gráfico C, a curva de aquecimento 1
corresponde às medidas realizadas pelo
termômetro na escala Kelvin e a curva de
aquecimento 2 corresponde às medidas
realizadas pelo termômetro na escala
Fahrenheit.
Resolução: A partir do zero absoluto, a relação
gráfica entre as escalas é a seguinte:
-273
0
-459,4
F
C
K
3
a) F: o encontro entre as escalas ocorre a -40.
b) V: a distância numérica entre as escalas será
sempre igual a 100 unidades.
c) F: como está explicado no item anterior
d) F: a curva para a escala Kelvin inicia no 0
(zero absoluto).
e) F: como está explicado no item anterior.
(Resposta B)
11) (CEFET – PR) A superfície gelada do
pequeno Plutão é composta por nitrogênio,
metano e traços de monóxido de carbono. A
temperatura do planeta anão varia ao longo de
sua órbita porque, no decorrer de sua trajetória,
aproxima-se do Sol até 30 UA e afasta-se até 50
UA. Existe uma tênue atmosfera que congela e
cai sobre o planeta anão quando este se afasta
do Sol. Sendo assim, dependendo da sua
posição em relação ao Sol, a temperatura sobre
a superfície do planeta anão varia de –230ºC a –
210ºC. Pode-se afirmar que: (UA = Unidade
Astronômica)
a) essas temperaturas não são lidas num
termômetro graduado na escala Kelvin, pois a
menor temperatura nesse termômetro é 0 K.
b) não se medem essas temperaturas num
termômetro graduado na escala Celsius, pois sua
escala varia de 0ºC a 100ºC.
c) se medem essas temperaturas com
termômetros graduados na escala Celsius, pois é
o único que mede temperaturas abaixo de zero.
d) na escala Fahrenheit, o módulo da variação
da temperatura sobre a superfície do pequeno
Plutão corresponde a 36ºF.
e) na escala Fahrenheit, o módulo da variação
da temperatura sobre a superfície do pequeno
Plutão corresponde a 20ºF.
Resolução: ∆C/5 = ∆F/9  (-210 – (-230))/5 =
∆F/9  ∆F = 36 ºF. (Resposta D)
12) (ITA – SP) Para medir a febre de pacientes,
um estudante de medicina criou sua própria
escala linear de temperaturas.
Nessa nova escala, os valores de 0 (zero) e 10
(dez) correspondem respectivamente a 37°C e
40°C. A temperatura de mesmo valor numérico
em ambas escalas é aproximadamente
a) 52,9 °C. b) 28,5 °C. c) 74,3 °C.
d) –8,5°C. e) –28,5°C.
Resolução: (t – t1)/(t2 – t1) = (C – C1)/(C2 – C1)
 (t – 0)/(10 – 0) = (C – 37)(40 – 37)  (x –
0)/(10 – 0) = (x – 37)(40 – 37)  x/10 = (x –
37)/3  x  28,5 ºC. (Resposta B)
13) (MACK – SP) Joãozinho, seguindo as
orientações de seu professor de Física, construiu
uma nova escala termométrica. Ao nível do mar,
ele atribuiu o valor -20 °J para a temperatura do
gelo fundente e 130 °J para a temperatura de
ebulição da água. A medida, que nessa escala
tem valor coincidente com o da escala Celsius,
refere-se à temperatura:
a) 20 °J
b) 30 °J
c) 40 °J
d) 50 °J
e) 60 °J
Resolução: (t – t1)/(t2 – t1) = (C – C1)/(C2 – C1)
 (t – (-20))/(130 – (-20)) = (C – 0)/(100 – 0)
 (x + 20)/150 = x/100  x = 40. (Resposta
C)
14) (MACK – SP) Um profissional,
necessitando efetuar uma medida de
temperatura, utilizou um termômetro cujas
escalas termométricas inicialmente impressas ao
lado da coluna de mercúrio estavam ilegíveis.
Para atingir seu objetivo, colocou o termômetro
inicialmente numa vasilha com gelo fundente,
sob pressão normal, e verificou que no
equilíbrio térmico a coluna de mercúrio atingiu
8,0 cm. Ao colocar o termômetro em contato
com água fervente, também sob pressão normal,
o equilíbrio térmico se deu com a coluna de
mercúrio atingindo 20,0 cm de altura. Se nesse
termômetro utilizarmos as escalas Celsius e
Fahrenheit e a temperatura a ser medida for
expressa pelo mesmo valor nas duas escalas, a
coluna de mercúrio terá altura de:
a) 0,33 cm
b) 0,80 cm
c) 3,2 cm
d) 4,0 cm
e) 6,0 cm
Resolução: A temperatura comum nas escalas
Celsius e Fahrenheit é: (F – 32)/9 = C/5 
(x – 32)/9 = x/5  x = -40
Resposta C
15) (UFMS – MS) Através de experimentos,
biólogos observaram que a taxa de canto de
grilos de uma determinada espécie estava
relacionada com a temperatura ambiente de uma
maneira que poderia ser considerada linear.
Experiências mostraram que, a uma temperatura
de 21º C, os grilos cantavam, em média, 120
vezes por minuto; e, a uma temperatura de 26º
C, os grilos cantavam, em média, 180 vezes por
minuto. Considerando T a temperatura em graus
0
100
8
20
-40 y
(0 – (-40))/(100 – (-40)) = (8 –
y)/(20 – y)  y = 3,2 cm
4
Celsius e n o número de vezes que os grilos
cantavam por minuto, podemos representar a
relação entre T e n pelo gráfico abaixo.
Supondo que os grilos estivessem cantando, em
média, 156 vezes por minuto, de acordo com o
modelo sugerido nesta questão, estima-se que a
temperatura deveria ser igual a
a) 21,5 ºC .
b) 22 ºC .
c) 23 ºC .
d) 24 ºC .
e) 25,5 ºC .
Resolução:
Resposta D
16) (UNIFESP – SP) O texto a seguir foi
extraído de uma matéria sobre congelamento de
cadáveres para sua preservação por muitos anos,
publicada no jornal “O Estado de S. Paulo” de
21.07.2002:
Após a morte clínica, o corpo é resfriado com
gelo. Uma injeção de anticoagulantes é
aplicada e um fluido especial é bombeado para
o coração, espalhando-se pelo corpo e
empurrando para fora os fluidos naturais. O
corpo é colocado numa câmara com gás
nitrogênio, onde os fluidos endurecem em vez
de congelar. Assim que atinge a temperatura de
-321°, o corpo é levado para um tanque de
nitrogênio líquido, onde fica de cabeça para
baixo.
Na matéria, não consta a unidade de
temperatura usada. Considerando que o valor
indicado de -321° esteja correto e que pertença a
uma das escalas (Kelvin, Celsius ou
Fahrenheit), pode-se concluir que foi usada a
escala:
a) Kelvin, pois se trata de um trabalho científico
e esta é a unidade adotada pelo Sistema
Internacional.
b) Fahrenheit, por ser um valor inferior ao zero
absoluto e, portanto, só pode ser medido nessa
escala.
c) Fahrenheit, pois as escalas Celsius e Kelvin
não admitem esse valor numérico de
temperatura.
d) Celsius, pois só ela tem valores numéricos
negativos para a indicação de temperaturas.
e) Celsius, por tratar-se de uma matéria
publicada em língua portuguesa e essa ser a
unidade adotada oficialmente no Brasil.
Resolução: A menor temperatura teoricamente
possível é o 0 K ou – 273 ºC ou -459,4 ºF.
Portanto a única escala que se enquadra com o
valor de temperatura descrito no texto é a
Fahrenheit. (Resposta C)
17) Uma escala termométrica X é construída de
modo que a temperatura de 0 °X corresponde a
– 4 °F, e a temperatura de 100 °X corresponde a
68°F. Nesta escala X, a temperatura de fusão do
gelo vale:
a) 10 °X b) 20 °X c) 30 °X
d) 40 °X e) 50 °X
Resolução:
18) Um termômetro de mercúrio é calibrado
com o ponto de gelo a 2cm de altura da coluna
de mercúrio e o ponto de vapor a 12cm.
Obtenha uma função que relacione a
temperatura T(°C) com a altura h (cm) e
determine a temperatura T(ºC) quando h = 10
cm.
Resolução:
19) Os pontos de fusão do gelo e de ebulição da
água na escala Fahrenheit são, respectivamente,
32 °F e 212 °F. Um termômetro A, graduado na
escala Fahrenheit, e outro B, graduado na escala
Celsius, são colocados simultaneamente em um
frasco contendo água quente.
Verifica-se que o termômetro A apresenta uma
leitura que supera em 80 unidades a leitura do
termômetro B. Podemos afirmar que a
temperatura da água no frasco é:
a) 60 °C b) 80 °C c) 112 °C
d) 50 °F e) 112 °F
Resolução: F = C + 80 e : (F – 32)/9 = C/5
 (C + 80 – 32)/9 = C/5  C = 60 ºC
(Resposta A)
x
26
156
180
21 120
(x – 21)/(26 – 21) = (156 –
120)/( 180 – 120)  (x –
21)/5 = 36/60  x = 24 ºC
x
100
32
68
0 -4
(x – 0)/(100 – 0)) = (32 – (-4)/(68
– (-4)  x/100 = 36/72  x =
50. (Resposta E)
C
100
x
12cm
0 2cm
(C – 0)/(100 – 0)) = (x – 2)/12 –
2)  C/100 = (x – 2)/10  C =
10x – 20 (Resposta: T(ºC) = 10
T(ºX) – 20)
5
20) (MACK – SP) Um termômetro defeituoso
está graduado na escala Fahrenheit, indicando
30 °F para o ponto de gelo e 214 °F para o
ponto de vapor. Neste termômetro, a única
temperatura medida corretamente, corresponde
a:
a) 0 °F b) 30 °F c) 40 °F
d) 50 °F e) 122 °F
21). Buscando satisfazer uma diversificada
clientela, algumas operadoras de turismo
lançaram a “viagem surpresa”: o viajante
compra um pacote para um local desconhecido,
só sabendo seu destino ao desembarcar. A única
informação é dada alguns dias antes da viagem
e refere-se à temperatura média no local a ser
visitado. Numa dessas viagens, um passageiro
recebeu o seguinte comunicado: a temperatura
média local à época da viagem é 82ºF. Esse
passageiro deve:
a) levar chapéu, filtro solar, bermudas e roupas
leves.
b) levar agasalhos leves, pois o clima é ameno.
c) precaver-se com capotes e agasalhos de lã,
pois nessa temperatura há possibilidade de
encontrar neve.
d) desistir de viajar, pois a temperatura é tão
baixa que dificilmente conseguirá sair do hotel.
Resolução: Comparando com a escala Celsius,
temos:
(F – 32)/9 = C/5  (82 – 32)/9 = C/5
 C = 87,7 ºF. (Resposta: A)
22) Um estudante construiu um termômetro,
adotando uma escala termométrica obtida do
seguinte modo:
- mergulhou uma haste metálica em gelo
fundente. Algum tempo depois, constatou que a
medida da haste era de 20 cm. A esta medida
atribuiu o valor de 50 graus Ypsolon (50 ºY);
- em seguida, mergulhou a mesma haste
metálica em um recipiente contendo água em
ebulição. Algum tempo depois, constatou que a
haste, dilatada, passou a medir 20,3 cm. A esta
medida atribuiu o valor de 80 graus Ypsolon (80
ºY).
Sendo assim, responda às questões a
seguir:
a) Ao mergulhar a haste metálica, por algum
tempo, na água contida em um recipiente, seu
comprimento se torna igual a 20,2 cm.
Determine, em graus Ypsolon, a temperatura da
água no recipiente.
b) A temperatura de 90 ºY está associada a que
medida de comprimento da haste metálica ?
Resolução: A equação de conversão será (y –
50)/(80 – 50) = (L – 20)/(20,3 – 20)  (y –
50)/30 = (L – 20)/0,3  y – 50 = 100L – 2000
a) y – 50 = 100.20,2 – 2000  y = 70 ºY.
b) 90 – 50 = 100L – 2000  L = 20,4 cm.
(Respostas: a) 70 ºY; b) 20,4 cm)
23) (ENEM) Nos processos industriais, como na
indústria de cerâmica, é necessário o uso de
fornos capazes de produzir elevadas
temperaturas e, em muitas situações, o tempo de
elevação dessa temperatura deve ser controlado,
para garantir a qualidade do produto final e a
economia do processo
Em uma indústria de cerâmica o forno é
programado para elevar a temperatura ao longo
do tempo de acordo com a função em que T é o
valor da temperatura atingida pelo forno, em
graus Celsius, e t é o tempo, em minutos,
decorrido desde o instante em que o forno é
ligado.
Uma peça deve ser colocada nesse forno quando
a temperatura for 48 o
C e retirada quando a
temperatura for 200 °C.
O tempo de permanência dessa peça no forno é,
em minutos, igual a
a) 100 b)108 c) 128 d) 130 e) 150
Resolução:
x
214
x
212
30 32
(x – 30)/(214 – 30) = (x –
32)/(212 – 32)  (x – 30) 184 =
(x – 32)/180  x = 122 ºF
(Resposta: E)
6
Temperatura inicial do forno T(t0) = 7/5.t0 + 20
 T(0) =7/5.0 + 20 = 20 ºC.
A temperatura do forno será 48 ºC (momento
em que a peça será introduzida em: T(t) = 7/5.t
+ 20  48 = 7/5.t + 20  t1 = 20 min
Após 100 minutos a temperatura da água será:
T(t2) = 7/5.t2 + 20  T(100) = 7/5.100 + 20 =
160 ºC.
Após isso, atingirá a temperatura de 200 ºC
quando: T(t3) = 2/125t3² - 16/5t3 + 320  200 =
2/125t3² - 16/5t3 + 320  t3 = 50 min.
Assim a peça permanecerá no forno durante t =
(t2 – t1) + t3 = (100 – 20) + 50 = 130 min.
(Resposta D)
Dilatação Térmica
24) (UFES – ES) Uma barra de metal tem
comprimento igual a 10,000 m a uma
temperatura de 10,0 °C e comprimento igual a
10,006 m a uma temperatura de 40 °C. O
coeficiente de dilatação linear do metal é:
a) 1,5 × 10-4
°C-1
b) 6,0 × 10-4
°C-1
c) 2,0 × 10-5
°C-1
d) 2,0 × 10-4
°C-1
e) 3,0 × 10-4
°C-1
Resolução: ∆L = L0.α.∆  0,006 = 10. α.30
 α = 2.10–5
ºC–1
(Resposta C)
25) (UFU – MG) Uma ponte de aço tem 1 000m
de comprimento. O coeficiente de dilatação
linear do aço é de 11. 10-6
°C–1
. A expansão da
ponte, quando a temperatura sobe de 0 para
30°C, é de:
a) 33cm.
b) 37cm.
c) 41cm.
d) 52cm.
e) 99cm.
Resolução: ∆L = L0.α.∆  ∆L = 1000. 11.
10-6
.30 = 0,33m ou 33 cm (Resposta A)
26) (MACK – SP) Duas barras metálicas, de
diferentes materiais, apresentam o mesmo
comprimento a 0 °C. Ao serem aquecidas, à
temperatura de 100 °C, a diferença entre seus
comprimentos passa a ser de 1 mm. Sendo 2,2.
10–5
°C–1
o coeficiente de dilatação linear do
material de uma barra e 1,7.10–5
°C–1
o do
material da outra, o comprimento dessas barras
a 0 °C era:
a) 0,2 m
b) 0,8 m
c) 1,0 m
d) 1,5 m
e) 2,0 m
Resolução: ∆L = L0.α1.∆ – L0.α2.∆ 
1.10-3
= L0. 2,2. 10–5
.100 – L0. 1,7.10–5
.100 
L0 = 2,0 m (Resposta E)
27) (UERJ – RJ) Uma torre de aço, usada para
transmissão de televisão, tem altura de 50 m
quando a temperatura ambiente é de 40 0
C.
Considere que o aço dilata-se, linearmente, em
média, na proporção de 1/100.000, para cada
variação de 1 0
C.
À noite, supondo que a temperatura caia para 20
0
C, em valor absoluto a variação de
comprimento da torre, em centímetros, será de:
a) 1,0
b) 1,5
c) 2,0
d) 2,5
Resolução:
∆L = L0.α.∆  ∆L=50.( 1/100.000).(-20) =
1/100 m ou 1 cm. (Resposta A)
28) Entre dois trilhos consecutivos de uma via
férrea, deixa-se um espaço apenas suficiente
para facilitar livremente a dilatação térmica dos
trilhos de 0 ºC até a temperatura de 70 ºC. O
coeficiente de dilatação térmica linear do
material dos trilhos é 1,0. 10–5
°C–1
. Cada trilho
mede 20m a 0 °C. Qual o espaço entre dois
trilhos consecutivos na temperatura de 0 °C?
Resolução: ∆L = L0.α.∆  ∆L = 20.
1,0. 10–5
.70 = 0,014m = 1,4 cm., ou seja, 0,7 cm
para cada lado. Desse modo, entre doius trilhos
sucessivos devermos ter d = 1,4 cm. (Resposta:
1,4 cm)
29) Uma telha de alumínio tem dimensões
lineares de 20cm x 500cm e seu coeficiente de
dilatação linear é igual a 2,2 x 10–5
ºC–1
. A telha,
ao ser exposta ao sol durante o dia, experimenta
uma variação de temperatura de 20 ºC. A
dilatação superficial máxima da chapa, em cm²,
durante esse dia, será.
a) 1,1
b) 2,2
c) 4,4
d) 6,6
e) 8,8
Resolução: ∆A = A0.β.∆  ∆A =
(20.500).(2. 2,2 x 10–5
).(20) = 8,8 cm².
(Resposta E)
30) Para se ligar estrutura metálicas em prédios
usa-se a técnica de rebitagem em que para se
colocarem os rebites é preferível que:
a) eles estejam à mesma temperatura da chapa;
b) eles estejam à temperatura superior a da
chapa, geralmente aquecidos ao rubro;
c) eles estejam resfriados a temperaturas abaixo
da chapa;
7
d) qualquer das possibilidades acima ocorre
desde que fiquem bem colocados
Resolução: A uma temperatura inferior a da
chapa, o diâmetro do rebite será menor que o da
chapa permitindo a introdução do rebite.
Quando a temperatura se elevar, o rebite estará
perfeitamente encaixado. (Resposta C)
31) (FCC – SP) Uma peça sólida tem uma
cavidade cujo volume vale 8cm³ a 20ºC. A
temperatura da peça varia para 920 ºC e o
coeficiente de dilatação linear do sólido (12.10–6
ºC–1
) pode ser considerado constante. Supondo
que a pressão interna da cavidade seja sempre
igual à externa, a variação percentual do volume
da cavidade foi de:
a) 1,2%.
b) 2,0%.
c) 3,2%.
d) 5,8%.
e) 12%.
Resolução: ∆V = V0.γ. ∆ sendo γ = 3α. Assim
∆V = V0.(3. 12.10–6
).900 = 0,0324. V0. Portanto
3,24/100 V0 = 3,24% V0.(Resposta C)
32) (UFPI – PI) Muitos sistemas de
aquecimento elétrico em que é necessário
manter certa temperatura, tais como estufas,
fornos, ferro com graduações apropriadas para
passar diversos tipos de tecidos, usam
termostato para manter sua temperatura mais ou
menos constante. Esses reguladores de
temperatura têm o mesmo princípio de
funcionamento. De uma forma simplificada,
pode-se dizer que eles possuem uma espiral
bimetálica como mostrada na figura abaixo.
Considere essa espiral bimetálica formada por
latão na parte interna e de aço na parte externa.
Com relação ao adequado funcionamento destes
sistemas reguladores de temperatura, coloque
V, para verdadeiro, ou F, para falso.
1 ( ) À medida que a temperatura do sistema de
aquecimento elétrico vai caindo, a espiral
bimetálica se contrai, e sua extremidade se
move para direita, isto é, para fora, abrindo o
contato elétrico.
2 ( ) À medida que a temperatura da espiral
bimetálica aumenta, o latão se dilata mais que o
aço e, quando a temperatura atinge o valor
máximo necessário, a espiral se move para
direita ou para fora, desligando o circuito.
3 ( ) Quando a temperatura da espiral aumenta,
além da necessária para o respectivo sistema de
aquecimento, o latão se dilata menos que o aço,
e a espiral se move para direita ou para fora,
desligando o circuito.
4 ( ) Quando a temperatura do sistema diminui,
o aço se contrai mais que o latão, e a espiral
bimetálica fecha novamente o circuito, até o
sistema atingir a temperatura máxima da
graduação do sistema de aquecimento.
Resolução: Observe que: internamente: latão
(2,0.10–5
) e externamente: aço (1,3.10–5
).
1- (F) pois o latão sofrerá maior contração
térmica que o aço, já que seu coeficiente de
dilatação é maior.
2- (V) pois o latão sofrerá maior dilatação.
3- (F) No aquecimento o latão se dilata mais que
o aço. Seu coeficiente é maior.
4- (F) no resfriamento o latão contrai mais que o
aço. (Resposta: F; V; F; F)
33) (IFCE – CE) Uma barra de aço de
comprimento L0 = 0,700 m é encaixada
praticamente sem folga em uma cavidade
lubrificada, como mostra a figura.
A temperatura inicial da barra é de 20,0 ºC e seu
coeficiente = 12,0 x 10-
6
ºC-1
.
Quando uma das extremidades da barra é
mantida fixa e a outra é puxada por uma força
T, seu comprimento se altera de acordo com a
equação L = L0 (1 + T/f), com f = 6,00 x 107
N
(t < 0 se T comprime a barra). Supondo-se que a
cavidade não se deforma, a força horizontal que
a barra exerce sobre a parede da cavidade à
direita, para uma temperatura de 22,0ºC, é igual
a
a) 1220 N
b) 1420 N
c) 1620 N
d) 1520 N
e) 1440 N
Resolução: L0 = 70cm e ∆L será ∆L = L0.α.∆
= 70. 12,0 x 10-6
.(22 – 20) = 168.10-5
= 0,00168
cm  L = 70,00168cm. A força será L = L0 (1
8
+ T/f)  70,00168 = 70(1 + T/6.107
)  t =
1440 N. (Resposta E)
34) (IFAL – AL) Um fato bem conhecido
levado em conta por técnicos, engenheiros,
cientistas e outros profissionais ao se
defrontarem com a tarefa de construir algum
equipamento, projetar uma construção, ou
mesmo efetuar um experimento científico, é que
as dimensões de um corpo aumentam quando
aumentamos sua temperatura.
Ressalvando algumas exceções, este é um
comportamento geral dos sólidos, líquidos e
gases. Sobre este fenômeno, denominado
dilatação térmica, analise as afirmativas a
seguir:
I. Toda dilatação térmica, em verdade, se
processa nas três dimensões espaciais:
comprimento, largura e altura.
II. A variação das dimensões de um corpo
depende, exclusivamente, do material de que ele
é feito e da sua variação de temperatura sofrida.
III. No caso dos líquidos, observamos que a
dilatação real deles depende do recipiente no
qual estão inseridos e é sempre menor que a
dilatação aparente observada.
Levando-se em conta os conceitos
cientificamente corretos, das afirmações acima
está(ao) correta(s):
a) apenas I.
b) apenas II.
c) apenas III.
d) I e II.
e) I, II e III.
Resolução:
I- (V) As dilatações ocorrem em todas as
direções.
II- (F) Depende também de suas dimensões
iniciais, já que a dilatação também é
proporcional a ela.
III- (F) A dilatação do real de um líquido não
depende da dilatação do frasco. Apenas a
aparente depende. (Resposta A)
35) (ACAFE – SC) O fenômeno da dilatação
térmica pode ser usado para manter um sistema
numa temperatura aproximadamente constante
através de um dispositivo chamado termostato.
Usa-se uma lâmina bimetálica (duas hastes de
materiais diferentes, porém com mesmo
comprimento inicial) que se encurva à medida
que a temperatura muda e que pode abrir ou
fechar circuitos elétricos, ou válvulas, e assim
manter o sistema, como um ferro elétrico, um
forno ou um sistema de ar condicionado em
uma temperatura prédeterminada.
A figura abaixo mostra esquematicamente um
dispositivo desse tipo onde, ao aumentar a
temperatura da lâmina, ela se curva para
_______ se o coeficiente de dilatação da haste
A for _____ coeficiente de dilatação da haste B.
A alternativa correta que completa as lacunas
acima, em seqüência, é:
a) baixo - menor que o
b) cima - maior que o
c) cima - menor que o
d) baixo - igual ao
e) cima - igual ao
Resolução: Se αA > αB e no caso de
aquecimento a lâmina se curva para cima
fechando o circuito. Portanto: cima; maior que
o. (Resposta B)
36) (ACAFE – SC) Em um laboratório de física,
um grupo de alunos aquece uma barra metálica
homogênea, A, de comprimento L0, a partir de
uma temperatura inicial T0 e mede o
comprimento da barra para cada grau de
aumento da temperatura. Um segundo grupo
esfria uma barra idêntica, B, a partir da mesma
temperatura inicial T0 e mede o comprimento da
barra para cada grau de diminuição da
temperatura. Depois de encerradas as medidas,
os alunos colocam os dados em um mesmo
gráfico L x T, onde L representa o comprimento
das barras A e B, e T representa a temperatura
correspondente.
A alternativa que apresenta o gráfico correto é:
Resolução: Se as barras são idênticas, para cada
grau de variação de temperatura, a dilatação será
igual a contração. Portanto. (Resposta D)
37) (MACK – SP) Um conjunto de certos
dispositivos para laboratório, proveniente dos
Estados Unidos, contém dentre outros produtos,
uma pequena lâmina retangular. No livreto de
informações, existe a recomendação para não
expor essa lâmina a temperaturas inferiores a 30
ºF e tampouco superiores a 85 ºF. Um estudante
9
brasileiro, interessado em saber de que material
era constituída a lâmina, descobriu, através de
um ensaio, que, durante um aquecimento nesse
intervalo de temperatura, a área da lâmina
aumentava de 0,165%. Consultando a tabela
abaixo, pode-se afirmar que, possivelmente, a
lâmina é de material
Material
Coeficiente de
dilatação linear ( o
C-
1
)
Ferro 12.10-6
Ouro 15.10-6
Bronze 18.10-6
Alumínio 22.l0-6
Chumbo 27.10-6
a) ferro.
b) ouro.
c) bronze.
d) alumínio.
e) chumbo.
Resolução: ∆ = 55 ºF = 30,55ºC e ∆A =
A0.β.∆  0,165% A0 = A0.β.30,55  β =
5,4 . 10-5
e como β = 2α temos α =
27.10-5
ºC-1
(chumbo). (Resposta E)
38) (CEFET – PR) Analise as assertivas abaixo
e marque (V) verdadeiro ou (F) falso.
1- ( ) A dilatação real de um líquido será maior
do que a dilatação aparente observada.
2- ( ) Entre 0ºC e 4ºC há um aumento da
temperatura da água e aumento também da sua
densidade.
3- ( ) O vidro temperado Pirex pode ser levado
diretamente ao fogo sem trincar porque seu
coeficiente de dilatação é maior que o do vidro
comum.
4- ( ) Ao aquecermos uma aliança de ouro, 18
quilates, ocorrerá dilatação em todas as
dimensões, inclusive para a região interna da
aliança.
5- ( ) Três gases diferentes, com o mesmo
volume e estando a mesma temperatura inicial,
se forem aquecidos de forma idêntica e
mantidos a uma pressão constante, apresentarão
o mesmo coeficiente de dilatação volumétrica.
A alternativa correta é:
a) V, F, F, V, F
b) F, F, V, V, F
c) V, V, F, F, V
d) F, F, F, V, F
e) V, V, F, V, F
Resolução:
1-(V) O coeficiente de dilatação de um líquido
é, normalmente, maior que o do frasco. Assim a
dilatação real é maior que a aparente.
2-(V) Ocorrerá uma contração em seu volume e
conseqüentemente em sua densidade.
3-(F) ele não trinca por sofrer pouquíssima
dilatação em virtude do seu baixo coeficiente.
4-(F) ocorrerá um aumento no diâmetro da
aliança.
5-(V) Já que o mesmo depende do volume
inicial e da natureza do gás. (Resposta C)
39) (UFC – CE) Numa experiência de
laboratório, sobre dilatação superficial, foram
feitas várias medidas das dimensões de uma
superfície S de uma lâmina circular de
vidro em função da temperatura T. Os
resultados das medidas estão representados no
gráfico abaixo.
Com base nos dados experimentais fornecidos
no gráfico, pode-se afirmar corretamente que o
valor numérico do coeficiente de dilatação
linear do vidro é:
a) 24x10–6 o
C–1
.
b) 18x10–6 o
C–1
.
c) 12x10–6 o
C–1
.
d) 9x10–6 o
C–1
.
e) 6x10–6 o
C–1
Resolução: ∆A = A0.β.∆  (25,00180 –
25,00000) = 25. β.(34 – 30)  β = 18,1.10–6
e
sendo β = 2α temos α = 0,05.10–6
ºC–1
(Resposta D)
40) (UFRN – RN) O dispositivo mostrado na
figura abaixo é utilizado em alguns laboratórios
escolares, para determinar o coeficiente de
dilatação linear de um sólido. Nesse dispositivo,
o sólido tem a forma de um tubo de
comprimento L0, inicialmente a temperatura
ambiente, no qual se faz passar vapor de água
em ebulição até que o tubo atinja a temperatura
do vapor ao entrar em equilíbrio térmico com
este. Há, no dispositivo, dois termômetros, TE1
e TE2, e um micrômetro, MI.
Face ao acima exposto, é correto afirmar que,
para a determinação do coeficiente de dilatação
linear desse tubo,
10
a) tanto o termômetro TE1 como o TE2 medem
a variação de temperatura do tubo, e o
micrômetro mede o comprimento inicial do
tubo.
b) o termômetro TE1 mede a temperatura
ambiente, o termômetro TE2 mede a
temperatura do vapor, e o micrômetro mede a
variação de comprimento do tubo.
c) o termômetro TE1 mede a temperatura do
vapor, o termômetro TE2 mede a temperatura
ambiente, e o micrômetro mede o comprimento
final do tubo.
d) tanto o termômetro TE1 como o TE2 medem
a variação de temperatura do tubo, e o
micrômetro mede a variação de comprimento do
tubo
Resolução: Os termômetros indicarão a
variação de temperatura do tubo em relação a do
ambiente e o micrômetro medirá a dilatação do
tubo. (Resposta E)
41) (UEBA – BA) Uma peça de zinco é
construída a partir de uma chapa quadrada de
lado 30cm, da qual foi retirado um pedaço de
área de 500cm². Elevando-se de 50°C a
temperatura da peça restante, sua área final, em
cm², será mais próxima de: Dado: coeficiente de
dilatação linear do zinco = 2,5 . 10–5
°C–1
a) 400
b) 401
c) 405
d) 408
e) 416
Resolução: A peça restante terá área inicial de
30.30 – 500 = 400 cm². A dilatação será ∆A =
A0.β.∆  ∆A = 400.(2. 2,5 . 10–5
).50 = 1cm²
e assim a área final da peça será 401 cm².
(Resposta B)
42) (UNESP – SP) Duas lâminas metálicas, a
primeira de latão e a segunda de aço, de mesmo
comprimento à temperatura ambiente, são
soldadas rigidamente uma à outra, formando
uma lâmina bimetálica, conforme a figura.
O coeficiente de dilatação térmica linear do
latão é maior que o do aço. A lâmina bimetálica
é aquecida a uma temperatura acima da
ambiente e depois resfriada até uma temperatura
abaixo da ambiente. A figura que melhor
representa as formas assumidas pela lâmina
bimetálica, quando aquecida (forma à esquerda)
e quando resfriada (forma à direita), é
Resolução: Ao ser aquecido, o latão dilata mais
que o aço e ao ser resfriado contrai mais.
(Resposta C)
43) (CESGRANRIO – RJ) Um bloco de certo
metal tem seu volume dilatado de 200 cm³ para
206 cm³ quando sua temperatura aumenta de 20
ºC para 520 ºC. Se um fio desse mesmo metal,
tendo 100 cm de comprimento a 20ºC for
aquecido até a temperatura de 520 ºC, então seu
comprimento em centímetros passará a valer:
a) 101.
b) 102.
c) 103.
d) 106.
e) 112.
Resolução: ∆V = V0.γ. ∆  (206 – 200) =
2000.(3α).(520 – 20)  α = 2.10–5
°C–1
e ∆L =
L0.α.∆  ∆L = 100. 2.10–5
.500 = 1cm.
Assim L = 101 cm. (Resposta A)
44) (UFMT – MT – Modificado) Uma peça
retangular maciça de alumínio encaixa-se
perfeitamente em um buraco feito em outra
chapa de alumínio. Analise as afirmativas e dê
como resposta a soma dos números que
antecedem as afirmações corretas:
(01) Se a peça e a chapa forem aquecidas até
uma mesma temperatura, a peça passará a não
se encaixar perfeitamente no buraco da chapa.
(02) Se somente a chapa for aquecida, a peça se
encaixará com folga no buraco da chapa.
(04) Se somente a chapa for resfriada, a peça
passará a não se encaixar no buraco da chapa.
(08) Se somente a peça for resfriada, então ela
passará a se encaixar com folga no buraco da
chapa.
(16) Se somente a peça for aquecida, então ela
passará a não se encaixar no buraco da chapa.
Resolução:
01 (F) A dilatação diametral será a mesma.
02 (V) Pois o diâmetro da chapa irá aumentar.
04 (V) Pois seu diâmetro será reduzido em
relação ao da peça.
08 (V) Pois seu diâmetro se tornará menor que o
da chapa.
16 (V) Seu diâmetro se tornará maior que o da
chapa.
11
(Resposta: 30 (02 + 04 + 08 + 16))
45) (ACAFE – SC) Um fio, de coeficiente linear
= 2x10-5
ºC-1
, tem comprimento igual a
160cm. Esse fio é dobrado de modo a formar
um quadrado e suas extremidades são soldadas.
Após sofrer um aumento de temperatura de
250ºC, a área interna desse quadrado:
2
.
b) 2
.
2
.
2
.
Resolução: A área inicial do quadrado (lado
160 cm/4 = 40 cm) será 40.40 = 1600 cm². O
perímetro do novo quadrado será: ∆L = L0.α.∆
 ∆L = 160. 2.10–5
.250  ∆L = 0,8 cm. (L =
160,8/4 = 40,2 cm. A nova área do quadrado
fica 40,2² = 1616,04 cm². Assim o acréscimo de
área é 1616,04 – 1600 = 16,04 cm².(Resposta
B)
46) (UNILASALLE – SP) Uma técnica
empregada por algumas indústrias para
aumentar a pressão sobre parafusos de fixação é
a de utilizar parafusos com bitola levemente
superior à do orifício de fixação. Nesse caso,
aquece-se a placa onde será fixado o parafuso.
Isso faz com que o orifício se dilate e o parafuso
possa ser colocado. Após o sistema atingir a
temperatura ambiente, o parafuso fica
praticamente preso ao orifício. Suponha que um
parafuso de aço de diâmetro 1,0000 cm deve ser
fixado num orifício circular de 0,9978 cm de
diâmetro, em uma placa também de aço. A
variação mínima na temperatura da placa, para
que o parafuso possa ser fixado, é a seguinte
(dado: αaço = 11.10-6
ºC-1
)
a) 400 ºC.
b) 200 ºC.
c) 100 ºC.
d) 75 ºC.
e) 50 ºC.
Resolução: ∆L = L0.α.∆  (0,9978 – 1,0000)
= 0,9978. 11.10-6
. ∆  ∆ = -200 ºC.
(Resposta B)
47) (UFPI – PI) O comportamento de dilatação
anômalo da água é verificado pelo fato de esta
se contrair quando sua temperatura aumenta de
0 °C a 4 °C. A partir desta temperatura, a água
se dilata como a maioria das substâncias.
Com relação a esse assunto, coloque V, para
verdadeiro, ou F, para falso.
1 ( ) Em regiões em que a temperatura atinge
valores abaixo de 0°C, os lagos ficam
congelados a partir da superfície. Isso ocorre
porque a água da superfície, ao se resfriar até
4°C, atinge sua densidade máxima, e por
convecção, vai para o fundo.
2 ( ) Numa região de clima muito frio, onde a
temperatura ambiente chega a 0°C ou menos, a
água mais fria por ser mais densa, desce para o
fundo do lago, ocorrendo o congelamento
inicialmente no fundo.
3 ( ) Numa região de clima muito frio, quando a
temperatura das superfícies dos lagos se tornam
inferior a 4°C, não ocorrerá mais movimentação
da água por diferença de densidade. Assim,
forma-se gelo na superfície quando a
temperatura ambiente atingir 0°C, sendo que a
água do fundo do lago continuará líquida.
4 ( ) Quando a temperatura ambiente de regiões
muito frias (onde a temperatura chega abaixo de
0°C) está subindo, o nível da água dos lagos
subirá somente a partir do momento em que a
temperatura da água passar de 4°C.
Resolução:
1 (V) a água mais densa tende a ir para o fundo.
2(F) no fundo a água encontra-se a 4 ºC.
Portanto o congelamento ocorrerá
primeiramente na superfície.
3(V). A água mais densa e mais quente
permanecerá no fundo.
4(V) A dilatação da água torna-se normal após 4
ºC .
(Respostas: 1(V); 2(F); 3(V); 4(V).
48) (ENEM) Durante uma ação de fiscalização
em postos de combustíveis, foi encontrado um
mecanismo inusitado para enganar o
consumidor. Durante o inverno, o responsável
por um posto de combustível compra álcool por
R$ 0,50/litro, a uma temperatura de 5 °C. Para
revender o líquido aos motoristas, instalou um
mecanismo na bomba de combustível para
aquecê-lo, para que atinja a temperatura de 35
°C, sendo o litro de álcool revendido a R$ 1,60.
Diariamente o posto compra 20 mil litros de
álcool a 5 ºC e os revende.
Com relação à situação hipotética descrita no
texto e dado que o coeficiente de dilatação
volumétrica do álcool é de 1×10-3
ºC-1
,
desprezando-se o custo da energia gasta no
aquecimento do combustível, o ganho
financeiro que o dono do posto teria obtido
devido ao aquecimento do álcool após uma
semana de vendas estaria entre
a) R$ 500,00 e R$ 1.000,00.
b) R$ 1.050,00 e R$ 1.250,00.
c) R$ 4.000,00 e R$ 5.000,00.
d) R$ 6.000,00 e R$ 6.900,00.
e) R$ 7.000,00 e R$ 7.950,00.
Resolução: Em 1 semana temos 20000L.7 =
140000L. A dilatação será: ∆V = V0.γ. ∆ =
140000. 1×10-3
. (35 – 5)  ∆V = 4200 L. O
ganho financeiro será 4200. R$ 1,10 =
R$4620,00. (Resposta C)
12
49) A gasolina é vendida por litro, mas em sua
utilização como combustível, a massa é o que
importa. Um aumento da temperatura do
ambiente leva a um aumento no volume da
gasolina. Para diminuir os efeitos práticos dessa
variação, os tanques dos postos de gasolina são
subterrâneos. Se os tanques não fossem
subterrâneos:
I. Você levaria vantagem ao abastecer o carro na
hora mais quente do dia pois estaria comprando
mais massa por litro de combustível.
II. Abastecendo com a temperatura mais baixa,
você estaria comprando mais massa de
combustível para cada litro.
III. Se a gasolina fosse vendida por kg em vez
de por litro, o problema comercial decorrente da
dilatação da gasolina estaria resolvido.
Destas considerações, somente
a) I é correta.
b) II é correta.
c) III é correta.
d) I e II são corretas.
e) II e III são corretas.
Resolução:
I(F) o combustível estaria dilatado.
II(V) o combustível estaria comprimido.
III(V) PIS dilatado ou comprimido a massa do
combustível não se alteraria com a dilatação.
(Resposta E)
50) (ITA – SP) Um pequeno tanque,
completamente preenchido com 20,0 litros de
gasolina a 0°F, é logo a seguir transferido para
uma garagem mantida à temperatura de 70°F.
Sendo  = 0,0012 °C–1
o coeficiente de
expansão volumétrica da gasolina, a alternativa
que melhor expressa o volume de gasolina em
litros que vazará em conseqüência do seu
aquecimento até a temperatura da garagem é
a) 0,507 b) 0,940 c) 1,68
d) 5,07 e) 0,17
Resolução: ∆ = (5/9).70 = 38,88 ºC. Assim
∆V = V0.γ. ∆ = 20.0,0012.38,88  0,940 L.
(Resposta B)
51) (PUC – RS) Um vendedor de gasolina
colocou 20,0 x 10³ litros de gasolina no tanque
de seu caminhão, à temperatura de 15,0 o
C.
Supondo que ele tenha vendido toda a gasolina
à temperatura de 35,0 o
C, e que o coeficiente de
dilatação volumétrica dessa gasolina seja igual a
1,00 x 10-3 o
C-1
, o acréscimo de volume, em
litros, devido à expansão térmica, foi
a) 20
b) 40
c) 100
d) 200
e) 400
Resolução: ∆V = V0.γ. ∆ = 20.10³. 1,00 x 10-3
.
(35 – 15) = 400 L. (Resposta E)
52) (UFF-RJ) O dono de um posto de gasolina
consulta uma tabela de coeficientes de dilatação
volumétrica obtendo γálcool = 10–3
°C–1
.
Assim, ele verifica que se comprar 14 000 litros
do combustível em um dia em que a
temperatura do álcool é de 20°C e revendê-los
num dia mais quente, em que esta temperatura
seja de 30°C, estará ganhando:
a) 1,4 . 10² litros.
b) 1,4 . 10³ litros.
c) 5,2 . 10³ litros.
d) 1,5 . 104
litros.
e) 5,2 . 104
litros.
Resolução: ∆V = V0.γ. ∆ = 14000. 10–3
. (30 –
20) = 140 L. (Resposta A)
53) Um recipiente tem, a 0 o
C, capacidade
volumétrica de 20cm³ e a 100 °C sua capacidade
é de 20,01cm³. Quando ele é completamente
preenchido com certo líquido a 0°C,
transbordam 0,05cm³ ao ser feito o referido
aquecimento. Determine: o coeficiente de
dilatação térmica volumétrica real do líquido.
Resolução: O coeficiente de dilatação do frasco
é ∆V = V0.γ. ∆  (20,01 – 20) = 20. γ.(100 –
0)  γF = 0,5. 10-5
ºC-1
. O coeficiente aparente
é ∆V = V0.γ. ∆  0,05 = 20.γ.(100 – 0) 
γAP = 0,25.10-5
ºC-1
O coeficiente real do líquido será γR = γF + γAP =
0,5. 10-5
+ 0,25.10-5
= 0,75.10-5
ºC-1
(Resposta:
0,75.10-5
ºC-1
)
Calorimetria
54) (UFRRJ – RJ) Uma pessoa bebe 200 gramas
de água a 20°C. Sabendo-se que a temperatura
do seu corpo é praticamente constante e vale
36,5 °C, a quantidade de calor absorvido pela
água é igual a: (calor específico da água =
1cal/g°C)
a) 730cal.
b) 15600cal.
c) 3300cal.
d) 1750cal
e) 0,01750cal.
Resolução: Q = m.c.∆ = 200.1.(36,5 – 20) =
3300 cal. (Resposta C)
55) (UNILASALLE – SP) Para tomar
chimarrão, aquecem-se 500 g de água a 20 ºC
até 80 ºC. A quantidade de energia fornecida à
água, desprezando-se perdas, é a seguinte (dado:
calor específico da água, c = 1 cal/gºC e 1caloria
vale aproximadamente 4,0 J)
a) 120 kJ.
b) 120 MJ.
13
c) 120 J.
d) 100 kJ.
e) 100 J.
Resolução: Q = m.c.∆ = 500.1.(80 – 20) =
30000 cal x 4 = 120000L = 120 kJ. (Resposta
A)
56) (UERJ – RJ) Considere X e Y dois corpos
homogêneos, constituídos por substâncias
distintas, cujas massas correspondem,
respectivamente, a 20 g e 10 g.
O gráfico abaixo mostra as variações da
temperatura desses corpos em função do calor
absorvido por eles durante um processo de
aquecimento.
Determine as capacidades térmicas de X e Y e,
também, os calores específicos das substâncias
que os constituem.
Resolução: CX = Q/∆ = 80/(281 – 273) = 10
cal/K ou 10cal/ºC e CY = Q/∆ = 40/(283 – 273)
= 4 cal/k ou 4 cal/ºC.
cX = CX/m = 10/20 = 0,5 cal/gºC e cY = CY/m =
4/10 = 0,4 cal/gºC.
(Resposta: capacidades térmicas CX =
10cal/ºC e CY = 4 cal/ºC; calores específicos
cX = 0,5 cal/gºC e cY = 0,4 cal/gºC)
57) (PUC – RS) Responder à questão seguinte
com base no gráfico a seguir, referente à
temperatura em função do tempo, de um corpo
que está sendo aquecido e que absorve 20cal/s.
A capacidade térmica do corpo é
a) 20 cal/ºC
b) 30 cal/ºC
c) 40 cal/ºC
d) 50 cal/ºC
e) 60 cal/ºC
Resolução: C = Q/∆ = (20.100)/(60 – 20) = 50
cal/ºC. (Resposta D)
58) (UNESP – SP) O gráfico representa a
temperatura em função do tempo de um líquido
aquecido em um calorímetro.
Considerando-se desprezível a capacidade
térmica do calorímetro e que o aquecimento foi
obtido através de uma resistência elétrica,
dissipando energia à taxa constante de 120 W, a
capacidade térmica do líquido vale:
a) 12 J/o
C.
b) 20 J/o
C.
c) 120 J/o
C.
d) 600 J/o
C.
e) 1 200 J/o
C.
Resolução: 120 W = 120 J/s e C = Q/∆ =
(120.300)/(54 – 24)= 10/30 = 1200 J/ o
C.
(Resposta E)
59) (UFRN – RN) Diariamente, Dona
Leopoldina coloca uma lata de refrigerante, cuja
temperatura é de 30 ºC, numa caixa térmica
contendo gelo e, após esperar algumas horas,
bebe o refrigerante a uma temperatura de
aproximadamente 5 ºC.
Nesse caso, é correto afirmar que a diminuição
da temperatura do refrigerante se explica
porque, no interior da caixa térmica, a lata de
refrigerante
a) cede calor para o gelo, e este cede calor para
ela, porém numa quantidade menor que a
recebida.
b) recebe frio do gelo, para o qual cede calor,
porém numa quantidade menor que o frio
recebido.
c) cede calor para o gelo, e este cede calor para
ela, porém numa quantidade maior que a
recebida.
d) recebe frio do gelo, para o qual cede calor,
porém numa quantidade maior que o frio
recebido.
Resolução: O fluxo do calor se dá nos dois
sentidos, sendo maior do refrigerante (maior
temperatura) para o gelo (menor temperatura).
(Resposta A)
60) (UFMS – MS) Quando a temperatura de
100g de água (calor específico = 4,2 J/(g.K)
diminui de 25ºC para 5ºC, há liberação de calor.
14
É correto afirmar que a quantidade de calor
liberada pela água é;
a) 8 400 Joule.
b) – 8 400 Joule.
c) 2 000/4,2 Joule.
d) – 2 000/4,2 Joule.
e) – 84/100 Joule.
Resolução: Q = m.c.∆ = 100. 4,2 (5 – 20) = –
8400 J. (Resposta B)
61) (UERJ – RJ) Um adulto, ao respirar durante
um minuto, inspira, em média, 8,0 litros de ar a
20 ºC, expelindo-os a 37 ºC.
Admita que o calor específico e a densidade do
ar sejam, respectivamente, iguais a 0,24 cal.g-1
.
ºC-1
e 1,2 g/L-1
.
Nessas condições, a energia mínima, em
quilocalorias, gasta pelo organismo apenas no
aquecimento do ar, durante 24 horas, é
aproximadamente igual a:
a) 15,4
b) 35,6
c) 56,4
d) 75,5
Resolução: A massa de ar em 1 minuto será d =
m/v  1,2 = m/8  m = 9,8 g. Em 24 horas
teremos 24.60.9,6 = 13824 g. Assim Q = m.c.∆
= 13824. 0,24. (37 – 20) = 56401 cal ou  56,4
kcal. (Resposta C)
62) (CEFET – PR) Um pequeno aquecedor
elétrico de imersão é usado para aquecer 100 g
de água para uma xícara de café instantâneo. O
aquecedor está rotulado com “200 W”, o que
significa que ele converte energia elétrica em
energia térmica com essa taxa. Calcule o tempo
necessário para levar toda essa água de 20ºC
para 100ºC, ignorando quaisquer perdas.
(Considerar cágua = 1cal/gºC e 1 cal = 4,19 J)
a) 40 s.
b) 20 s.
c) 400 s.
d) 167,6 s.
e) 1676 s.
Resolução: Q = m.c.∆ = 100.4,19.(100 – 20) =
33520 J. Assim Pot = Q/∆t  200 = 35520/∆t
 ∆t = 167,6 s. (Resposta D)
63) (PUCCAMP – SP) Admita que o corpo
humano transfira calor para o meio ambiente na
razão de 2,0kcal/min. Se esse calor pudesse ser
aproveitado para aquecer água de 20°C até
100°C, a quantidade de calor transferida em 1
hora aqueceria uma quantidade de água, em kg,
igual a: (calor específico da água = 1,0kcal/kg
°C)
Resolução: Em 1 hora teremos Q = 2.60 = 120
kcal. Assim: Q = m.c.∆  120 = m.1.20 m
= 6 kg. (Resposta: 6 kg)
64) (MACK – SP) Na festa de seu aniversário, o
aniversariante comeu salgadinhos e bebeu
refrigerantes, ingerindo o total de 2 000 kcal.
Preocupado com o excesso alimentar, ele
pensou em perder as “calorias” adquiridas,
ingerindo água gelada a 12 ºC, pois, estando o
interior de seu organismo a 37 ºC, as “calorias”
adquiridas seriam consumidas para aquecer a
água.
Admitindo que seu raciocínio esteja correto, o
volume de água a 12 ºC que deve beber é:
a) 60 litros.
b) 65 litros.
c) 70 litros.
d) 80 litros.
e) 90 litros.
Resolução: Q = m.c.∆  2000000= m.1. (37
– 12)  m = 80000g o que corresponde a 80 L.
(Resposta D)
65) (UNESP – SP) Massas iguais de cinco
líquidos distintos, cujos calores específicos
estão dados na tabela adiante, encontram-se
armazenadas, separadamente e à mesma
temperatura, dentro de cinco recipientes com
boa isolação e capacidade térmica desprezível.
Se cada líquido receber a mesma quantidade de
calor, suficiente apenas para aquecê-lo, mas sem
alcançar seu ponto de ebulição, aquele que
apresentará temperatura mais alta, após o
aquecimento, será:
a) a água.
b) o petróleo.
c) a glicerina.
d) o leite.
e) o mercúrio.
Resolução: O de menor calor específico sofrerá
maior aquecimento já que cada grama
necessitará de menos calor para variar sua
temperatura em 1 grau. Portanto, em ordem
decrescente de temperaturas teremos mercúrio;
petróleo; glicerina; leite; água. (Resposta E)
66) (ENEM) A energia geotérmica tem sua
origem no núcleo derretido da Terra, onde as
temperaturas atingem 4.000 ºC. Essa energia é
primeiramente produzida pela decomposição de
materiais radiativos dentro do planeta.
Em fontes geotérmicas, a água, aprisionada em
um reservatório subterrâneo, é aquecida pelas
15
rochas ao redor e fica submetida a altas
pressões, podendo atingir temperaturas de até
370 ºC sem entrar em ebulição. Ao ser liberada
na superfície, à pressão ambiente, ela se
vaporiza e se resfria, formando fontes ou
gêiseres. O vapor de poços geotérmicos é
separado da água e é utilizado no
funcionamento de turbinas para gerar
eletricidade. A água quente pode ser utilizada
para aquecimento direto ou em usinas de
dessalinização.
Roger A. Hinrichs e Merlin Kleinbach. Energia e meio
ambiente. Ed. ABDR (com adaptações).
Depreende-se das informações acima que as
usinas geotérmicas
a) utilizam a mesma fonte primária de energia
que as usinas nucleares, sendo, portanto,
semelhantes os riscos decorrentes de ambas.
b) funcionam com base na conversão de energia
potencial gravitacional em energia térmica.
c) podem aproveitar a energia química
transformada em térmica no processo de
dessalinização.
d) assemelham-se às usinas nucleares no que diz
respeito à conversão de energia térmica em
cinética e, depois, em elétrica.
e) transformam inicialmente a energia solar em
energia cinética e, depois, em energia térmica.
Resolução: A energia térmica resultante das
reações nucleares é transformada em cinética e
em seguida em elétrica, como nas usinas
nucleares. (Resposta D)
67) (FUVEST – SP) Um fogão, alimentado por
um botijão de gás, com as características
descritas no quadro abaixo, tem em uma de suas
bocas um recipiente com um litro de água que
leva 10 minutos para passar de 20 ºC a 100 ºC.
Para estimar o tempo de duração de um botijão,
um fator relevante é a massa de gás consumida
por hora. Mantida a taxa de geração de calor das
condições acima, e desconsideradas as perdas de
calor, a massa de gás consumida por hora, em
uma boca de gás desse fogão, é
aproximadamente
a) 8 g
b) 12 g
c) 48 g
d) 320 g
e) 1920 g
Resolução: 1L contém 1kg de água. Assim, em
10 minutos temos Q = m.c.∆ = 1.1.(100 – 20)
= 80 kJ e como o calor de combustão é 40000
kJ/kg, temos 40000 = 80/m  m = 0,002 kg ou
2 g e em 1 hora fica 2 x 60 min/10 min = 12 g.
(Resposta B)
68) (UERJ – RJ) O gráfico abaixo representa o
consumo de oxigênio de uma pessoa que se
exercita, em condições aeróbicas, numa
bicicleta ergométrica. Considere que o
organismo libera, em média, 4,8 kcal para cada
litro de oxigênio absorvido.
A energia liberada no período entre 5 e 15
minutos, em kcal, é:
a) 48,0
b) 52,4
c) 67,2
d) 93,6
Resolução: Q = 1,4 (15 – 5) = 14 L x 4,8 = 67,2
kcal. (Resposta C)
69) (UERJ – RJ) O excesso de gordura no
organismo é nocivo à saúde. Considere uma
pessoa, com massa corporal estável, que deseje
perder gordura, sem alterar sua dieta alimentar.
Para essa pessoa, um dispêndio energético de 9
kcal em atividades físicas corresponde à perda
de 1 g de gordura corporal.
Para perder 6,0 kg de gordura, o tempo, em
minutos, que ela necessita dedicar a atividades
físicas, despendendo, em média, 12 kcal/min,
corresponde a:
a) 2,0. 10²
b) 4,5.10³
c 4
d) 6,0.105
Resolução: A perda de 6000 g de gordura
requer um dispêndio energético de 6000.9 =
54000 kcal. Assim, ∆t = 54000/12 = 4500
minutos (Resposta B)
70) (MACK – SP – Modificado) A preocupação
com a qualidade e com o tempo de vida leva o
homem moderno urbano a manter uma dieta
alimentar adequada, acompanhada de exercícios
físicos corretos.
- Considere que, para Pedro, o valor energético
adequado, consumido por almoço, seja de 700
kcal de alimentos.
- Considere, ainda, que Pedro, no almoço, ingira
100 g de arroz, 100 g de feijão, 150 g de bife e
16
50 g de batata frita, além de uma lata de
refrigerante.
Consultando a tabela, pode-se afirmar que, para
consumir o excesso energético ingerido, Pedro
deve correr, aproximadamente:
Alimento Energia kcal/g
Arroz 3,6
Feijão 3,4
Bife 3,8
Batata frita 1,4
Considere:
1) Lata de refrigerante 48 kcal
2) Energia consumida em corrida 1080
kcal/hora
a) 80 minutos.
b) 40 minutos.
c) 34 minutos.
d) 60 minutos.
e) 92 minutos.
Resolução: A energia será 100.3,6 + 100.3,4 +
150.3,8 + 50.1,4 + 48 = 360 + 340 + 570 + 48 =
1318 kcal. O excesso é 1318 – 700 = 618 kcal.
Assim, ∆t = 618/1080x 60 min  34 min =
(Resposta C)
71) (UFF – RJ) Para se resfriar um motor em
funcionamento, é necessário acionar seu sistema
de refrigeração, podendo-se usar as substâncias
ar ou água.
A massa de ar m1 e a massa de água m2 sofrem
a mesma variação de temperatura e
proporcionam a mesma refrigeração ao motor.
Neste caso, a razão m1/m2 é:
Dados: calor específico da água = 1,0 cal/g o
C
calor específico do ar = 0,25 cal/g o
C
a) 4,0
b) 0,67
c) 2,0
d) 0,25
e) 1,0
Resolução: Q1 = Q2  m1.c1.∆ = m2.c2.∆
 m1. 1 = m2.0,25  m1. / m2 = 0,25. (Resposta
D)
72) (UFPI – PI) Para perfurar um buraco num
bloco de cobre, de 1,00 kg de massa, um
operário utilizou uma furadeira de potência de
200W durante 200s . Supondo-se que somente
70% da energia envolvida no processo de
perfuração tenha sido absorvida pelo bloco na
forma de calor, o aumento médio da
temperatura do bloco foi de, aproximadamente:
Dados: o calor específico do cobre é
0,093cal/go
C e 1cal = 4,2 J.
a) 30,7 o
C.
b) 71,7 o
C.
c) 1,02×10² o
C.
d) 9,30×10² o
C.
e) 9,52×10³ o
C.
Resolução: Pot = Q/∆t  70%.200 = Q/200
 Q = 28000 J. Sendo Q = m.c.∆ fica 28000
= 1000. 0,093.4,2.∆  ∆  71,7 ºC.
(Resposta B)
73) (UERJ – RJ) Duas barras metálicas A e B,
de massas mA=100 g e mB=120 g, inicialmente à
temperatura de 0 ºC, são colocadas, durante 20
minutos, em dois fornos. Considere que toda a
energia liberada pelas fontes térmicas seja
absorvida pelas barras.
O gráfico a seguir indica a relação entre as
potências térmicas fornecidas a cada barra e o
tempo de aquecimento.
Após esse período, as barras são retiradas dos
fornos e imediatamente introduzidas em um
calorímetro ideal.
O diagrama abaixo indica a variação da
capacidade térmica de cada barra em função de
sua massa.
A temperatura que corresponde ao equilíbrio
térmico entre as barras A e B é, em ºC,
aproximadamente igual a:
a) 70
b) 66
c) 60
d) 54
Resolução: As quantidades de calor fornecidas
a cada barra em 20 min serão dadas através das
áreas do 1º diagrama. Assim: QA = 600.20/2 =
6000 cal e QB = 240.20/2 = 2400 cal. As
temperaturas iniciais das barras serão, portanto:
CA = QA/∆A  80 = 6000/∆A  ∆A = 75
ºC e CB = QB/∆B  48 = 2400/∆B  ∆B =
50 ºC. O equilíbrio térmico se dará a:
mA.cA.∆A + mB.cB.∆ B = 0  80( – 75) +
17
48( – 50)   = 65,625 ºC.
(Resposta B)
74) (UFF – RJ) Duelo de Gigantes:
O rio Amazonas é o maior rio do mundo em
volume d’água com uma vazão em sua foz de,
aproximadamente, 175 milhões de litros por
segundo.
A usina hidroelétrica de Itaipu também é a
maior do mundo, em operação. A potência
instalada da usina é de 12,6.109
W. Suponha que
toda essa potência fosse utilizada para aquecer a
água que flui pela foz do rio Amazonas, sem
que houvesse perdas de energia.
Veja, 24/09/ 2003. (Adaptado)
Nesse caso, a variação de temperatura dessa
água, em grau Celsius, seria da ordem de:
Dados:
calor específico da água c = 1,0 cal/g o
C,
densidade da água = 1,0 g/cm³ e 1 cal = 4,2
Joules
a) 10–2
b) 10–1
c) 100
d) 101
e) 102
Resolução: Pot = Q/∆t = m.c.∆/∆t 
12,6.109
= 175. 109
.1.∆/1  ∆ = 0,072 =
7,2 . 10–2
ºC. ordem de grandeza 10–1
já que 7,2
> 10 (Resposta B)
75) (UFG – GO) O cérebro de um homem
típico, saudável e em repouso, consome uma
potência de aproximadamente 16 W. Supondo
que a energia gasta pelo cérebro em 1 min fosse
completamente usada para aquecer 10 ml de
água, a variação de temperatura seria de,
aproximadamente,
Densidade da água: 1,0·10³ kg/m³
Calor específico da água: 4,2·10³ J/kg·o
C
a) 0,5 o
C
b) 2 o
C
c) 11 o
C
d) 23 o
C
e) 48 o
C
Resolução: Pot = Q/∆t  16 = Q/60 
Q = 960 J e, sendo Q = m.c.∆, fica 960 =
10.4,2. ∆  ∆ = 22,85 ºC. (Resposta D)
76) (FGV – SP) Os trajes de neopreme, um
tecido emborrachado e isolante térmico, são
utilizados por mergulhadores para que certa
quantidade de água seja mantida próxima ao
corpo, aprisionada nos espaços vazios no
momento em que o mergulhador entra na água.
Essa porção de água em contato com o corpo é
por ele aquecida, mantendo assim uma
temperatura constante e agradável ao
mergulhador. Suponha que, ao entrar na água,
um traje retenha 2,5 L de água inicialmente a
21°C. A energia envolvida no processo de
aquecimento dessa água até 35°C é:
Dados: densidade da água = 1 kg/L e calor
específico da água = 1 cal/(g°C)
a) 25,5 kcal.
b) 35,0 kcal.
c) 40,0 kcal.
d) 50,5 kcal.
e) 70,0 kcal.
Resolução: Q = m.c.∆ = 2500. 1. (35 – 21) =
35000cal ou 35 kcal. (Resposta B)
77) (UNIFESP – SP) Dois corpos, A e B, com
massas iguais e a temperaturas tA = 50°C e tB =
10°C, são colocados em contato até atingirem a
temperatura de equilíbrio. O calor específico de
A é o triplo do de B. Se os dois corpos estão
isolados termicamente, a temperatura de
equilíbrio é:
a) 28°C
b) 30°C
c) 37°C
d) 40°C
e) 45°C
Resolução: QA + QB = 0  mA.cA.∆A +
mB.cB.∆B = 0  m. 3 cB.( – 50) + m.cB.(
– 10) = 0  3 – 150 +  – 10 = 0   = 40
ºC. (Resposta D)
78) (UFRJ – RJ) Três amostras de um mesmo
líquido são introduzidas num calorímetro
adiabático de capacidade térmica desprezível:
uma de12 g a 25 °C, outra de 18 g a 15 °C e a
terceira de 30 g a 5 °C.
Calcule a temperatura do líquido quando se
estabelecer o equilíbrio térmico no interior do
calorímetro.
Resolução: QA + QB + QC = 0  mA.cA.∆A +
mB.cB.∆B + mC.cC.∆C = 0  12.c.( – 25) +
18.c.( – 15) + 30.c.( – 5) = 0   = 12 ºC.
(Resposta: 12 ºC)
79) (FUVEST – SP) Dois recipientes iguais A e
B, contendo dois líquidos diferentes,
inicialmente a 20°C, são colocados sobre uma
placa térmica, da qual recebem
aproximadamente a mesma quantidade de calor.
Com isso, o líquido em A atinge 40°C, enquanto
o líquido em B, 80°C. Se os recipientes forem
retirados da placa e seus líquidos misturados, a
temperatura final da mistura ficará em torno de:
18
a) 45°C
b) 50°C
c) 55°C
d) 60°C
e) 65°C
Resolução: A relação entre as capacidades
térmicas dos líquidos é: CA/CB = (Q/∆A)/
(Q/∆B)  CA/CB = ∆B/∆A  CA/CB = (80
– 20)/(40 – 20) = 3 ou seja CA = 3CB
Na mistura fica QA + QB = 0  mA.cA.∆A +
mB.cB.∆B = 0  CA. ( – 40) + CB. ( – 80)
= 0  3CB. ( – 40) + CB. ( – 80) = 0 
3 – 120 +  – 80 = 0   = 50 ºC. (Resposta
B)
80) (UFMG – MG) Numa aula de Física, o
Professor Carlos Heitor apresenta a seus alunos
esta experiência: dois blocos, um de alumínio e
outro de ferro, de mesma massa e, inicialmente,
à temperatura ambiente, recebem a mesma
quantidade de calor, em determinado processo
de aquecimento.
O calor específico do alumínio e o do ferro são,
respectivamente, 0,90 J/go
C e 0,46 J/go
C.
Questionados quanto ao que ocorreria em
seguida, dois dos alunos, Alexandre e Lorena,
fazem, cada um deles, um comentário:
• Alexandre: “Ao final desse processo de
aquecimento, os blocos estarão à mesma
temperatura.”
• Lorena: “Após esse processo de aquecimento,
ao se colocarem os dois blocos em contato,
fluirá calor do bloco de ferro para o bloco de
alumínio.”
Considerando-se essas informações, é correto
afirmar que:
a) apenas o comentário de Alexandre está certo.
b) apenas o comentário de Lorena está certo.
c) ambos os comentários estão certos.
d) nenhum dos dois comentários está certo.
Resolução: Ao receberem iguais quantidades de
calor, o bloco de ferro fica mais aquecido pois
seu calor específico é menor, sendo que cada
grama desse material necessita de menos calor
para cada grau de variação em sua temperatura.
Assim Alexandre está errado pois o ferro está
mais aquecido. Lorena está certa pois o fluxo de
calor é maior do ferro para o alumínio.
(Resposta B)
81) (INATEL – MG) Calor de combustão é a
quantidade de calor liberada na queima de uma
unidade de massa do combustível. O calor de
combustão do gás de cozinha é 6,0.106
cal/kg.
Calcule o volume de água, em litros, que pode
ser aquecido de 20°C a 100°C com um botijão
de gás de cozinha de 13 kg, admitindo que esse
processo tenha uma eficiência de 40%. Dados:
calor específico da água c = 1cal/g°C, densidade
da água d = 1,0.10³g/L)
Resolução: Em 1 botijão temos CC = Q/m 
6,0.106
= Q/13  Q = 78. 106
cal. Sendo a
eficiência 40% então o calor aproveitado será Q
= 40%.78. 106
= 31,2. 106
cal. A massa de água
será Q = m.c.∆  31,2. 106
= m.1. (100 – 0)
 m = 31,2. 104
g = 312.10³g = 312 kg e sendo
a densidade igual a 1 teremos V = 312 L.
(Resposta: 312 litros)
82) (FUVEST – SP) O processo de
pasteurização do leite consiste em aquecê-lo a
altas temperaturas, por alguns segundos, e
resfriá-lo em seguida. Para isso, o leite percorre
um sistema, em fluxo constante, passando por
três etapas:
I) O leite entra no sistema (através de A), a 5 ºC,
sendo aquecido (no trocador de calor B) pelo
leite que já foi pasteurizado e está saindo do
sistema.
II) Em seguida, completa-se o aquecimento do
leite, através da resistência R, até que ele atinja
80 ºC. Com essa temperatura, o leite retorna a
B.
III) Novamente em B, o leite quente é resfriado
pelo leite frio que entra por A, saindo do
sistema (através de C), a 20ºC.
Em condições de funcionamento estáveis, e
supondo que o sistema seja bem isolado
termicamente, pode-se afirmar que a
temperatura indicada pelo termômetro T, que
monitora a temperatura do leite na saída de B, é
aproximadamente de
a) 20 ºC
b) 25 ºC
c) 60 ºC
d) 65 ºC
e) 75 ºC
Resolução: QCED = m.c.(20 – 80) = -60mc
QREC = m.c. ( – 5) e, sendo QCED +QREC = 0,
fica -60mc + m.c. ( – 5) = 0   – 5 = 60 e 
= 65 ºC. (Resposta D)
83) (UFPR – PR) Um recipiente termicamente
isolado contém 500 g de água na qual se
mergulha uma barra metálica homogênea de
250 g. A temperatura inicial da água é 25,0 °C e
19
a da barra 80,0 °C. Considere o calor específico
da água igual a 1,00 cal/g.°C, o do metal igual a
0,200 cal/g.°C e despreze a capacidade térmica
do recipiente. Com base nesses dados, é correto
afirmar que:
(01) A temperatura final de equilíbrio térmico é
de 52,5 °C.
(02) O comprimento da barra permanece
constante durante o processo de troca de calor.
(04) A temperatura inicial da barra, na escala
kelvin, é de 353 K.
(08) A quantidade de calor recebida pela água é
igual à cedida pela barra.
(16) A energia interna final da água, no
equilíbrio térmico, é menor que sua energia
interna inicial.
Soma = ( )
Resolução:
01(F) QA + QB = 0  mA.cA.∆A + mB.cB.∆B
= 0  500.1.(  - 25) + 250.0,2. ( - 80) = 0
  = 30 ºC.
02(F) Já que a barra diminui sua temperatura
então sofrerá uma contração térmica.
04(V) T(K) = C + 273 = 80 + 273 = 353 K
08(V) Já que não ocorrem trocas de calor com o
meio externo e tampouco com o recipiente.
16(F) Já que a água experimentou uma elevação
em sua temperatura. (Resposta: 12 (04 + 08))
84) (ITA – SP) Colaborando com a campanha
de economia de energia, um grupo de escoteiros
construiu um fogão solar, consistindo de um
espelho de alumínio curvado que foca a energia
térmica incidente sobre uma placa coletora.
O espelho tem um diâmetro efetivo de 1,00m e
70% da radiação solar incidente é aproveitada
para de fato aquecer certa quantidade de água.
Sabemos ainda que o fogão solar demora 18,4
minutos para aquecer 1,00 L de água desde a
temperatura de 20 °C até 100 °C, e que
4,186.10³ J é a energia necessária para elevar a
temperatura de 1,00 L de água de 1,000 K.
Com base nos dados, estime a intensidade
irradiada pelo Sol na superfície da Terra, em
W/m². .Justifique.
Resolução:
A potência solar útil no espelho será: Pot = Q/∆t
 Pot = m.c.∆/∆t =
1kg.418,6J/(kg.K).80K/18,4.60s = 30,33 J/s =
30,33 W. A potência total será 30,33/70% =
43,33 W. assim a intensidade da radiação é: I =
Pot/A = 43,33/,r² = 43,33/.0,5²  55,17 W/m².
(Resposta: 55,17 W/m²)
85) (UFPE – PE) Um certo volume de um
líquido A, de massa M e que está inicialmente a
20ºC, é despejado no interior de uma garrafa
térmica que contém uma massa 2M de um outro
líquido, B, na temperatura de 80ºC. Se a
temperatura final da mistura líquida resultante
for de 40ºC, podemos afirmar que a razão cA/cB
entre os calores específicos das substâncias A e
B vale:
a) 6
b) 4
c) 3
d) ½
e) 1/3
Resolução: QA + QB = 0  mA.cA.∆A +
mB.cB.∆B = 0  M. cA.(40 – 20) + 2M.
cB.(40 – 80) = 0  20cA – 80.cB = 0  cA/cB
= 4. (Resposta B)
86) (ENEM) Nos últimos anos, o gás natural
(GNV: gás natural veicular) vem sendo
utilizado pela frota de veículos nacional, por ser
viável economicamente e menos agressivo do
ponto de vista ambiental.
O quadro compara algumas características do
gás natural e da gasolina em condições
ambiente.
d (g/cm³)
GNV 0,8
Gasolina 738
Apesar das vantagens no uso de GNV, sua
utilização implica algumas adaptações técnicas,
pois, em condições ambientes, o volume de
combustível necessário, em relação ao de
gasolina, para produzir a mesma energia, seria:
a) muito maior, o que requer um motor muito
mais potente.
b) muito maior, o que requer que ele seja
armazenado a alta pressão.
c) igual, mas sua potência será muito menor.
d) muito menor, o que o torna o veículo menos
eficiente.
e) muito menor, o que facilita sua dispersão para
a atmosfera.
Resolução: A baixa densidade do GNV requer
que, para se obter uma massa equivalente a da
gasolina, o volume deverá ser bem maior
requerendo armazenamento sb alta pressão para
reduz[i-lo a ponto de ser armazenado.
(Resposta B)
Mudanças de estados físicos da
matéria
87) (UNESP – SP) Nos quadrinhos da tira, a
mãe menciona as fases da água conforme a
mudança das estações.
20
Entendendo “boneco de neve” como sendo
“boneco de gelo” e que com o termo
“evaporou” a mãe se refira à transição
água/vapor, pode-se supor que ela imaginou a
seqüência gelo/água/vapor/água. As mudanças
de estado que ocorrem nessa seqüência são:
a) fusão, sublimação e condensação.
b) fusão, vaporização e condensação.
c) sublimação, vaporização e condensação.
d) condensação, vaporização e fusão.
Resolução: gelo/água  fusão; água/vapor 
vaporização; vapor/água  condensação.
(Resposta B)
88) (UFRN – RN - Modificado) A existência da
água em seus três estados físicos, sólido, líquido
e gasoso, torna nosso Planeta um local peculiar
em relação aos outros Planetas do Sistema
Solar. Sem tal peculiaridade, a vida em nosso
Planeta seria possivelmente inviável. Portanto,
conhecer as propriedades físicas da água ajuda a
melhor utilizá-la e assim contribuir para a
preservação do Planeta.
Na superfície da Terra, em altitudes próximas
ao nível do mar, os estados físicos da água estão
diretamente relacionados à sua temperatura
conforme mostrado no gráfico abaixo. Esse
gráfico representa o comportamento de uma
massa de 1,0 g de gelo a uma temperatura
inicial de – 50 o
C, colocada em um calorímetro
que, ligado a um computador, permite
determinar a temperatura da água em função da
quantidade de calor que lhe é cedida.
Observando-se o gráfico, pode-se concluir que a
quantidade de calor necessária para elevar a
temperatura do gelo a 0 o
C até água a 100 o
C é:
a) 180 cal.
b) 200 cal.
c) 240 cal.
d) 100 cal
Resolução: Q = 205 – 25 = 180 cal. (Resposta
A)
89) (IFPE – PE) Uma amostra de determinada
substância com massa 30g encontra-se
inicialmente no estado liquido, a 60°C. Está
representada pelo gráfico ao lado a temperatura
dessa substância em função da quantidade de
calor por ela cedida.
Analisando esse gráfico, é correto afirmar que:
a) a temperatura de solidificação da substância é
10°C.
b) o calor específico latente de solidificação é -
1,0 cal/g.
c) o calor específico sensível no estado líquido é
1/3 cal/g°C.
d) o calor específico sensível no estado sólido é
1/45 cal/g°C.
e) ao passar do estado líquido a 60°C para o
sólido a 10°C a substância perdeu 180 cal.
Resolução:
a(F) o gráfico mostra que a temperatura se torna
constante a 30 ºC. É a temperatura de mudança
de estado (solidificação).
b(V) L = Q/m = -(60 – 30)/30 = -1 cal/g
c(F) c = Q/m.∆ = 30/30.30 = 1/30 cal/g°C
d(F) c = Q/m.∆ = 30/30.20 = 1/20 cal/g°C
e(F) o gráfico mostra 90 calorias.
(Resposta B)
90) (UFG – GO) No gráfico, está representada a
variação da temperatura em função do tempo de
uma massa de 200 g d’água. Suponha que a
fonte forneceu energia térmica a uma potência
constante desde o instante t = 0, e que toda essa
energia foi utilizada para aquecer a água. Dado:
c = 1,0 cal/g°C.
Analisando-se o gráfico, pode-se afirmar que
1-( ) no primeiro minuto, não ocorreu absorção
de calor.
21
2-( ) a potência da fonte térmica é de 16
kcal/min.
3-( ) a temperatura da água para t = 2 min é 80
°C.
4-( ) de 0 a 9 minutos, ocorreram três transições
de fase.
Resolução:
1(F) A fonte tinha potência constante. A água
encontrava-se a 0 o
C e no estado sólido.
2(V) Pot = m.c.∆/ ∆t = 200.1.100/1,25 = 16000
cal/min ou 16 kcal/min
3(V) Pot = m.c.∆/∆t  16000 = 200.1./1 
 = 80 o
C.
4(F) ocorreram 2. Fusão e vaporização.
(Resposta: 1-F; 2-V; 3-V; 4-F)
91) (PUC – RS) Muitas pessoas, ao cozinharem,
se preocupam com a economia de gás e adotam
algumas medidas práticas, como:
I. deixar o fogo baixo do início ao fim, pois
assim se obtém cozimento mais rápido;
II. baixar o fogo quando a água começa a ferver,
pois a temperatura permanece constante durante
a ebulição;
III. deixar o fogo alto do início ao fim, obtendo
uma constante elevação de temperatura, mesmo
após o início da ebulição.
Pela análise das afirmativas, conclui-se que
somente
a) está correta a I.
b) está correta a II.
c) está correta a III.
d) estão corretas a I e a III.
e) estão corretas a II e a III.
Resolução:
I-F a água demorará mais tempo para chegar a
sua temperatura de ebulição e ali estabilizar.
II-V a temperatura permanecerá constante dali
por diante, independentemente do fluxo do
calor, apenas mudando de estado físico.
III-F durante a mudança de estado a temperatura
permanece constante. (Resposta B)
92) (UFRN – RN) Cotidianamente são usados
recipientes de barro (potes, quartinhas, filtros
etc.) para esfriar um pouco a água neles contida.
Considere um sistema constituído por uma
quartinha cheia d´água. Parte da água que chega
à superfície externa da quartinha, através de
seus poros, evapora, retirando calor do barro e
da água que o permeia. Isso implica que
também a temperatura da água que está em seu
interior diminui nesse processo.
Tal processo se explica porque, na água que
evapora, são as moléculas de água
a) com menor energia cinética média que
escapam do líquido, aumentando, assim, a
energia cinética média desse sistema.
b) que, ao escaparem do líquido, aumentam a
pressão atmosférica, diminuindo, assim, a
pressão no interior da quartinha.
c) com maior energia cinética média que
escapam do líquido, diminuindo, assim, a
energia cinética média desse sistema.
d) que, ao escaparem do líquido, diminuem a
pressão atmosférica, aumentando, assim, a
pressão no interior da quartinha.
Resolução: A água, ao evaporar, absorve
energia do sistema, resfriando-o. (Resposta C)
93) (ITA – SP) Um pedaço de gelo flutua em
equilíbrio térmico com uma certa quantidade de
água depositada em um balde. À medida que o
gelo derrete, podemos afirmar que
a) o nível da água no balde aumenta, pois
haverá uma queda de temperatura da água.
b) o nível da água no balde diminui, pois haverá
uma queda de temperatura da água.
c) o nível da água no balde aumenta, pois a
densidade da água é maior que a densidade do
gelo.
d) o nível da água no balde diminui, pois a
densidade da água é maior que a densidade do
gelo.
e) o nível da água no balde não se altera.
Resolução: Ao fundir, a parte submersa do gelo
sofre uma contração térmica de mesmo volume
que a parte emersa do gelo. Assim o nível da
água não se altera com o derretimento do gelo.
(Resposta E)
94) (CEFET – PR) Uma estudante colocou em
um recipiente 2,0 litros de água cuja densidade é
1 g/cm³, inicialmente a 20 ºC, para ferver.
Distraindo-se, esqueceu a água no fogo por um
certo tempo e, quando percebeu, metade da água
havia evaporado. Curiosa, desejou saber que
quantidade de calor a água havia consumido no
processo.
Sendo o calor específico e o calor de
vaporização da água, respectivamente, iguais a
1,0 cal/g. ºC e 540 cal/g, encontrou:
a) 700 kcal.
b) 620 kcal.
c) 160 kcal.
d) 540 kcal.
e) 80 kcal.
Resolução: Q = QS + QL = m.c.∆ + m1.LF =
2000.1.(100 – 20) + 1000.540 = 700000cal ou
700 kcal. (Resposta A)
95) (FUVEST – SP) Um recipiente de isopor,
que é um bom isolante térmico, tem em seu
interior água e gelo em equilíbrio térmico. Num
dia quente, a passagem de calor por suas
paredes pode ser estimada, medindo-se a massa
de gelo Q presente no interior do isopor, ao
22
longo de algumas horas, como representado no
gráfico.
Esses dados permitem estimar a transferência de
calor pelo isopor, como sendo,
aproximadamente, de: (Calor latente de fusão do
gelo ≈ 320 kJ/kg)
a) 0,5 kJ/h
b) 5 kJ/h
c) 120 kJ/h
d) 160 kJ/h
e) 320 kJ/h
Resolução: Fluxo = Q/∆t = m.L/∆t = (20 – 8).
320/24 = 160 kJ. (Resposta D)
96) (ACAFE – SC) Patrícia deseja “gelar” um
refrigerante que se encontra à temperatura
ambiente. Para isso, dispõe de dois recipientes:
um com certa massa de água e outro com igual
massa de gelo, ambas a uma temperatura de
0ºC.
Para conseguir o seu intento, da melhor forma
possível, é aconselhável que Patrícia mergulhe o
refrigerante:
a) no gelo, porque inicialmente ele absorve
calor e não aumenta de temperatura.
b) no gelo, porque ele tem maior calor
específico do que a água.
c) na água, porque o equilíbrio térmico se dará a
uma temperatura menor.
d) na água, porque ela tem maior calor
específico que o gelo.
e) no gelo, porque ele contém menos calor do
que a água.
Resolução: Enquanto o gelo estiver derretendo
sua temperatura permanece constante,
abaixando apenas a temperatura do refrigerante.
(Resposta A)
97) (UFES – ES) Quantas calorias são
necessárias para vaporizar 1,00 litro de água, se
a sua temperatura é, inicialmente, igual a 10,0
°C?
Dados:
- calor específico da água: 1,00 cal/g°C;
- densidade da água: 1,00 g/cm³;
- calor latente de vaporização da água: 540
cal/g.
a) 5,40 × 104
cal
b) 6,30 × 105
cal
c) 9,54 × 106
cal
d) 5,40 × 101
cal
e) 6,30 × 10² cal
Resolução: Q = m.c.∆ + m.LV = 1000.1.(100 –
10) + 1000.540 = 90000 + 540000 = 630000cal
ou 6,30 × 105
cal. (Resposta B)
98) (UNICAMP – SP) Em um dia quente, um
atleta corre dissipando 750 W durante 30 min.
Suponha que ele só transfira esta energia para o
meio externo através da evaporação do suor e
que todo o seu suor seja aproveitado para sua
refrigeração. Adote L = 2 500 J/g para o calor
latente de evaporação da água na temperatura
ambiente.
a) Qual é a taxa de perda de água do atleta em
kg/min?
b) Quantos litros de água ele perde em 30 min
de corrida?
Resolução: a) Pot = Q/∆t  750 = Q/30.60
 Q = 1350000 J. A massa de água Q = m.L 
1350000 = m.2500  m = 540 g. A taxa será
540g/30 min = 18g/min = 0,018 kg/min.
b) Em 30 min perde 0,018 x 30 = 0,540 g o que
corresponde a 0,54 litros.
(Respostas: a) 18g/min; b) 0,54 litros)
99) (PUC – RS) Colocam-se 420g de gelo a 0°C
num calorímetro com água a 30°C. Após
atingida a temperatura de equilíbrio térmico,
verifica-se que sobraram 20g de gelo. Sendo de
80cal/g o calor de fusão da água, é correto
afirmar que a temperatura final de equilíbrio
térmico e a quantidade de calor ganho pelo gelo
são, respectivamente,
a) 30°C e 50kcal
b) 30°C e 45kcal
c) 15°C e 40kcal
d) 0°C e 38kcal
e) 0°C e 32kcal
Resolução: O equilíbrio térmico se deu a 0 o
C
já que no recipiente sobraram ainda 20 g de
gelo.
A quantidade de calor absorvida pelo gelo é Q =
m.L = (420 – 20).80 = 32000 cal ou 32 kcal..
(Resposta E)
100) (UERJ – RJ) Uma menina deseja fazer um
chá de camomila, mas só possui 200 g de gelo a
0 o
C e um forno de microondas cuja potência
máxima é 800 W. Considere que a menina está
no nível do mar, o calor latente de fusão do gelo
é 80 cal/g , o calor específico da água é 1
cal/go
C e que 1 cal vale aproximadamente 4
joules.
Usando esse forno sempre na potência máxima,
o tempo necessário para a água entrar em
ebulição é:
a) 45 s.
b) 90 s.
c) 180 s.
d) 360 s.
23
Resolução: Pot = (Q1 + Q2)/∆t  800/4 =
(m.L + m.c.∆)/∆t  200 = (200.80 +
200.1.100)/∆t  ∆t = 180 s. (Resposta C)
101) (UERJ – RJ) Uma bolinha de aço a 120º C
é colocada sobre um pequeno cubo de gelo a 0º
C.
Em escala linear, o gráfico que melhor
representa a variação, no tempo, das
temperaturas da bolinha de aço e do cubo de
gelo, até alcançarem um estado de equilíbrio, é:
Resolução: Inicialmente, enquanto derrete, a
temperatura do gelo se mantém constante. Após
isso a água resultante tem sua temperatura
elevada até o equilíbrio térmico com a bolinha
de aço. (Resposta D)
102) (UERJ – RJ) Quatro esferas metálicas e
maciças, E1 , E2 , E3 e E4 , todas com a mesma
massa, são colocadas simultaneamente no
interior de um recipiente contendo água em
ebulição. A tabela abaixo indica o calor
específico e a massa específica do metal que
constitui cada esfera.
Atingido o equilíbrio térmico, essas esferas são
retiradas da água e colocadas imediatamente na
superfície de um grande bloco de gelo que se
encontra na temperatura de fusão.
A esfera que fundiu a maior quantidade de gelo
e a esfera que produziu a cavidade de menor
diâmetro no bloco de gelo é respectivamente:
a) E3 ; E4
b) E2 ; E4
c) E1 ; E3
d) E1 ; E2
Resolução: Com as esferas aquecidas a 100 ºC,
a de maior calor específico deverá ceder uma
quantidade maior de calor ao gelo até o
equilíbrio térmico. Assim a esfera que forma a
maior cavidade é a E1. Como as massas são
iguais, a esfera que produzirá a cavidade de
menor volume e, conseqüentemente, de menor
diâmetro é a de maior densidade, ou seja, E3.
(Resposta C)
103) (UFGO – GO) Em um copo grande,
termicamente isolado, contendo água à
temperatura ambiente (25ºC), são colocados 2
cubos de gelo a 0ºC. A temperatura da água
passa a ser, aproximadamente, de 1ºC. Nas
mesmas condições se, em vez de 2, fossem
colocados 4 cubos de gelo iguais aos anteriores,
ao ser atingido o equilíbrio, haveria no copo:
a) apenas água acima de 0ºC
b) apenas água a 0ºC
c) gelo a 0ºC e água acima de 0ºC
d) gelo e água a 0ºC
e) apenas gelo a 0ºC
Resolução: A quantidade de calor cedida pelo
gelo será suficiente para reduzir a temperatura
1ºC ficando o sistema estabilizado a 0 o
C com
parte sólida e parte líquida, pois a 0 o
C os
fluxos de calor trocados entre as partes se
tornam iguais, (Resposta D)
104) (UFF – RJ) Gelo seco nada mais é que gás
carbônico (CO2) solidificado e sua aplicação vai
de efeitos especiais em shows à conservação de
alimentos. Tal substância é conhecida desde
meados do século XIX e recebeu esse nome
devido ao fato de não passar pela fusão, quando
submetida à pressão atmosférica e à temperatura
ambiente, como ocorre com o gelo comum.
Considere um cubo de 0,10 kg de gelo seco, a -
78 o
C, e um bloco de gelo comum de 1,0 kg, a -
10 o
C,colocados em um recipiente.
Desprezando a capacidade térmica do recipiente
e a troca de calor com o ambiente:
a) determine a temperatura de equilíbrio
térmico;
b) descreva os elementos que comporão o
sistema no equilíbrio térmico.
Dados:
Temperatura de sublimação do gelo seco = - 78
o
C
Temperatura de fusão do gelo comum = 0 o
C
Calor latente de vaporização do gelo seco = 134
cal/g
Calor específico do vapor de gelo seco = 0,20
cal/g o
C
Calor específico do gelo comum = 0,50 cal/g o
C
Resolução:
24
a) Abaixando a temperatura do gelo comum até
a do gelo seco, temos: Q = m.c.∆ = 1000.0,5. (-
78 – (-10)) = - 34000 cal.
Cedendo pouco a pouco esta quantidade de
calor ao sistema, temos, para sublimar o gelo
seco:
Q = m.LS = 100.134 = 13400cal. Sobram,
portanto, 34000 – 13400 = 20600 cal. No
recipiente temos vapor de gelo seco e gelo
comum, ambos a -78 ºC. Com o calor restante, a
temperatura do sistema fica: Q = m.c.∆ +
m.c.∆  20600 = 1000.0,5. ( – (-78)) +
100.0,2. ( – (-78))  20600 = 500 +
39000 + 20 + 1560  -19960 = 520   =
-38,38 ºC.
b) No recipiente temos vapor de gelo seco e
gelo comum a -38,38 ºC.
(Resposta: a) -38,38 ºC.; b) vapor de gelo seco
e gelo comum)
105) (FATEC – SP - Modificado) Um frasco
contém 20 g de água a 0 °C. Em seu interior é
colocado um objeto de 50 g de alumínio a 80
°C. Os calores específicos da água e do
alumínio são respectivamente 1,0 cal/g°C e 0,10
cal/g°C e o calor latente de vaporização da água
é 540 cl/g.
Supondo não haver trocas de calor com o frasco
e com o meio ambiente, após atingida a
temperatura de equilíbrio, Qual a quantidade de
calor necessária para que toda toda a água do
sistema vaporize?
a) 12,9 kcal
b) 10 kcal
c) 5,4 kcal
d) 12,48 kcal
e) 6,24 kcal.
Resolução: QA + QB = 0  mA.cA.∆A +
mB.cB.∆B = 0  20.1.( - 0 ) + 50.0,1.( -
80) = 0  20 + 5 - 400 = 0   = 16 ºC.
Assim Q = mA.cA.∆A + mA.LV + mB.cB.∆B =
20.1.(100 – 16) + 20.540 + 50.0,1.(100 – 16) =
1680 + 10800 + 420 = 12900 cal ou 12,9 kcal.
(Resposta A)
106) (UFPR – PR) Um esquiador desce, com
velocidade constante, uma encosta com
inclinação de 30° em relação à horizontal. A
massa do esquiador e de seu equipamento é 72
kg. Considere que todo o calor gerado pelo
atrito no movimento seja gasto na fusão da
neve, cujo calor latente de fusão é 3,6 x 105
J/kg, e suponha a aceleração da gravidade igual
a 10 m/s2
. Determine a massa de neve fundida
após o esquiador descer 90 m na encosta.
Expresse o resultado em gramas.
Resolução: A energia dissipada durante a
descida é Q = m.g.h = 72.10.90.sen 30º = 32400
J.
Na descida de 90 metros, a quantidade de gelo
fundida será Q = m.LF  32400 = m. 3,6 x
105
 m = 0,09 kg = 90 g. (Resposta: 90 g)
Complementos de mudanças
de fases (estados físicos)
01) Conhecer os processo de mudanças de
estados físicos da matéria requer alguns
conhecimentos básicos. O primeiro deles é saber
qual a denominação de cada uma das mudanças
e qual a influência da pressão na temperatura de
mudança de estado. Assim, analise as
afirmações seguintes e aponte a única incorreta:
a) Qualquer passagem da fase líquida para a
fase gasosa é denominada evaporação.
b) A passagem da fase gasosa para a fase líquida
é denominada condensação ou liquefação.
c) Fusão é a passagem do estado sólido para o
estado líquido.
d) A passagem do estado gasoso para o estado
sólido recebe o nome de cristalização.
e) A evaporação de um líquido ocorre a uma
temperatura inferior a denominada temperatura
de ebulição.
Resolução: Vaporização é a passagem da fase
líquida para a gasosa e pode ocorrer de três
maneiras distintas; por ebulição, por evaporação
ou por calefação. (Resposta A)
02) (ENEM) A adaptação dos integrantes da
seleção brasileira de futebol à altitude de La Paz
foi muito comentada em 1995, por ocasião de
um torneio, como pode ser lido no texto abaixo.
“A seleção brasileira embarca hoje para La
Paz, capital da Bolívia, situada a 3.700 metros
de altitude, onde disputará o torneio
Interamérica. A adaptação deverá ocorrer em
um prazo de 10 dias, aproximadamente. O
organismo humano, em altitudes elevadas,
necessita desse tempo para se adaptar,
evitando-se, assim, risco de um colapso
circulatório.”
(Adaptado da revista Placar, edição fev.1995)
A adaptação da equipe foi necessária
principalmente porque a atmosfera de La Paz,
quando comparada à das cidades brasileiras,
apresenta:
a) menor pressão e menor concentração de
oxigênio.
b) maior pressão e maior quantidade de
oxigênio.
c) maior pressão e maior concentração de gás
carbônico.
d) menor pressão e maior temperatura.
e) maior pressão e menor temperatura.
25
Resolução: À medida que a altitude aumenta
ocorre uma diminuição na pressão atmosférica e
uma conseqüente diminuição do nível de
oxigênio. (Resposta A)
03) (PUC – RS) A temperatura de fusão de uma
substância depende da pressão que é exercida
sobre a mesma substância. O aumento de
pressão sobre um corpo ocasiona, na sua
temperatura de fusão:
a) um acréscimo, se o corpo, ao se fundir, se
expande.
b) um acréscimo, se o corpo, ao se fundir, se
contrai.
c) um decréscimo, se o corpo, ao se fundir, se
expande.
d) um decréscimo para qualquer substância.
e) um acréscimo para qualquer substância.
Resolução: As substâncias ditas com
comportamento normal (se expandem ao fundir)
sofrem uma aumento em sua temperatura de
fusão e ebulição quando ocorre um aumento de
pressão. (Resposta A)
04) Em um passado recente o Paraná foi o maior
produtora de café do mundo, cultura essa que
promoveu o desenvolvimento acentuado de
várias cidades produtoras no estado. A falta de
subsídios para as lavouras de café, a
insegurança dos agricultores frente às políticas
de preços propostas pelo governo e a
possibilidade de destruição da lavoura de café
frente às geadas levaram a erradicação quase
que total das plantações de café, dando lugar a
outros tipos de lavouras como a soja, o trigo e o
algodão, além da formação de enormes áreas de
pastagens.
A geada, um tormento para os proprietários de
certos tipos de lavouras tem sua formação e
ação explicada a seguir de três modos.
Distintos:
I- O vapor-d'água existente na atmosfera sofre
condensação, precipitando-se, suavemente, nas
plantas, sob a forma de pequenos cristais de
gelo.
II- A água existente no interior das plantas
congela quando a temperatura ambiente atinge
um valor muito pequeno, aumentando seu
volume e rompendo células e vasos que
compõem a planta.
III- O vapor-d'água existente na atmosfera
condensa-se em contato com a planta,
transformando-se em pequenas gotículas
(orvalho). Com a queda de temperatura, o
orvalho congela (solidifica) e, ao fato,
denominamos geada.
a) apenas I é correta;
b) apenas II é correta;
c) apenas III é correta:
d) existem duas afirmações corretas apenas
e) as três afirmações estão corretas.
Resolução: A condensação dos vapores em
contato com a planta e a conseqüente queda de
temperatura ocasiona o congelamento das
gotículas e também da água existente no interior
das plantas. Ao congelar, a água aumenta seu
volume rompendo os vasos, resultando na morte
da planta. (Resposta D)
05) (PUC-RS) Há uma relação entre a pressão e
a temperatura nas quais ocorrem as mudanças
de fase. Assim, é correto afirmar que a
temperatura de:
a) fusão do gelo é superior a 0ºC, quando a
pressão é superior a 1atm.
b) fusão do gelo é inferior a 0ºC, quando a
pressão é superior a 1atm
c) fusão do gelo é sempre 0ºC, independente da
pressão.
c) ebulição da água é inferior a 100ºC, quando a
pressão é superior a 1atm.
e) ebulição da água é sempre 100ºC,
independente da pressão.
Resolução: O diagrama representa as fases da
água. Observe que sob pressão de 1 atm o ponto
de fusão é 0 o
C e o de ebulição, 100 o
C. O
aumento de pressão acarreta uma queda na
temperatura de fusão e um aumento da
temperatura de ebulição. (Resposta B)
06) (UFPI – PI) A experiência do regelo, feita
pela primeira vez pelo físico John Tyndal
(1820-1893), pode facilmente ser reproduzida
da seguinte forma:
- coloque um bloco de gelo a aproximadamente
–2 °C, apoiado sobre dois suportes de madeira,
de modo que a parte central do bloco fique livre;
- prenda pesos nas extremidades de um fio fino
de arame e,
- coloque este fio transversalmente sobre a parte
central do bloco de gelo, deixando os pesos
pendentes.
Observe que o fio de arame atravessa o bloco de
gelo sem que este fique dividido em duas partes.
A explicação para essa observação experimental
é:
a) O arame, estando naturalmente mais
aquecido, funde o gelo, que perde calor para o
ambiente, deixando o bloco novamente sólido.
26
b) O acréscimo de pressão no contato fio-gelo
diminui a temperatura de fusão e produz
derretimento do gelo sob o fio; este, por sua vez,
desloca-se através da água formada, a qual se
congela ao voltar à pressão normal.
c) A pressão exercida pelo arame sobre o gelo
aumenta seu ponto de fusão que, ao ser atingido,
ocorre o degelo. Assim, o fio se desloca através
da água formada, a qual se congela depois que o
fio a atravessa.
d) O ponto de fusão do gelo independe da
pressão do arame sobre ele, o degelo ocorre
simplesmente por causa do contato de
substâncias diferentes, no caso fio-gelo.
Cessado o contato, a água volta a congelar.
e) O acréscimo de pressão sobre o bloco
aumenta a temperatura de fusão e produz
derretimento do gelo sob o fio; este se desloca
através da água formada, a qual perde calor para
o ambiente e congela novamente.
Resolução: O aumento de pressão ocasionado
pelo arame faz com que o gelo sob ele funda
devido a uma diminuição na sua o temperatura
de fusão (no caso inferior a -2º C). A água
resultante desliza sobre o arame retornando a
pressão inicial e congelando imediatamente.
(Resposta B)
07) Você acorda num dia frio, prepara-se e vai
para a escola ou trabalho caminhando,
naturalmente bem agasalhado. Você repara que,
ao respirar, “solta” uma pequena nuvem de
"fumaça" durante a aspiração do ar.
Três possíveis explicações são formuladas para
o fato.
I- Durante a respiração, seu organismo separa o
vapor-d'água do restante dos componentes
aproveitáveis do ar atmosférico. Tal vapor, por
não ser aproveitado, é expelido, o que dá a
impressão da formação de uma pequena “nuvem
de fumaça”.
II- A respiração nos dias frios exige um grande
esforço. A energia necessária para que você
respire vem da queima dos alimentos que
ingere. Como a queima é muito grande, ocorre a
formação de fumaça de vapor cuja quantidade é
muito intensa ao ser expelida.
III- Quando respiramos, aspiramos o ar
atmosférico, expelindo-o em seguida. O vapor-
d'água expelido, em contato com o meio
ambiente, cuja temperatura é menor, a ele cede
calor, condensando-se sob a forma de pequenas
gotículas que, agrupadas, parecem uma pequena
nuvem.
a) apenas I é correta;
b) apenas II é correta;
c) apenas III é correta:
d) existem duas afirmações corretas apenas
e) as três afirmações estão corretas.
Resolução: A “fumaça” é simplesmente o vapor
d’água expelido que condensa sob a forma de
pequenas gotículas formando a “nuvem”.
(Resposta: C)
08) Uma garrafa de vidro cheia de água pode
quebrar, dentro do congelador, por que:
a) a água, ao se congelar, sofre aumento de
volume, e suas moléculas exercem uma força
muito intensa sobre as paredes da garrafa de
vidro.
c) a água quimicamente destrói as paredes do
vidro;
d) a garrafa não pode ter temperatura baixa;
e) o gelo externo à garrafa exerce pressão muito
maior que o gelo interno.
Resolução: O congelamento da água ocasiona
sua expansão durante a formação do gelo. As
moléculas, com isso, exercem forças imensas
nas paredes do recipiente, rompendo-as.
(Resposta A)
09) A figura seguinte indica os diferentes pontos
de ebulição em algumas cidades localizadas em
altitudes diferentes.
a) Qual a relação entre o ponto de ebulição e a
altitude das cidades?
b) Em Santos e em Quito, a água a 0º C se
encontraria em qual estado físico? Justifique.
Resolução: a) À medida que a altitude aumento
ocorre uma queda na pressão e com isso uma
redução na temperatura de ebulição.
b) Em Quito a água está no estado sólido, pois
sua temperatura de fusão é maior que 0 o
C já
que a pressão é menor que a atmosférica. Em
Santos a água pode estar no estado sólido,
líquido ou em transição pois 0 o
C é justamente
sua temperatura de fusão.
(Respostas: a) Diminui com o aumento da
altitude; b) Quito: estado sólido. Santos:
estado sólido, líquido ou em transição)
10) A decomposição de inúmeras rochas que se
fragmentam em pedaços menores deve-se ao
fato de possuírem fendas ou porosidades que
permitem a infiltração de água. Nos países de
clima muito frio, ou quando ocorre grande
queda de temperatura observa-se a
fragmentação mais acentuada.
Explique fisicamente por que isto ocorre.
Resolução: A água infiltrada nas rochas
congela e durante a formação do gelo ocorre sua
27
expansão e o rompimento das fendas e poros.
(Resposta: Devido ao congelamento da água)
11) (UFPI – PI) Podemos conhecer melhor uma
substância através de seu diagrama de fases, que
são curvas que delimitam as regiões
correspondentes às fases sólida, líquida e gasosa
da substância. A figura abaixo mostra o
diagrama de fase da água, líquido vital aos seres
vivos e que é estudado universalmente. Analise
o diagrama de fase mostrado e assinale, nas
afirmativas, V (para verdadeiro) ou F (para
falso).
1( ) Sob pressão de 4,58 mmHg a à temperatura
de 0,01 ºC há a coexistência das três fases da
água em equilíbrio: sólida, líquida e vapor. Esse
ponto é o ponto triplo.
2( ) A curva AT do diagrama representa a curva
de sublimação, cujos pontos representam os
estados de equilíbrio entre as fases sólida e
liquida.
3( ) À pressão de 2,51 mmHg e à temperatura
ambiente de 20 ºC, o diagrama mostra que a
água está no estado de vapor.
4( ) A curva TB do diagrama mostra que a
fusão é facilitada pelo aumento da pressão.
Resolução:
1)(V) No ponto triplo (intersecção das três
curvas) as condições de temperatura e pressão
determinam a coexistência dos três estados
físicos.
2(F) Entre os estados sólido e gasoso (no caso,
vapor).
3(V) o ponto (pxV) estará determinado abaixo
das curvas AT e TC.
4(V) Já que o aumento de pressão ocasiona uma
queda na temperatura de fusão.
(Resposta: 1(V); 2(F); 3(V); 4(V))
12) (UFMS – MS) Uma cozinheira resolve
ferver água em uma panela de pressão, para
atingir maior temperatura que em panelas
abertas. Para isso, coloca água no seu interior,
onde todo o sistema, panela e água, está em
equilíbrio térmico com o ambiente na
temperatura TA e pressão atmosférica pA. Em
seguida, fecha a panela e coloca-a sobre a
chama de um fogão. A água, no interior da
panela, vai aumentando a pressão e a
temperatura e, após certo tempo entra em
ebulição liberando vapores para o ambiente pela
válvula.
As figuras abaixo representam diagramas da
pressão p x temperatura T da água, onde as
linhas mais finas definem as regiões, de
temperatura e pressão em que a água pode se
apresentar em cada um dos possíveis estados,
sólido (S), líquido (L) e vapor (V). Já as linhas
mais grossas (negrito) representam processos
termodinâmicos.
Assinale qual dos diagramas representa
corretamente o processo termodinâmico que a
água, no interior da panela, sofreu desde o
instante em que começou a ser aquecida,
quando estava à temperatura TA e pressão pA,
até o momento em que entra em ebulição à
temperatura TB e Pressão pB.
Resolução: Partindo do ponto pA,TA (estado
líquido) a água vai gradativamente sofrendo
aumento de pressão e temperatura determinando
na curva de ebulição o ponto pB,TB conforme o
diagrama D. (Resposta D)
13) As afirmações a seguir referem às
substâncias que têm comportamento normal, ou
seja, aumentam de volume durante a fusão.
Analise cada uma delas e indicando se é
verdadeira (V) ou falsa (F).
Revisão geral-de-termologia
Revisão geral-de-termologia
Revisão geral-de-termologia
Revisão geral-de-termologia
Revisão geral-de-termologia
Revisão geral-de-termologia
Revisão geral-de-termologia
Revisão geral-de-termologia
Revisão geral-de-termologia
Revisão geral-de-termologia
Revisão geral-de-termologia
Revisão geral-de-termologia
Revisão geral-de-termologia
Revisão geral-de-termologia
Revisão geral-de-termologia
Revisão geral-de-termologia
Revisão geral-de-termologia
Revisão geral-de-termologia
Revisão geral-de-termologia
Revisão geral-de-termologia
Revisão geral-de-termologia
Revisão geral-de-termologia
Revisão geral-de-termologia
Revisão geral-de-termologia
Revisão geral-de-termologia
Revisão geral-de-termologia
Revisão geral-de-termologia
Revisão geral-de-termologia
Revisão geral-de-termologia
Revisão geral-de-termologia
Revisão geral-de-termologia
Revisão geral-de-termologia
Revisão geral-de-termologia

Contenu connexe

Tendances

Fisica exercicios resolvidos 003
Fisica exercicios resolvidos  003Fisica exercicios resolvidos  003
Fisica exercicios resolvidos 003
comentada
 
Exercícios extras_Pricípios da eletrostática
Exercícios extras_Pricípios da eletrostáticaExercícios extras_Pricípios da eletrostática
Exercícios extras_Pricípios da eletrostática
O mundo da FÍSICA
 
Movimento uniforme
Movimento uniformeMovimento uniforme
Movimento uniforme
Betine Rost
 
Exercícios extras 9ano densidade
Exercícios extras 9ano densidadeExercícios extras 9ano densidade
Exercícios extras 9ano densidade
Professora Raquel
 

Tendances (20)

Termometria
TermometriaTermometria
Termometria
 
Fisica exercicios resolvidos 003
Fisica exercicios resolvidos  003Fisica exercicios resolvidos  003
Fisica exercicios resolvidos 003
 
Exercícios extras_Pricípios da eletrostática
Exercícios extras_Pricípios da eletrostáticaExercícios extras_Pricípios da eletrostática
Exercícios extras_Pricípios da eletrostática
 
3° ano(potencial elétrico) física
3° ano(potencial elétrico) física3° ano(potencial elétrico) física
3° ano(potencial elétrico) física
 
Exercícios dilatação térmica
Exercícios dilatação térmicaExercícios dilatação térmica
Exercícios dilatação térmica
 
CONCEITOS BÁSICOS PAR APLICAÇÃO DA LEI DE FOURIER:
CONCEITOS BÁSICOS PAR APLICAÇÃO DA LEI DE FOURIER:CONCEITOS BÁSICOS PAR APLICAÇÃO DA LEI DE FOURIER:
CONCEITOS BÁSICOS PAR APLICAÇÃO DA LEI DE FOURIER:
 
Exercicios ondas
Exercicios ondasExercicios ondas
Exercicios ondas
 
Exercícios termometria 8º ano
Exercícios termometria 8º anoExercícios termometria 8º ano
Exercícios termometria 8º ano
 
Potencial elétrico
Potencial elétricoPotencial elétrico
Potencial elétrico
 
Gabarito física
Gabarito físicaGabarito física
Gabarito física
 
Ciclo trigonométrico
Ciclo trigonométricoCiclo trigonométrico
Ciclo trigonométrico
 
Física - Exercícios Resolvidos de Cinemática
Física - Exercícios Resolvidos de CinemáticaFísica - Exercícios Resolvidos de Cinemática
Física - Exercícios Resolvidos de Cinemática
 
3 fisica
3 fisica3 fisica
3 fisica
 
Termodinâmica resolvido
Termodinâmica resolvidoTermodinâmica resolvido
Termodinâmica resolvido
 
Física (calorimetria)
Física (calorimetria)Física (calorimetria)
Física (calorimetria)
 
Calor sensivel e calor latente
Calor sensivel e calor latenteCalor sensivel e calor latente
Calor sensivel e calor latente
 
Movimento uniforme
Movimento uniformeMovimento uniforme
Movimento uniforme
 
Lista de exercícios 9° ano
Lista de exercícios 9° anoLista de exercícios 9° ano
Lista de exercícios 9° ano
 
Exercícios extras 9ano densidade
Exercícios extras 9ano densidadeExercícios extras 9ano densidade
Exercícios extras 9ano densidade
 
Termologia
TermologiaTermologia
Termologia
 

En vedette

Fisica tópico 1 – termologia
Fisica tópico 1 – termologiaFisica tópico 1 – termologia
Fisica tópico 1 – termologia
comentada
 
Física 2º ano prof. pedro ivo - (relação entre as escalas termométricas )
Física 2º ano   prof. pedro ivo - (relação entre as escalas termométricas )Física 2º ano   prof. pedro ivo - (relação entre as escalas termométricas )
Física 2º ano prof. pedro ivo - (relação entre as escalas termométricas )
Pedro Ivo Andrade Sousa
 
Fisica exercicios resolvidos 012
Fisica exercicios resolvidos  012Fisica exercicios resolvidos  012
Fisica exercicios resolvidos 012
comentada
 
Questõesdetermologia1
Questõesdetermologia1Questõesdetermologia1
Questõesdetermologia1
afpinto
 
Exercícios da uem
Exercícios da uemExercícios da uem
Exercícios da uem
Patyoli
 
QUESTÕES - FILOSOFIA - ENEM 2013
QUESTÕES - FILOSOFIA - ENEM 2013QUESTÕES - FILOSOFIA - ENEM 2013
QUESTÕES - FILOSOFIA - ENEM 2013
Marta Caregnato
 
Gabarito calor e temperatura
Gabarito   calor e temperaturaGabarito   calor e temperatura
Gabarito calor e temperatura
professoraludmila
 
Atividades de física 2 ano prof waldir montenegro 2014
Atividades de física 2  ano prof waldir  montenegro 2014Atividades de física 2  ano prof waldir  montenegro 2014
Atividades de física 2 ano prof waldir montenegro 2014
Waldir Montenegro
 

En vedette (20)

Questões Corrigidas, em Word: Temperatura e Dilatação - Conteúdo vinculado ...
Questões Corrigidas, em Word: Temperatura e Dilatação   - Conteúdo vinculado ...Questões Corrigidas, em Word: Temperatura e Dilatação   - Conteúdo vinculado ...
Questões Corrigidas, em Word: Temperatura e Dilatação - Conteúdo vinculado ...
 
Fisica tópico 1 – termologia
Fisica tópico 1 – termologiaFisica tópico 1 – termologia
Fisica tópico 1 – termologia
 
Fisica20031
Fisica20031Fisica20031
Fisica20031
 
Lista de exercício 2ª etapa
Lista de exercício 2ª etapaLista de exercício 2ª etapa
Lista de exercício 2ª etapa
 
Física 2º ano prof. pedro ivo - (relação entre as escalas termométricas )
Física 2º ano   prof. pedro ivo - (relação entre as escalas termométricas )Física 2º ano   prof. pedro ivo - (relação entre as escalas termométricas )
Física 2º ano prof. pedro ivo - (relação entre as escalas termométricas )
 
Apostila de-fisica-2º-ano
Apostila de-fisica-2º-anoApostila de-fisica-2º-ano
Apostila de-fisica-2º-ano
 
Fisica exercicios resolvidos 012
Fisica exercicios resolvidos  012Fisica exercicios resolvidos  012
Fisica exercicios resolvidos 012
 
Reso 17 10
Reso 17 10Reso 17 10
Reso 17 10
 
Questõesdetermologia1
Questõesdetermologia1Questõesdetermologia1
Questõesdetermologia1
 
Exercícios da uem
Exercícios da uemExercícios da uem
Exercícios da uem
 
Calorimetria
CalorimetriaCalorimetria
Calorimetria
 
Calorimetria
CalorimetriaCalorimetria
Calorimetria
 
Revparalela2ano
Revparalela2anoRevparalela2ano
Revparalela2ano
 
Lista 2° ano
Lista 2° anoLista 2° ano
Lista 2° ano
 
QUESTÕES - FILOSOFIA - ENEM 2013
QUESTÕES - FILOSOFIA - ENEM 2013QUESTÕES - FILOSOFIA - ENEM 2013
QUESTÕES - FILOSOFIA - ENEM 2013
 
227516957 cad-c3-curso-a-prof-exercicios-fisica
227516957 cad-c3-curso-a-prof-exercicios-fisica227516957 cad-c3-curso-a-prof-exercicios-fisica
227516957 cad-c3-curso-a-prof-exercicios-fisica
 
Gabarito calor e temperatura
Gabarito   calor e temperaturaGabarito   calor e temperatura
Gabarito calor e temperatura
 
Atividades de física 2 ano prof waldir montenegro 2014
Atividades de física 2  ano prof waldir  montenegro 2014Atividades de física 2  ano prof waldir  montenegro 2014
Atividades de física 2 ano prof waldir montenegro 2014
 
Comentario exatas
Comentario exatasComentario exatas
Comentario exatas
 
Apostila 2ano presao e atividade sensivel
Apostila 2ano  presao e atividade sensivelApostila 2ano  presao e atividade sensivel
Apostila 2ano presao e atividade sensivel
 

Similaire à Revisão geral-de-termologia

1° Etapa_2° Avaliação_Simulado_2° ano
1° Etapa_2° Avaliação_Simulado_2° ano1° Etapa_2° Avaliação_Simulado_2° ano
1° Etapa_2° Avaliação_Simulado_2° ano
O mundo da FÍSICA
 
Lista de exercícios de física pra segunda feira
Lista de exercícios de física pra segunda feiraLista de exercícios de física pra segunda feira
Lista de exercícios de física pra segunda feira
Paulo Lopes
 
1° Etapa_3° Avaliação_Recuperação_2° Ano
1° Etapa_3° Avaliação_Recuperação_2° Ano1° Etapa_3° Avaliação_Recuperação_2° Ano
1° Etapa_3° Avaliação_Recuperação_2° Ano
O mundo da FÍSICA
 
Exerccioslivro2 temperaturaeescalastermomtricas-090523121818-phpapp01
Exerccioslivro2 temperaturaeescalastermomtricas-090523121818-phpapp01Exerccioslivro2 temperaturaeescalastermomtricas-090523121818-phpapp01
Exerccioslivro2 temperaturaeescalastermomtricas-090523121818-phpapp01
Cleiton Turibio
 
Basica termo e dilatação
Basica termo e dilataçãoBasica termo e dilatação
Basica termo e dilatação
rodrigoateneu
 
Atividades de ciências para o 9° ano prof waldir 2014
Atividades de ciências para o 9° ano prof waldir 2014Atividades de ciências para o 9° ano prof waldir 2014
Atividades de ciências para o 9° ano prof waldir 2014
Waldir Montenegro
 

Similaire à Revisão geral-de-termologia (20)

Lista de exercícios complementar 8°
Lista de exercícios complementar 8°Lista de exercícios complementar 8°
Lista de exercícios complementar 8°
 
Física 2 Ramalho (testes propostos)
Física 2 Ramalho (testes propostos)Física 2 Ramalho (testes propostos)
Física 2 Ramalho (testes propostos)
 
Escalas termometricas.
Escalas termometricas.Escalas termometricas.
Escalas termometricas.
 
Termometria - Escalas termométricas
Termometria - Escalas termométricasTermometria - Escalas termométricas
Termometria - Escalas termométricas
 
Td 1 física ii
Td 1   física iiTd 1   física ii
Td 1 física ii
 
Exescalas
ExescalasExescalas
Exescalas
 
1° Etapa_2° Avaliação_Simulado_2° ano
1° Etapa_2° Avaliação_Simulado_2° ano1° Etapa_2° Avaliação_Simulado_2° ano
1° Etapa_2° Avaliação_Simulado_2° ano
 
Lista de exercícios de física pra segunda feira
Lista de exercícios de física pra segunda feiraLista de exercícios de física pra segunda feira
Lista de exercícios de física pra segunda feira
 
Termologiatransmissaodecalordilatacao
TermologiatransmissaodecalordilatacaoTermologiatransmissaodecalordilatacao
Termologiatransmissaodecalordilatacao
 
Termometria
TermometriaTermometria
Termometria
 
Lista Termologia (T1)
Lista Termologia (T1)Lista Termologia (T1)
Lista Termologia (T1)
 
1° Etapa_3° Avaliação_Recuperação_2° Ano
1° Etapa_3° Avaliação_Recuperação_2° Ano1° Etapa_3° Avaliação_Recuperação_2° Ano
1° Etapa_3° Avaliação_Recuperação_2° Ano
 
Exerccioslivro2 temperaturaeescalastermomtricas-090523121818-phpapp01
Exerccioslivro2 temperaturaeescalastermomtricas-090523121818-phpapp01Exerccioslivro2 temperaturaeescalastermomtricas-090523121818-phpapp01
Exerccioslivro2 temperaturaeescalastermomtricas-090523121818-phpapp01
 
E. MÉDIO 2 MANHÃ FÍSICA 07 03 2023 DEFINIÇÃO DE TEMPERATURA CONCEITOS FUNDAME...
E. MÉDIO 2 MANHÃ FÍSICA 07 03 2023 DEFINIÇÃO DE TEMPERATURA CONCEITOS FUNDAME...E. MÉDIO 2 MANHÃ FÍSICA 07 03 2023 DEFINIÇÃO DE TEMPERATURA CONCEITOS FUNDAME...
E. MÉDIO 2 MANHÃ FÍSICA 07 03 2023 DEFINIÇÃO DE TEMPERATURA CONCEITOS FUNDAME...
 
Introdução à TERMOLOGIA - Escalas Termométricas
Introdução à TERMOLOGIA - Escalas TermométricasIntrodução à TERMOLOGIA - Escalas Termométricas
Introdução à TERMOLOGIA - Escalas Termométricas
 
Basica termo e dilatação
Basica termo e dilataçãoBasica termo e dilatação
Basica termo e dilatação
 
Atividades de ciências para o 9° ano prof waldir 2014
Atividades de ciências para o 9° ano prof waldir 2014Atividades de ciências para o 9° ano prof waldir 2014
Atividades de ciências para o 9° ano prof waldir 2014
 
Exercícios Livro 2 Temperatura E Escalas Termométricas
Exercícios Livro 2   Temperatura E Escalas TermométricasExercícios Livro 2   Temperatura E Escalas Termométricas
Exercícios Livro 2 Temperatura E Escalas Termométricas
 
Exercícios Livro 2 Temperatura E Escalas Termométricas
Exercícios Livro 2   Temperatura E Escalas TermométricasExercícios Livro 2   Temperatura E Escalas Termométricas
Exercícios Livro 2 Temperatura E Escalas Termométricas
 
Apostila física 1serie
Apostila física 1serieApostila física 1serie
Apostila física 1serie
 

Plus de Marcia Marcia.Cristina2

Plus de Marcia Marcia.Cristina2 (20)

Exercícios avaliativos
Exercícios avaliativosExercícios avaliativos
Exercícios avaliativos
 
Fde f.halliday.8.ed.vol.2
Fde f.halliday.8.ed.vol.2Fde f.halliday.8.ed.vol.2
Fde f.halliday.8.ed.vol.2
 
Física v1 (beatriz alvarenga e antônio máximo)
Física v1 (beatriz alvarenga e antônio máximo)Física v1 (beatriz alvarenga e antônio máximo)
Física v1 (beatriz alvarenga e antônio máximo)
 
Aula 10 analise combinatoria parte i
Aula 10   analise combinatoria parte iAula 10   analise combinatoria parte i
Aula 10 analise combinatoria parte i
 
Cbc fisica
Cbc fisicaCbc fisica
Cbc fisica
 
F.d.f. vol. 3.halli.- 8ª ed -
F.d.f.   vol. 3.halli.- 8ª ed -F.d.f.   vol. 3.halli.- 8ª ed -
F.d.f. vol. 3.halli.- 8ª ed -
 
1323093414122
13230934141221323093414122
1323093414122
 
Exercicioreforco7
Exercicioreforco7Exercicioreforco7
Exercicioreforco7
 
Questões do enem
Questões do enemQuestões do enem
Questões do enem
 
Usinaseletricas
UsinaseletricasUsinaseletricas
Usinaseletricas
 
Apresentacao corrente eletrica
Apresentacao corrente eletricaApresentacao corrente eletrica
Apresentacao corrente eletrica
 
Prova 2 tri_1
Prova 2 tri_1Prova 2 tri_1
Prova 2 tri_1
 
Dilatacao linear exercicios
Dilatacao linear exerciciosDilatacao linear exercicios
Dilatacao linear exercicios
 
Dilatacao
DilatacaoDilatacao
Dilatacao
 
Efeito estufa
Efeito estufaEfeito estufa
Efeito estufa
 
Simulado de física ii
Simulado de física iiSimulado de física ii
Simulado de física ii
 
Dilatacao
DilatacaoDilatacao
Dilatacao
 
Simulado de fisica
Simulado de fisicaSimulado de fisica
Simulado de fisica
 
Simulado de fisica
Simulado de fisicaSimulado de fisica
Simulado de fisica
 
Transformações de energia enem
Transformações de energia enemTransformações de energia enem
Transformações de energia enem
 

Dernier

Aula 03 - Filogenia14+4134684516498481.pptx
Aula 03 - Filogenia14+4134684516498481.pptxAula 03 - Filogenia14+4134684516498481.pptx
Aula 03 - Filogenia14+4134684516498481.pptx
andrenespoli3
 
PROJETO DE EXTENSÃO I - TECNOLOGIA DA INFORMAÇÃO Relatório Final de Atividade...
PROJETO DE EXTENSÃO I - TECNOLOGIA DA INFORMAÇÃO Relatório Final de Atividade...PROJETO DE EXTENSÃO I - TECNOLOGIA DA INFORMAÇÃO Relatório Final de Atividade...
PROJETO DE EXTENSÃO I - TECNOLOGIA DA INFORMAÇÃO Relatório Final de Atividade...
HELENO FAVACHO
 

Dernier (20)

Seminário Biologia e desenvolvimento da matrinxa.pptx
Seminário Biologia e desenvolvimento da matrinxa.pptxSeminário Biologia e desenvolvimento da matrinxa.pptx
Seminário Biologia e desenvolvimento da matrinxa.pptx
 
Camadas da terra -Litosfera conteúdo 6º ano
Camadas da terra -Litosfera  conteúdo 6º anoCamadas da terra -Litosfera  conteúdo 6º ano
Camadas da terra -Litosfera conteúdo 6º ano
 
Plano de aula Nova Escola períodos simples e composto parte 1.pptx
Plano de aula Nova Escola períodos simples e composto parte 1.pptxPlano de aula Nova Escola períodos simples e composto parte 1.pptx
Plano de aula Nova Escola períodos simples e composto parte 1.pptx
 
EDUCAÇÃO ESPECIAL NA PERSPECTIVA INCLUSIVA
EDUCAÇÃO ESPECIAL NA PERSPECTIVA INCLUSIVAEDUCAÇÃO ESPECIAL NA PERSPECTIVA INCLUSIVA
EDUCAÇÃO ESPECIAL NA PERSPECTIVA INCLUSIVA
 
PROJETO DE EXTENÇÃO - GESTÃO DE RECURSOS HUMANOS.pdf
PROJETO DE EXTENÇÃO - GESTÃO DE RECURSOS HUMANOS.pdfPROJETO DE EXTENÇÃO - GESTÃO DE RECURSOS HUMANOS.pdf
PROJETO DE EXTENÇÃO - GESTÃO DE RECURSOS HUMANOS.pdf
 
Slides Lição 6, CPAD, As Nossas Armas Espirituais, 2Tr24.pptx
Slides Lição 6, CPAD, As Nossas Armas Espirituais, 2Tr24.pptxSlides Lição 6, CPAD, As Nossas Armas Espirituais, 2Tr24.pptx
Slides Lição 6, CPAD, As Nossas Armas Espirituais, 2Tr24.pptx
 
PRÁTICAS PEDAGÓGICAS GESTÃO DA APRENDIZAGEM
PRÁTICAS PEDAGÓGICAS GESTÃO DA APRENDIZAGEMPRÁTICAS PEDAGÓGICAS GESTÃO DA APRENDIZAGEM
PRÁTICAS PEDAGÓGICAS GESTÃO DA APRENDIZAGEM
 
O PLANETA TERRA E SEU SATÉLITE NATURAL - LUA
O PLANETA TERRA E SEU SATÉLITE NATURAL - LUAO PLANETA TERRA E SEU SATÉLITE NATURAL - LUA
O PLANETA TERRA E SEU SATÉLITE NATURAL - LUA
 
Aula 03 - Filogenia14+4134684516498481.pptx
Aula 03 - Filogenia14+4134684516498481.pptxAula 03 - Filogenia14+4134684516498481.pptx
Aula 03 - Filogenia14+4134684516498481.pptx
 
PROJETO DE EXTENSÃO I - SERVIÇOS JURÍDICOS, CARTORÁRIOS E NOTARIAIS.pdf
PROJETO DE EXTENSÃO I - SERVIÇOS JURÍDICOS, CARTORÁRIOS E NOTARIAIS.pdfPROJETO DE EXTENSÃO I - SERVIÇOS JURÍDICOS, CARTORÁRIOS E NOTARIAIS.pdf
PROJETO DE EXTENSÃO I - SERVIÇOS JURÍDICOS, CARTORÁRIOS E NOTARIAIS.pdf
 
About Vila Galé- Cadeia Empresarial de Hotéis
About Vila Galé- Cadeia Empresarial de HotéisAbout Vila Galé- Cadeia Empresarial de Hotéis
About Vila Galé- Cadeia Empresarial de Hotéis
 
Texto dramático com Estrutura e exemplos.ppt
Texto dramático com Estrutura e exemplos.pptTexto dramático com Estrutura e exemplos.ppt
Texto dramático com Estrutura e exemplos.ppt
 
PROJETO DE EXTENSÃO I - TECNOLOGIA DA INFORMAÇÃO Relatório Final de Atividade...
PROJETO DE EXTENSÃO I - TECNOLOGIA DA INFORMAÇÃO Relatório Final de Atividade...PROJETO DE EXTENSÃO I - TECNOLOGIA DA INFORMAÇÃO Relatório Final de Atividade...
PROJETO DE EXTENSÃO I - TECNOLOGIA DA INFORMAÇÃO Relatório Final de Atividade...
 
Aula sobre o Imperialismo Europeu no século XIX
Aula sobre o Imperialismo Europeu no século XIXAula sobre o Imperialismo Europeu no século XIX
Aula sobre o Imperialismo Europeu no século XIX
 
P P P 2024 - *CIEJA Santana / Tucuruvi*
P P P 2024  - *CIEJA Santana / Tucuruvi*P P P 2024  - *CIEJA Santana / Tucuruvi*
P P P 2024 - *CIEJA Santana / Tucuruvi*
 
Aula de jornada de trabalho - reforma.ppt
Aula de jornada de trabalho - reforma.pptAula de jornada de trabalho - reforma.ppt
Aula de jornada de trabalho - reforma.ppt
 
PROJETO DE EXTENSÃO - EDUCAÇÃO FÍSICA BACHARELADO.pdf
PROJETO DE EXTENSÃO - EDUCAÇÃO FÍSICA BACHARELADO.pdfPROJETO DE EXTENSÃO - EDUCAÇÃO FÍSICA BACHARELADO.pdf
PROJETO DE EXTENSÃO - EDUCAÇÃO FÍSICA BACHARELADO.pdf
 
E a chuva ... (Livro pedagógico para ser usado na educação infantil e trabal...
E a chuva ...  (Livro pedagógico para ser usado na educação infantil e trabal...E a chuva ...  (Livro pedagógico para ser usado na educação infantil e trabal...
E a chuva ... (Livro pedagógico para ser usado na educação infantil e trabal...
 
Araribá slides 9ano.pdf para os alunos do medio
Araribá slides 9ano.pdf para os alunos do medioAraribá slides 9ano.pdf para os alunos do medio
Araribá slides 9ano.pdf para os alunos do medio
 
migração e trabalho 2º ano.pptx fenomenos
migração e trabalho 2º ano.pptx fenomenosmigração e trabalho 2º ano.pptx fenomenos
migração e trabalho 2º ano.pptx fenomenos
 

Revisão geral-de-termologia

  • 1. 1 Revisão Geral de Termologia Prof. Marcos Guimarães Sampaio (só prá moçadinha) Termometria 01) (ITA – SP) O verão de 1994 foi particularmente quente nos Estados Unidos da América. A diferença entre a máxima temperatura do verão e a mínima no inverno anterior foi de 60 °C. Qual o valor dessa diferença na escala Fahrenheit? a) 108 °F b) 60 °F c) 140 °F d) 33 °F e) 92 °F Resolução: ∆C/5 = ∆F/9  60/5 = ∆F/9  ∆F = 108 °F (Resposta A) 02) (MACK – SP) Uma pessoa mediu a temperatura de seu corpo, utilizando-se de um termômetro graduado na escala Fahrenheit, e encontrou o valor 97,7 o F. Essa temperatura, na escala Celsius, corresponde a: a) 36,5 o C b) 37,0 o C c) 37,5 o C d) 38,0 o C e) 38,5 o C Resolução: (F – 32)/9 = C/5  (97,7 – 32)/9 = C/5  C = 36,5 ºC (Resposta A) 03) (UFPI - PI) O Aquecimento Global é um fenômeno climático de larga extensão. As previsões mais catastróficas para a região Amazônica incluem o desaparecimento completo da floresta se a temperatura média da região tiver um aumento superior aos 5ºC. Com isso a temperatura média anual da cidade de Manaus passaria a ser de 33ºC, que lida na escala Kelvin corresponderia a: a) 300 K b) 310 K c) 290 K d) 306 K e) 302 K Resolução: T(K) = C + 273  T(K) = 33 + 273 = 306 K. (Resposta D) 04) (IFAL - AL) Uma pessoa que estava viajando para Londres, no dia seguinte, procura se informar a respeito das condições do tempo no local de destino. Num canal de TV a cabo, toma conhecimento de que a temperatura naquela cidade e de 26,6°F, pois, na Inglaterra, costuma-se usar termômetros graduados na escala Fahrenheit. Baseando-se nessa informação, o viajante chegara a seguinte conclusão: a) Terá que levar pesados agasalhos para suportar uma temperatura abaixo do zero absoluto. b) Com uma temperatura alem dos 36°C, não devera se preocupar com agasalhos. c) Levara roupas leves, pois a temperatura esta entre 27°C e 36° C. d) Como a temperatura esta abaixo de 0°C, providenciara roupas adequadas a um inverno rigoroso. e) Usará suas roupas normais, pois as temperaturas nas duas escalas são iguais para todos os valores. Resolução: (F – 32)/9 = C/5  (26,6 – 32)/9 = C/5  C = – 3 ºC. (Resposta D) 05) (UFPE - PE) As escalas de temperatura mais conhecidas são Celsius (ºC) e Fahrenheit (ºF). Nessas escalas, o ponto de congelamento da água corresponde a 0ºC e 32ºF, e o ponto de ebulição corresponde a 100ºC e 212ºF. A equivalência entre as escalas é obtida por uma função polinomial do 1º grau, ou seja, uma função da forma f(x) = ax + b, em que f(x) é a temperatura em grau Fahrenheit (ºF) e x a temperatura em grau Celsius (ºC). Se em um determinado dia a temperatura no centro do Recife era de 29ºC, a temperatura equivalente em grau Fahrenheit (ºF) era de: a) 84ºF b) 84,02ºF c) 84,1ºF d) 84,12ºF e) 84,2ºF Resolução: (F – 32)/9 = C/5  (F – 32)/9 = 29/5  F = 84,2ºF (Resposta E) 06) (PUC – RS) Podemos caracterizar uma escala absoluta de temperatura quando: a) dividimos a escala em 100 partes iguais. b) associamos o zero da escala ao estado de energia cinética mínima das partículas de um sistema. c) associamos o zero da escala ao estado de energia cinética máxima das partículas de um sistema. d) associamos o zero da escala ao ponto de fusão do gelo. e) associamos o valor 100 da escala ao ponto de ebulição da água.
  • 2. 2 Resolução: Zero absoluto corresponde ao estado de agitação mínimo das partículas o que significa energia cinética mínima. (Resposta B) 07) Imagine dois termômetros graduados. Um na escala Celsius e o outro na escala Fahrenheit. Sendo assim, responda às questões seguintes: a) Quando o termômetro Celsius acusar uma variação de temperatura igual a 20 ºC determine a variação de temperatura acusada por um termômetro Fahrenheit e por um termômetro Kelvin. b) Quando um termômetro Celsius acusar uma temperatura de 20 ºC, qual será a temperatura acusada, no mesmo local por um termômetro Fahrenheit e por um Kelvin. Resolução: a) ∆C/5 = ∆F/9  20/5 = ∆F/9  ∆F = 36 ºF e ∆T(K) = ∆C  ∆T(K) = 20 K. b) (F – 32)/9 = C/5  (F – 32)/9 = 20/5  F = 68ºF e T(K) = C + 273 = 20 + 273 = 293 K. (Respostas: a) 36 ºF e 20 K; b) 68ºF e 293 K) 08) (UFRPE - PE) Duas escalas termométricas, ºX e ºY, têm suas respectivas temperaturas, TX e TY, relacionadas pela expressão 2TX – 3TY + 2 = 0. Pode-se afirmar que uma variação de temperatura de 30 ºX corresponde, na escala Y, a uma variação de: a) 10 ºY b) 20 ºY c) 30 ºY d) 40 ºY e) 50 ºY Resolução: Para cada 1ºX de variação teremos, em ºY: 2.1 – 3TY + 2 = 0  TY = 4/3 ºY. assim, para uma variação de 30 ºX teremos 30.4/3 = 40ºY. (Resposta: D) 09) Três afirmações são feitas a respeito de temperatura. I) Zero absoluto equivale à temperatura em que ocorre o congelamento da água. II) Podemos obter água a - 350 ºC. III) Quando um corpo tem sua temperatura variando 50 ºC pode-se dizer que, se a escala fosse a Kelvin, a variação de temperatura, numericamente, seria a mesma. a) Apenas I é falsa b) Apenas II é falsa c) Apenas III é falsa d) Apenas III é verdadeira e) As três afirmações são falsas. Resolução: I- (F) O congelamento da água sob pressão normal ocorre a 273 K. II- (F) A menor temperatura teoricamente possível é o 0 K ou – 273 ºC. III- (V) Já que o intervalo entre os pontos fixos de fusão do gelo e ebulição da água são os mesmos. (Resposta D) 10) (ACAFE – SC) Três blocos de metais diferentes foram aquecidos de forma contínua e suas temperaturas medidas, instante após instante, por três termômetros, simultaneamente. Um, na escala Celsius, outro na escala Fahrenheit e o terceiro na escala Kelvin. A figura abaixo mostra três gráficos da temperatura versus tempo, T X t, confeccionados a partir dos dados obtidos. As curvas de aquecimento, em cada gráfico, foram obtidas por termômetros diferentes. Analisando os gráficos, é correto o que se afirma em: a) No gráfico C, a curva de aquecimento 1 corresponde às medidas realizadas pelo termômetro na escala Fahrenheit e a curva de aquecimento 2 corresponde às medidas realizadas pelo termômetro na escala Celsius. b) No gráfico A, a curva de aquecimento 2 corresponde às medidas realizadas pelo termômetro na escala Celsius e a curva de aquecimento 1 corresponde às medidas realizadas pelo termômetro na escala Kelvin. c) No gráfico B, a curva de aquecimento 2 corresponde às medidas realizadas pelo termômetro na escala Fahrenheit e a curva de aquecimento 1 corresponde às medidas realizadas pelo termômetro na escala Celsius. d) No gráfico B, a curva de aquecimento 1 corresponde às medidas realizadas pelo termômetro na escala Fahrenheit e a curva de aquecimento 2 corresponde às medidas realizadas pelo termômetro na escala Kelvin. e) No gráfico C, a curva de aquecimento 1 corresponde às medidas realizadas pelo termômetro na escala Kelvin e a curva de aquecimento 2 corresponde às medidas realizadas pelo termômetro na escala Fahrenheit. Resolução: A partir do zero absoluto, a relação gráfica entre as escalas é a seguinte: -273 0 -459,4 F C K
  • 3. 3 a) F: o encontro entre as escalas ocorre a -40. b) V: a distância numérica entre as escalas será sempre igual a 100 unidades. c) F: como está explicado no item anterior d) F: a curva para a escala Kelvin inicia no 0 (zero absoluto). e) F: como está explicado no item anterior. (Resposta B) 11) (CEFET – PR) A superfície gelada do pequeno Plutão é composta por nitrogênio, metano e traços de monóxido de carbono. A temperatura do planeta anão varia ao longo de sua órbita porque, no decorrer de sua trajetória, aproxima-se do Sol até 30 UA e afasta-se até 50 UA. Existe uma tênue atmosfera que congela e cai sobre o planeta anão quando este se afasta do Sol. Sendo assim, dependendo da sua posição em relação ao Sol, a temperatura sobre a superfície do planeta anão varia de –230ºC a – 210ºC. Pode-se afirmar que: (UA = Unidade Astronômica) a) essas temperaturas não são lidas num termômetro graduado na escala Kelvin, pois a menor temperatura nesse termômetro é 0 K. b) não se medem essas temperaturas num termômetro graduado na escala Celsius, pois sua escala varia de 0ºC a 100ºC. c) se medem essas temperaturas com termômetros graduados na escala Celsius, pois é o único que mede temperaturas abaixo de zero. d) na escala Fahrenheit, o módulo da variação da temperatura sobre a superfície do pequeno Plutão corresponde a 36ºF. e) na escala Fahrenheit, o módulo da variação da temperatura sobre a superfície do pequeno Plutão corresponde a 20ºF. Resolução: ∆C/5 = ∆F/9  (-210 – (-230))/5 = ∆F/9  ∆F = 36 ºF. (Resposta D) 12) (ITA – SP) Para medir a febre de pacientes, um estudante de medicina criou sua própria escala linear de temperaturas. Nessa nova escala, os valores de 0 (zero) e 10 (dez) correspondem respectivamente a 37°C e 40°C. A temperatura de mesmo valor numérico em ambas escalas é aproximadamente a) 52,9 °C. b) 28,5 °C. c) 74,3 °C. d) –8,5°C. e) –28,5°C. Resolução: (t – t1)/(t2 – t1) = (C – C1)/(C2 – C1)  (t – 0)/(10 – 0) = (C – 37)(40 – 37)  (x – 0)/(10 – 0) = (x – 37)(40 – 37)  x/10 = (x – 37)/3  x  28,5 ºC. (Resposta B) 13) (MACK – SP) Joãozinho, seguindo as orientações de seu professor de Física, construiu uma nova escala termométrica. Ao nível do mar, ele atribuiu o valor -20 °J para a temperatura do gelo fundente e 130 °J para a temperatura de ebulição da água. A medida, que nessa escala tem valor coincidente com o da escala Celsius, refere-se à temperatura: a) 20 °J b) 30 °J c) 40 °J d) 50 °J e) 60 °J Resolução: (t – t1)/(t2 – t1) = (C – C1)/(C2 – C1)  (t – (-20))/(130 – (-20)) = (C – 0)/(100 – 0)  (x + 20)/150 = x/100  x = 40. (Resposta C) 14) (MACK – SP) Um profissional, necessitando efetuar uma medida de temperatura, utilizou um termômetro cujas escalas termométricas inicialmente impressas ao lado da coluna de mercúrio estavam ilegíveis. Para atingir seu objetivo, colocou o termômetro inicialmente numa vasilha com gelo fundente, sob pressão normal, e verificou que no equilíbrio térmico a coluna de mercúrio atingiu 8,0 cm. Ao colocar o termômetro em contato com água fervente, também sob pressão normal, o equilíbrio térmico se deu com a coluna de mercúrio atingindo 20,0 cm de altura. Se nesse termômetro utilizarmos as escalas Celsius e Fahrenheit e a temperatura a ser medida for expressa pelo mesmo valor nas duas escalas, a coluna de mercúrio terá altura de: a) 0,33 cm b) 0,80 cm c) 3,2 cm d) 4,0 cm e) 6,0 cm Resolução: A temperatura comum nas escalas Celsius e Fahrenheit é: (F – 32)/9 = C/5  (x – 32)/9 = x/5  x = -40 Resposta C 15) (UFMS – MS) Através de experimentos, biólogos observaram que a taxa de canto de grilos de uma determinada espécie estava relacionada com a temperatura ambiente de uma maneira que poderia ser considerada linear. Experiências mostraram que, a uma temperatura de 21º C, os grilos cantavam, em média, 120 vezes por minuto; e, a uma temperatura de 26º C, os grilos cantavam, em média, 180 vezes por minuto. Considerando T a temperatura em graus 0 100 8 20 -40 y (0 – (-40))/(100 – (-40)) = (8 – y)/(20 – y)  y = 3,2 cm
  • 4. 4 Celsius e n o número de vezes que os grilos cantavam por minuto, podemos representar a relação entre T e n pelo gráfico abaixo. Supondo que os grilos estivessem cantando, em média, 156 vezes por minuto, de acordo com o modelo sugerido nesta questão, estima-se que a temperatura deveria ser igual a a) 21,5 ºC . b) 22 ºC . c) 23 ºC . d) 24 ºC . e) 25,5 ºC . Resolução: Resposta D 16) (UNIFESP – SP) O texto a seguir foi extraído de uma matéria sobre congelamento de cadáveres para sua preservação por muitos anos, publicada no jornal “O Estado de S. Paulo” de 21.07.2002: Após a morte clínica, o corpo é resfriado com gelo. Uma injeção de anticoagulantes é aplicada e um fluido especial é bombeado para o coração, espalhando-se pelo corpo e empurrando para fora os fluidos naturais. O corpo é colocado numa câmara com gás nitrogênio, onde os fluidos endurecem em vez de congelar. Assim que atinge a temperatura de -321°, o corpo é levado para um tanque de nitrogênio líquido, onde fica de cabeça para baixo. Na matéria, não consta a unidade de temperatura usada. Considerando que o valor indicado de -321° esteja correto e que pertença a uma das escalas (Kelvin, Celsius ou Fahrenheit), pode-se concluir que foi usada a escala: a) Kelvin, pois se trata de um trabalho científico e esta é a unidade adotada pelo Sistema Internacional. b) Fahrenheit, por ser um valor inferior ao zero absoluto e, portanto, só pode ser medido nessa escala. c) Fahrenheit, pois as escalas Celsius e Kelvin não admitem esse valor numérico de temperatura. d) Celsius, pois só ela tem valores numéricos negativos para a indicação de temperaturas. e) Celsius, por tratar-se de uma matéria publicada em língua portuguesa e essa ser a unidade adotada oficialmente no Brasil. Resolução: A menor temperatura teoricamente possível é o 0 K ou – 273 ºC ou -459,4 ºF. Portanto a única escala que se enquadra com o valor de temperatura descrito no texto é a Fahrenheit. (Resposta C) 17) Uma escala termométrica X é construída de modo que a temperatura de 0 °X corresponde a – 4 °F, e a temperatura de 100 °X corresponde a 68°F. Nesta escala X, a temperatura de fusão do gelo vale: a) 10 °X b) 20 °X c) 30 °X d) 40 °X e) 50 °X Resolução: 18) Um termômetro de mercúrio é calibrado com o ponto de gelo a 2cm de altura da coluna de mercúrio e o ponto de vapor a 12cm. Obtenha uma função que relacione a temperatura T(°C) com a altura h (cm) e determine a temperatura T(ºC) quando h = 10 cm. Resolução: 19) Os pontos de fusão do gelo e de ebulição da água na escala Fahrenheit são, respectivamente, 32 °F e 212 °F. Um termômetro A, graduado na escala Fahrenheit, e outro B, graduado na escala Celsius, são colocados simultaneamente em um frasco contendo água quente. Verifica-se que o termômetro A apresenta uma leitura que supera em 80 unidades a leitura do termômetro B. Podemos afirmar que a temperatura da água no frasco é: a) 60 °C b) 80 °C c) 112 °C d) 50 °F e) 112 °F Resolução: F = C + 80 e : (F – 32)/9 = C/5  (C + 80 – 32)/9 = C/5  C = 60 ºC (Resposta A) x 26 156 180 21 120 (x – 21)/(26 – 21) = (156 – 120)/( 180 – 120)  (x – 21)/5 = 36/60  x = 24 ºC x 100 32 68 0 -4 (x – 0)/(100 – 0)) = (32 – (-4)/(68 – (-4)  x/100 = 36/72  x = 50. (Resposta E) C 100 x 12cm 0 2cm (C – 0)/(100 – 0)) = (x – 2)/12 – 2)  C/100 = (x – 2)/10  C = 10x – 20 (Resposta: T(ºC) = 10 T(ºX) – 20)
  • 5. 5 20) (MACK – SP) Um termômetro defeituoso está graduado na escala Fahrenheit, indicando 30 °F para o ponto de gelo e 214 °F para o ponto de vapor. Neste termômetro, a única temperatura medida corretamente, corresponde a: a) 0 °F b) 30 °F c) 40 °F d) 50 °F e) 122 °F 21). Buscando satisfazer uma diversificada clientela, algumas operadoras de turismo lançaram a “viagem surpresa”: o viajante compra um pacote para um local desconhecido, só sabendo seu destino ao desembarcar. A única informação é dada alguns dias antes da viagem e refere-se à temperatura média no local a ser visitado. Numa dessas viagens, um passageiro recebeu o seguinte comunicado: a temperatura média local à época da viagem é 82ºF. Esse passageiro deve: a) levar chapéu, filtro solar, bermudas e roupas leves. b) levar agasalhos leves, pois o clima é ameno. c) precaver-se com capotes e agasalhos de lã, pois nessa temperatura há possibilidade de encontrar neve. d) desistir de viajar, pois a temperatura é tão baixa que dificilmente conseguirá sair do hotel. Resolução: Comparando com a escala Celsius, temos: (F – 32)/9 = C/5  (82 – 32)/9 = C/5  C = 87,7 ºF. (Resposta: A) 22) Um estudante construiu um termômetro, adotando uma escala termométrica obtida do seguinte modo: - mergulhou uma haste metálica em gelo fundente. Algum tempo depois, constatou que a medida da haste era de 20 cm. A esta medida atribuiu o valor de 50 graus Ypsolon (50 ºY); - em seguida, mergulhou a mesma haste metálica em um recipiente contendo água em ebulição. Algum tempo depois, constatou que a haste, dilatada, passou a medir 20,3 cm. A esta medida atribuiu o valor de 80 graus Ypsolon (80 ºY). Sendo assim, responda às questões a seguir: a) Ao mergulhar a haste metálica, por algum tempo, na água contida em um recipiente, seu comprimento se torna igual a 20,2 cm. Determine, em graus Ypsolon, a temperatura da água no recipiente. b) A temperatura de 90 ºY está associada a que medida de comprimento da haste metálica ? Resolução: A equação de conversão será (y – 50)/(80 – 50) = (L – 20)/(20,3 – 20)  (y – 50)/30 = (L – 20)/0,3  y – 50 = 100L – 2000 a) y – 50 = 100.20,2 – 2000  y = 70 ºY. b) 90 – 50 = 100L – 2000  L = 20,4 cm. (Respostas: a) 70 ºY; b) 20,4 cm) 23) (ENEM) Nos processos industriais, como na indústria de cerâmica, é necessário o uso de fornos capazes de produzir elevadas temperaturas e, em muitas situações, o tempo de elevação dessa temperatura deve ser controlado, para garantir a qualidade do produto final e a economia do processo Em uma indústria de cerâmica o forno é programado para elevar a temperatura ao longo do tempo de acordo com a função em que T é o valor da temperatura atingida pelo forno, em graus Celsius, e t é o tempo, em minutos, decorrido desde o instante em que o forno é ligado. Uma peça deve ser colocada nesse forno quando a temperatura for 48 o C e retirada quando a temperatura for 200 °C. O tempo de permanência dessa peça no forno é, em minutos, igual a a) 100 b)108 c) 128 d) 130 e) 150 Resolução: x 214 x 212 30 32 (x – 30)/(214 – 30) = (x – 32)/(212 – 32)  (x – 30) 184 = (x – 32)/180  x = 122 ºF (Resposta: E)
  • 6. 6 Temperatura inicial do forno T(t0) = 7/5.t0 + 20  T(0) =7/5.0 + 20 = 20 ºC. A temperatura do forno será 48 ºC (momento em que a peça será introduzida em: T(t) = 7/5.t + 20  48 = 7/5.t + 20  t1 = 20 min Após 100 minutos a temperatura da água será: T(t2) = 7/5.t2 + 20  T(100) = 7/5.100 + 20 = 160 ºC. Após isso, atingirá a temperatura de 200 ºC quando: T(t3) = 2/125t3² - 16/5t3 + 320  200 = 2/125t3² - 16/5t3 + 320  t3 = 50 min. Assim a peça permanecerá no forno durante t = (t2 – t1) + t3 = (100 – 20) + 50 = 130 min. (Resposta D) Dilatação Térmica 24) (UFES – ES) Uma barra de metal tem comprimento igual a 10,000 m a uma temperatura de 10,0 °C e comprimento igual a 10,006 m a uma temperatura de 40 °C. O coeficiente de dilatação linear do metal é: a) 1,5 × 10-4 °C-1 b) 6,0 × 10-4 °C-1 c) 2,0 × 10-5 °C-1 d) 2,0 × 10-4 °C-1 e) 3,0 × 10-4 °C-1 Resolução: ∆L = L0.α.∆  0,006 = 10. α.30  α = 2.10–5 ºC–1 (Resposta C) 25) (UFU – MG) Uma ponte de aço tem 1 000m de comprimento. O coeficiente de dilatação linear do aço é de 11. 10-6 °C–1 . A expansão da ponte, quando a temperatura sobe de 0 para 30°C, é de: a) 33cm. b) 37cm. c) 41cm. d) 52cm. e) 99cm. Resolução: ∆L = L0.α.∆  ∆L = 1000. 11. 10-6 .30 = 0,33m ou 33 cm (Resposta A) 26) (MACK – SP) Duas barras metálicas, de diferentes materiais, apresentam o mesmo comprimento a 0 °C. Ao serem aquecidas, à temperatura de 100 °C, a diferença entre seus comprimentos passa a ser de 1 mm. Sendo 2,2. 10–5 °C–1 o coeficiente de dilatação linear do material de uma barra e 1,7.10–5 °C–1 o do material da outra, o comprimento dessas barras a 0 °C era: a) 0,2 m b) 0,8 m c) 1,0 m d) 1,5 m e) 2,0 m Resolução: ∆L = L0.α1.∆ – L0.α2.∆  1.10-3 = L0. 2,2. 10–5 .100 – L0. 1,7.10–5 .100  L0 = 2,0 m (Resposta E) 27) (UERJ – RJ) Uma torre de aço, usada para transmissão de televisão, tem altura de 50 m quando a temperatura ambiente é de 40 0 C. Considere que o aço dilata-se, linearmente, em média, na proporção de 1/100.000, para cada variação de 1 0 C. À noite, supondo que a temperatura caia para 20 0 C, em valor absoluto a variação de comprimento da torre, em centímetros, será de: a) 1,0 b) 1,5 c) 2,0 d) 2,5 Resolução: ∆L = L0.α.∆  ∆L=50.( 1/100.000).(-20) = 1/100 m ou 1 cm. (Resposta A) 28) Entre dois trilhos consecutivos de uma via férrea, deixa-se um espaço apenas suficiente para facilitar livremente a dilatação térmica dos trilhos de 0 ºC até a temperatura de 70 ºC. O coeficiente de dilatação térmica linear do material dos trilhos é 1,0. 10–5 °C–1 . Cada trilho mede 20m a 0 °C. Qual o espaço entre dois trilhos consecutivos na temperatura de 0 °C? Resolução: ∆L = L0.α.∆  ∆L = 20. 1,0. 10–5 .70 = 0,014m = 1,4 cm., ou seja, 0,7 cm para cada lado. Desse modo, entre doius trilhos sucessivos devermos ter d = 1,4 cm. (Resposta: 1,4 cm) 29) Uma telha de alumínio tem dimensões lineares de 20cm x 500cm e seu coeficiente de dilatação linear é igual a 2,2 x 10–5 ºC–1 . A telha, ao ser exposta ao sol durante o dia, experimenta uma variação de temperatura de 20 ºC. A dilatação superficial máxima da chapa, em cm², durante esse dia, será. a) 1,1 b) 2,2 c) 4,4 d) 6,6 e) 8,8 Resolução: ∆A = A0.β.∆  ∆A = (20.500).(2. 2,2 x 10–5 ).(20) = 8,8 cm². (Resposta E) 30) Para se ligar estrutura metálicas em prédios usa-se a técnica de rebitagem em que para se colocarem os rebites é preferível que: a) eles estejam à mesma temperatura da chapa; b) eles estejam à temperatura superior a da chapa, geralmente aquecidos ao rubro; c) eles estejam resfriados a temperaturas abaixo da chapa;
  • 7. 7 d) qualquer das possibilidades acima ocorre desde que fiquem bem colocados Resolução: A uma temperatura inferior a da chapa, o diâmetro do rebite será menor que o da chapa permitindo a introdução do rebite. Quando a temperatura se elevar, o rebite estará perfeitamente encaixado. (Resposta C) 31) (FCC – SP) Uma peça sólida tem uma cavidade cujo volume vale 8cm³ a 20ºC. A temperatura da peça varia para 920 ºC e o coeficiente de dilatação linear do sólido (12.10–6 ºC–1 ) pode ser considerado constante. Supondo que a pressão interna da cavidade seja sempre igual à externa, a variação percentual do volume da cavidade foi de: a) 1,2%. b) 2,0%. c) 3,2%. d) 5,8%. e) 12%. Resolução: ∆V = V0.γ. ∆ sendo γ = 3α. Assim ∆V = V0.(3. 12.10–6 ).900 = 0,0324. V0. Portanto 3,24/100 V0 = 3,24% V0.(Resposta C) 32) (UFPI – PI) Muitos sistemas de aquecimento elétrico em que é necessário manter certa temperatura, tais como estufas, fornos, ferro com graduações apropriadas para passar diversos tipos de tecidos, usam termostato para manter sua temperatura mais ou menos constante. Esses reguladores de temperatura têm o mesmo princípio de funcionamento. De uma forma simplificada, pode-se dizer que eles possuem uma espiral bimetálica como mostrada na figura abaixo. Considere essa espiral bimetálica formada por latão na parte interna e de aço na parte externa. Com relação ao adequado funcionamento destes sistemas reguladores de temperatura, coloque V, para verdadeiro, ou F, para falso. 1 ( ) À medida que a temperatura do sistema de aquecimento elétrico vai caindo, a espiral bimetálica se contrai, e sua extremidade se move para direita, isto é, para fora, abrindo o contato elétrico. 2 ( ) À medida que a temperatura da espiral bimetálica aumenta, o latão se dilata mais que o aço e, quando a temperatura atinge o valor máximo necessário, a espiral se move para direita ou para fora, desligando o circuito. 3 ( ) Quando a temperatura da espiral aumenta, além da necessária para o respectivo sistema de aquecimento, o latão se dilata menos que o aço, e a espiral se move para direita ou para fora, desligando o circuito. 4 ( ) Quando a temperatura do sistema diminui, o aço se contrai mais que o latão, e a espiral bimetálica fecha novamente o circuito, até o sistema atingir a temperatura máxima da graduação do sistema de aquecimento. Resolução: Observe que: internamente: latão (2,0.10–5 ) e externamente: aço (1,3.10–5 ). 1- (F) pois o latão sofrerá maior contração térmica que o aço, já que seu coeficiente de dilatação é maior. 2- (V) pois o latão sofrerá maior dilatação. 3- (F) No aquecimento o latão se dilata mais que o aço. Seu coeficiente é maior. 4- (F) no resfriamento o latão contrai mais que o aço. (Resposta: F; V; F; F) 33) (IFCE – CE) Uma barra de aço de comprimento L0 = 0,700 m é encaixada praticamente sem folga em uma cavidade lubrificada, como mostra a figura. A temperatura inicial da barra é de 20,0 ºC e seu coeficiente = 12,0 x 10- 6 ºC-1 . Quando uma das extremidades da barra é mantida fixa e a outra é puxada por uma força T, seu comprimento se altera de acordo com a equação L = L0 (1 + T/f), com f = 6,00 x 107 N (t < 0 se T comprime a barra). Supondo-se que a cavidade não se deforma, a força horizontal que a barra exerce sobre a parede da cavidade à direita, para uma temperatura de 22,0ºC, é igual a a) 1220 N b) 1420 N c) 1620 N d) 1520 N e) 1440 N Resolução: L0 = 70cm e ∆L será ∆L = L0.α.∆ = 70. 12,0 x 10-6 .(22 – 20) = 168.10-5 = 0,00168 cm  L = 70,00168cm. A força será L = L0 (1
  • 8. 8 + T/f)  70,00168 = 70(1 + T/6.107 )  t = 1440 N. (Resposta E) 34) (IFAL – AL) Um fato bem conhecido levado em conta por técnicos, engenheiros, cientistas e outros profissionais ao se defrontarem com a tarefa de construir algum equipamento, projetar uma construção, ou mesmo efetuar um experimento científico, é que as dimensões de um corpo aumentam quando aumentamos sua temperatura. Ressalvando algumas exceções, este é um comportamento geral dos sólidos, líquidos e gases. Sobre este fenômeno, denominado dilatação térmica, analise as afirmativas a seguir: I. Toda dilatação térmica, em verdade, se processa nas três dimensões espaciais: comprimento, largura e altura. II. A variação das dimensões de um corpo depende, exclusivamente, do material de que ele é feito e da sua variação de temperatura sofrida. III. No caso dos líquidos, observamos que a dilatação real deles depende do recipiente no qual estão inseridos e é sempre menor que a dilatação aparente observada. Levando-se em conta os conceitos cientificamente corretos, das afirmações acima está(ao) correta(s): a) apenas I. b) apenas II. c) apenas III. d) I e II. e) I, II e III. Resolução: I- (V) As dilatações ocorrem em todas as direções. II- (F) Depende também de suas dimensões iniciais, já que a dilatação também é proporcional a ela. III- (F) A dilatação do real de um líquido não depende da dilatação do frasco. Apenas a aparente depende. (Resposta A) 35) (ACAFE – SC) O fenômeno da dilatação térmica pode ser usado para manter um sistema numa temperatura aproximadamente constante através de um dispositivo chamado termostato. Usa-se uma lâmina bimetálica (duas hastes de materiais diferentes, porém com mesmo comprimento inicial) que se encurva à medida que a temperatura muda e que pode abrir ou fechar circuitos elétricos, ou válvulas, e assim manter o sistema, como um ferro elétrico, um forno ou um sistema de ar condicionado em uma temperatura prédeterminada. A figura abaixo mostra esquematicamente um dispositivo desse tipo onde, ao aumentar a temperatura da lâmina, ela se curva para _______ se o coeficiente de dilatação da haste A for _____ coeficiente de dilatação da haste B. A alternativa correta que completa as lacunas acima, em seqüência, é: a) baixo - menor que o b) cima - maior que o c) cima - menor que o d) baixo - igual ao e) cima - igual ao Resolução: Se αA > αB e no caso de aquecimento a lâmina se curva para cima fechando o circuito. Portanto: cima; maior que o. (Resposta B) 36) (ACAFE – SC) Em um laboratório de física, um grupo de alunos aquece uma barra metálica homogênea, A, de comprimento L0, a partir de uma temperatura inicial T0 e mede o comprimento da barra para cada grau de aumento da temperatura. Um segundo grupo esfria uma barra idêntica, B, a partir da mesma temperatura inicial T0 e mede o comprimento da barra para cada grau de diminuição da temperatura. Depois de encerradas as medidas, os alunos colocam os dados em um mesmo gráfico L x T, onde L representa o comprimento das barras A e B, e T representa a temperatura correspondente. A alternativa que apresenta o gráfico correto é: Resolução: Se as barras são idênticas, para cada grau de variação de temperatura, a dilatação será igual a contração. Portanto. (Resposta D) 37) (MACK – SP) Um conjunto de certos dispositivos para laboratório, proveniente dos Estados Unidos, contém dentre outros produtos, uma pequena lâmina retangular. No livreto de informações, existe a recomendação para não expor essa lâmina a temperaturas inferiores a 30 ºF e tampouco superiores a 85 ºF. Um estudante
  • 9. 9 brasileiro, interessado em saber de que material era constituída a lâmina, descobriu, através de um ensaio, que, durante um aquecimento nesse intervalo de temperatura, a área da lâmina aumentava de 0,165%. Consultando a tabela abaixo, pode-se afirmar que, possivelmente, a lâmina é de material Material Coeficiente de dilatação linear ( o C- 1 ) Ferro 12.10-6 Ouro 15.10-6 Bronze 18.10-6 Alumínio 22.l0-6 Chumbo 27.10-6 a) ferro. b) ouro. c) bronze. d) alumínio. e) chumbo. Resolução: ∆ = 55 ºF = 30,55ºC e ∆A = A0.β.∆  0,165% A0 = A0.β.30,55  β = 5,4 . 10-5 e como β = 2α temos α = 27.10-5 ºC-1 (chumbo). (Resposta E) 38) (CEFET – PR) Analise as assertivas abaixo e marque (V) verdadeiro ou (F) falso. 1- ( ) A dilatação real de um líquido será maior do que a dilatação aparente observada. 2- ( ) Entre 0ºC e 4ºC há um aumento da temperatura da água e aumento também da sua densidade. 3- ( ) O vidro temperado Pirex pode ser levado diretamente ao fogo sem trincar porque seu coeficiente de dilatação é maior que o do vidro comum. 4- ( ) Ao aquecermos uma aliança de ouro, 18 quilates, ocorrerá dilatação em todas as dimensões, inclusive para a região interna da aliança. 5- ( ) Três gases diferentes, com o mesmo volume e estando a mesma temperatura inicial, se forem aquecidos de forma idêntica e mantidos a uma pressão constante, apresentarão o mesmo coeficiente de dilatação volumétrica. A alternativa correta é: a) V, F, F, V, F b) F, F, V, V, F c) V, V, F, F, V d) F, F, F, V, F e) V, V, F, V, F Resolução: 1-(V) O coeficiente de dilatação de um líquido é, normalmente, maior que o do frasco. Assim a dilatação real é maior que a aparente. 2-(V) Ocorrerá uma contração em seu volume e conseqüentemente em sua densidade. 3-(F) ele não trinca por sofrer pouquíssima dilatação em virtude do seu baixo coeficiente. 4-(F) ocorrerá um aumento no diâmetro da aliança. 5-(V) Já que o mesmo depende do volume inicial e da natureza do gás. (Resposta C) 39) (UFC – CE) Numa experiência de laboratório, sobre dilatação superficial, foram feitas várias medidas das dimensões de uma superfície S de uma lâmina circular de vidro em função da temperatura T. Os resultados das medidas estão representados no gráfico abaixo. Com base nos dados experimentais fornecidos no gráfico, pode-se afirmar corretamente que o valor numérico do coeficiente de dilatação linear do vidro é: a) 24x10–6 o C–1 . b) 18x10–6 o C–1 . c) 12x10–6 o C–1 . d) 9x10–6 o C–1 . e) 6x10–6 o C–1 Resolução: ∆A = A0.β.∆  (25,00180 – 25,00000) = 25. β.(34 – 30)  β = 18,1.10–6 e sendo β = 2α temos α = 0,05.10–6 ºC–1 (Resposta D) 40) (UFRN – RN) O dispositivo mostrado na figura abaixo é utilizado em alguns laboratórios escolares, para determinar o coeficiente de dilatação linear de um sólido. Nesse dispositivo, o sólido tem a forma de um tubo de comprimento L0, inicialmente a temperatura ambiente, no qual se faz passar vapor de água em ebulição até que o tubo atinja a temperatura do vapor ao entrar em equilíbrio térmico com este. Há, no dispositivo, dois termômetros, TE1 e TE2, e um micrômetro, MI. Face ao acima exposto, é correto afirmar que, para a determinação do coeficiente de dilatação linear desse tubo,
  • 10. 10 a) tanto o termômetro TE1 como o TE2 medem a variação de temperatura do tubo, e o micrômetro mede o comprimento inicial do tubo. b) o termômetro TE1 mede a temperatura ambiente, o termômetro TE2 mede a temperatura do vapor, e o micrômetro mede a variação de comprimento do tubo. c) o termômetro TE1 mede a temperatura do vapor, o termômetro TE2 mede a temperatura ambiente, e o micrômetro mede o comprimento final do tubo. d) tanto o termômetro TE1 como o TE2 medem a variação de temperatura do tubo, e o micrômetro mede a variação de comprimento do tubo Resolução: Os termômetros indicarão a variação de temperatura do tubo em relação a do ambiente e o micrômetro medirá a dilatação do tubo. (Resposta E) 41) (UEBA – BA) Uma peça de zinco é construída a partir de uma chapa quadrada de lado 30cm, da qual foi retirado um pedaço de área de 500cm². Elevando-se de 50°C a temperatura da peça restante, sua área final, em cm², será mais próxima de: Dado: coeficiente de dilatação linear do zinco = 2,5 . 10–5 °C–1 a) 400 b) 401 c) 405 d) 408 e) 416 Resolução: A peça restante terá área inicial de 30.30 – 500 = 400 cm². A dilatação será ∆A = A0.β.∆  ∆A = 400.(2. 2,5 . 10–5 ).50 = 1cm² e assim a área final da peça será 401 cm². (Resposta B) 42) (UNESP – SP) Duas lâminas metálicas, a primeira de latão e a segunda de aço, de mesmo comprimento à temperatura ambiente, são soldadas rigidamente uma à outra, formando uma lâmina bimetálica, conforme a figura. O coeficiente de dilatação térmica linear do latão é maior que o do aço. A lâmina bimetálica é aquecida a uma temperatura acima da ambiente e depois resfriada até uma temperatura abaixo da ambiente. A figura que melhor representa as formas assumidas pela lâmina bimetálica, quando aquecida (forma à esquerda) e quando resfriada (forma à direita), é Resolução: Ao ser aquecido, o latão dilata mais que o aço e ao ser resfriado contrai mais. (Resposta C) 43) (CESGRANRIO – RJ) Um bloco de certo metal tem seu volume dilatado de 200 cm³ para 206 cm³ quando sua temperatura aumenta de 20 ºC para 520 ºC. Se um fio desse mesmo metal, tendo 100 cm de comprimento a 20ºC for aquecido até a temperatura de 520 ºC, então seu comprimento em centímetros passará a valer: a) 101. b) 102. c) 103. d) 106. e) 112. Resolução: ∆V = V0.γ. ∆  (206 – 200) = 2000.(3α).(520 – 20)  α = 2.10–5 °C–1 e ∆L = L0.α.∆  ∆L = 100. 2.10–5 .500 = 1cm. Assim L = 101 cm. (Resposta A) 44) (UFMT – MT – Modificado) Uma peça retangular maciça de alumínio encaixa-se perfeitamente em um buraco feito em outra chapa de alumínio. Analise as afirmativas e dê como resposta a soma dos números que antecedem as afirmações corretas: (01) Se a peça e a chapa forem aquecidas até uma mesma temperatura, a peça passará a não se encaixar perfeitamente no buraco da chapa. (02) Se somente a chapa for aquecida, a peça se encaixará com folga no buraco da chapa. (04) Se somente a chapa for resfriada, a peça passará a não se encaixar no buraco da chapa. (08) Se somente a peça for resfriada, então ela passará a se encaixar com folga no buraco da chapa. (16) Se somente a peça for aquecida, então ela passará a não se encaixar no buraco da chapa. Resolução: 01 (F) A dilatação diametral será a mesma. 02 (V) Pois o diâmetro da chapa irá aumentar. 04 (V) Pois seu diâmetro será reduzido em relação ao da peça. 08 (V) Pois seu diâmetro se tornará menor que o da chapa. 16 (V) Seu diâmetro se tornará maior que o da chapa.
  • 11. 11 (Resposta: 30 (02 + 04 + 08 + 16)) 45) (ACAFE – SC) Um fio, de coeficiente linear = 2x10-5 ºC-1 , tem comprimento igual a 160cm. Esse fio é dobrado de modo a formar um quadrado e suas extremidades são soldadas. Após sofrer um aumento de temperatura de 250ºC, a área interna desse quadrado: 2 . b) 2 . 2 . 2 . Resolução: A área inicial do quadrado (lado 160 cm/4 = 40 cm) será 40.40 = 1600 cm². O perímetro do novo quadrado será: ∆L = L0.α.∆  ∆L = 160. 2.10–5 .250  ∆L = 0,8 cm. (L = 160,8/4 = 40,2 cm. A nova área do quadrado fica 40,2² = 1616,04 cm². Assim o acréscimo de área é 1616,04 – 1600 = 16,04 cm².(Resposta B) 46) (UNILASALLE – SP) Uma técnica empregada por algumas indústrias para aumentar a pressão sobre parafusos de fixação é a de utilizar parafusos com bitola levemente superior à do orifício de fixação. Nesse caso, aquece-se a placa onde será fixado o parafuso. Isso faz com que o orifício se dilate e o parafuso possa ser colocado. Após o sistema atingir a temperatura ambiente, o parafuso fica praticamente preso ao orifício. Suponha que um parafuso de aço de diâmetro 1,0000 cm deve ser fixado num orifício circular de 0,9978 cm de diâmetro, em uma placa também de aço. A variação mínima na temperatura da placa, para que o parafuso possa ser fixado, é a seguinte (dado: αaço = 11.10-6 ºC-1 ) a) 400 ºC. b) 200 ºC. c) 100 ºC. d) 75 ºC. e) 50 ºC. Resolução: ∆L = L0.α.∆  (0,9978 – 1,0000) = 0,9978. 11.10-6 . ∆  ∆ = -200 ºC. (Resposta B) 47) (UFPI – PI) O comportamento de dilatação anômalo da água é verificado pelo fato de esta se contrair quando sua temperatura aumenta de 0 °C a 4 °C. A partir desta temperatura, a água se dilata como a maioria das substâncias. Com relação a esse assunto, coloque V, para verdadeiro, ou F, para falso. 1 ( ) Em regiões em que a temperatura atinge valores abaixo de 0°C, os lagos ficam congelados a partir da superfície. Isso ocorre porque a água da superfície, ao se resfriar até 4°C, atinge sua densidade máxima, e por convecção, vai para o fundo. 2 ( ) Numa região de clima muito frio, onde a temperatura ambiente chega a 0°C ou menos, a água mais fria por ser mais densa, desce para o fundo do lago, ocorrendo o congelamento inicialmente no fundo. 3 ( ) Numa região de clima muito frio, quando a temperatura das superfícies dos lagos se tornam inferior a 4°C, não ocorrerá mais movimentação da água por diferença de densidade. Assim, forma-se gelo na superfície quando a temperatura ambiente atingir 0°C, sendo que a água do fundo do lago continuará líquida. 4 ( ) Quando a temperatura ambiente de regiões muito frias (onde a temperatura chega abaixo de 0°C) está subindo, o nível da água dos lagos subirá somente a partir do momento em que a temperatura da água passar de 4°C. Resolução: 1 (V) a água mais densa tende a ir para o fundo. 2(F) no fundo a água encontra-se a 4 ºC. Portanto o congelamento ocorrerá primeiramente na superfície. 3(V). A água mais densa e mais quente permanecerá no fundo. 4(V) A dilatação da água torna-se normal após 4 ºC . (Respostas: 1(V); 2(F); 3(V); 4(V). 48) (ENEM) Durante uma ação de fiscalização em postos de combustíveis, foi encontrado um mecanismo inusitado para enganar o consumidor. Durante o inverno, o responsável por um posto de combustível compra álcool por R$ 0,50/litro, a uma temperatura de 5 °C. Para revender o líquido aos motoristas, instalou um mecanismo na bomba de combustível para aquecê-lo, para que atinja a temperatura de 35 °C, sendo o litro de álcool revendido a R$ 1,60. Diariamente o posto compra 20 mil litros de álcool a 5 ºC e os revende. Com relação à situação hipotética descrita no texto e dado que o coeficiente de dilatação volumétrica do álcool é de 1×10-3 ºC-1 , desprezando-se o custo da energia gasta no aquecimento do combustível, o ganho financeiro que o dono do posto teria obtido devido ao aquecimento do álcool após uma semana de vendas estaria entre a) R$ 500,00 e R$ 1.000,00. b) R$ 1.050,00 e R$ 1.250,00. c) R$ 4.000,00 e R$ 5.000,00. d) R$ 6.000,00 e R$ 6.900,00. e) R$ 7.000,00 e R$ 7.950,00. Resolução: Em 1 semana temos 20000L.7 = 140000L. A dilatação será: ∆V = V0.γ. ∆ = 140000. 1×10-3 . (35 – 5)  ∆V = 4200 L. O ganho financeiro será 4200. R$ 1,10 = R$4620,00. (Resposta C)
  • 12. 12 49) A gasolina é vendida por litro, mas em sua utilização como combustível, a massa é o que importa. Um aumento da temperatura do ambiente leva a um aumento no volume da gasolina. Para diminuir os efeitos práticos dessa variação, os tanques dos postos de gasolina são subterrâneos. Se os tanques não fossem subterrâneos: I. Você levaria vantagem ao abastecer o carro na hora mais quente do dia pois estaria comprando mais massa por litro de combustível. II. Abastecendo com a temperatura mais baixa, você estaria comprando mais massa de combustível para cada litro. III. Se a gasolina fosse vendida por kg em vez de por litro, o problema comercial decorrente da dilatação da gasolina estaria resolvido. Destas considerações, somente a) I é correta. b) II é correta. c) III é correta. d) I e II são corretas. e) II e III são corretas. Resolução: I(F) o combustível estaria dilatado. II(V) o combustível estaria comprimido. III(V) PIS dilatado ou comprimido a massa do combustível não se alteraria com a dilatação. (Resposta E) 50) (ITA – SP) Um pequeno tanque, completamente preenchido com 20,0 litros de gasolina a 0°F, é logo a seguir transferido para uma garagem mantida à temperatura de 70°F. Sendo  = 0,0012 °C–1 o coeficiente de expansão volumétrica da gasolina, a alternativa que melhor expressa o volume de gasolina em litros que vazará em conseqüência do seu aquecimento até a temperatura da garagem é a) 0,507 b) 0,940 c) 1,68 d) 5,07 e) 0,17 Resolução: ∆ = (5/9).70 = 38,88 ºC. Assim ∆V = V0.γ. ∆ = 20.0,0012.38,88  0,940 L. (Resposta B) 51) (PUC – RS) Um vendedor de gasolina colocou 20,0 x 10³ litros de gasolina no tanque de seu caminhão, à temperatura de 15,0 o C. Supondo que ele tenha vendido toda a gasolina à temperatura de 35,0 o C, e que o coeficiente de dilatação volumétrica dessa gasolina seja igual a 1,00 x 10-3 o C-1 , o acréscimo de volume, em litros, devido à expansão térmica, foi a) 20 b) 40 c) 100 d) 200 e) 400 Resolução: ∆V = V0.γ. ∆ = 20.10³. 1,00 x 10-3 . (35 – 15) = 400 L. (Resposta E) 52) (UFF-RJ) O dono de um posto de gasolina consulta uma tabela de coeficientes de dilatação volumétrica obtendo γálcool = 10–3 °C–1 . Assim, ele verifica que se comprar 14 000 litros do combustível em um dia em que a temperatura do álcool é de 20°C e revendê-los num dia mais quente, em que esta temperatura seja de 30°C, estará ganhando: a) 1,4 . 10² litros. b) 1,4 . 10³ litros. c) 5,2 . 10³ litros. d) 1,5 . 104 litros. e) 5,2 . 104 litros. Resolução: ∆V = V0.γ. ∆ = 14000. 10–3 . (30 – 20) = 140 L. (Resposta A) 53) Um recipiente tem, a 0 o C, capacidade volumétrica de 20cm³ e a 100 °C sua capacidade é de 20,01cm³. Quando ele é completamente preenchido com certo líquido a 0°C, transbordam 0,05cm³ ao ser feito o referido aquecimento. Determine: o coeficiente de dilatação térmica volumétrica real do líquido. Resolução: O coeficiente de dilatação do frasco é ∆V = V0.γ. ∆  (20,01 – 20) = 20. γ.(100 – 0)  γF = 0,5. 10-5 ºC-1 . O coeficiente aparente é ∆V = V0.γ. ∆  0,05 = 20.γ.(100 – 0)  γAP = 0,25.10-5 ºC-1 O coeficiente real do líquido será γR = γF + γAP = 0,5. 10-5 + 0,25.10-5 = 0,75.10-5 ºC-1 (Resposta: 0,75.10-5 ºC-1 ) Calorimetria 54) (UFRRJ – RJ) Uma pessoa bebe 200 gramas de água a 20°C. Sabendo-se que a temperatura do seu corpo é praticamente constante e vale 36,5 °C, a quantidade de calor absorvido pela água é igual a: (calor específico da água = 1cal/g°C) a) 730cal. b) 15600cal. c) 3300cal. d) 1750cal e) 0,01750cal. Resolução: Q = m.c.∆ = 200.1.(36,5 – 20) = 3300 cal. (Resposta C) 55) (UNILASALLE – SP) Para tomar chimarrão, aquecem-se 500 g de água a 20 ºC até 80 ºC. A quantidade de energia fornecida à água, desprezando-se perdas, é a seguinte (dado: calor específico da água, c = 1 cal/gºC e 1caloria vale aproximadamente 4,0 J) a) 120 kJ. b) 120 MJ.
  • 13. 13 c) 120 J. d) 100 kJ. e) 100 J. Resolução: Q = m.c.∆ = 500.1.(80 – 20) = 30000 cal x 4 = 120000L = 120 kJ. (Resposta A) 56) (UERJ – RJ) Considere X e Y dois corpos homogêneos, constituídos por substâncias distintas, cujas massas correspondem, respectivamente, a 20 g e 10 g. O gráfico abaixo mostra as variações da temperatura desses corpos em função do calor absorvido por eles durante um processo de aquecimento. Determine as capacidades térmicas de X e Y e, também, os calores específicos das substâncias que os constituem. Resolução: CX = Q/∆ = 80/(281 – 273) = 10 cal/K ou 10cal/ºC e CY = Q/∆ = 40/(283 – 273) = 4 cal/k ou 4 cal/ºC. cX = CX/m = 10/20 = 0,5 cal/gºC e cY = CY/m = 4/10 = 0,4 cal/gºC. (Resposta: capacidades térmicas CX = 10cal/ºC e CY = 4 cal/ºC; calores específicos cX = 0,5 cal/gºC e cY = 0,4 cal/gºC) 57) (PUC – RS) Responder à questão seguinte com base no gráfico a seguir, referente à temperatura em função do tempo, de um corpo que está sendo aquecido e que absorve 20cal/s. A capacidade térmica do corpo é a) 20 cal/ºC b) 30 cal/ºC c) 40 cal/ºC d) 50 cal/ºC e) 60 cal/ºC Resolução: C = Q/∆ = (20.100)/(60 – 20) = 50 cal/ºC. (Resposta D) 58) (UNESP – SP) O gráfico representa a temperatura em função do tempo de um líquido aquecido em um calorímetro. Considerando-se desprezível a capacidade térmica do calorímetro e que o aquecimento foi obtido através de uma resistência elétrica, dissipando energia à taxa constante de 120 W, a capacidade térmica do líquido vale: a) 12 J/o C. b) 20 J/o C. c) 120 J/o C. d) 600 J/o C. e) 1 200 J/o C. Resolução: 120 W = 120 J/s e C = Q/∆ = (120.300)/(54 – 24)= 10/30 = 1200 J/ o C. (Resposta E) 59) (UFRN – RN) Diariamente, Dona Leopoldina coloca uma lata de refrigerante, cuja temperatura é de 30 ºC, numa caixa térmica contendo gelo e, após esperar algumas horas, bebe o refrigerante a uma temperatura de aproximadamente 5 ºC. Nesse caso, é correto afirmar que a diminuição da temperatura do refrigerante se explica porque, no interior da caixa térmica, a lata de refrigerante a) cede calor para o gelo, e este cede calor para ela, porém numa quantidade menor que a recebida. b) recebe frio do gelo, para o qual cede calor, porém numa quantidade menor que o frio recebido. c) cede calor para o gelo, e este cede calor para ela, porém numa quantidade maior que a recebida. d) recebe frio do gelo, para o qual cede calor, porém numa quantidade maior que o frio recebido. Resolução: O fluxo do calor se dá nos dois sentidos, sendo maior do refrigerante (maior temperatura) para o gelo (menor temperatura). (Resposta A) 60) (UFMS – MS) Quando a temperatura de 100g de água (calor específico = 4,2 J/(g.K) diminui de 25ºC para 5ºC, há liberação de calor.
  • 14. 14 É correto afirmar que a quantidade de calor liberada pela água é; a) 8 400 Joule. b) – 8 400 Joule. c) 2 000/4,2 Joule. d) – 2 000/4,2 Joule. e) – 84/100 Joule. Resolução: Q = m.c.∆ = 100. 4,2 (5 – 20) = – 8400 J. (Resposta B) 61) (UERJ – RJ) Um adulto, ao respirar durante um minuto, inspira, em média, 8,0 litros de ar a 20 ºC, expelindo-os a 37 ºC. Admita que o calor específico e a densidade do ar sejam, respectivamente, iguais a 0,24 cal.g-1 . ºC-1 e 1,2 g/L-1 . Nessas condições, a energia mínima, em quilocalorias, gasta pelo organismo apenas no aquecimento do ar, durante 24 horas, é aproximadamente igual a: a) 15,4 b) 35,6 c) 56,4 d) 75,5 Resolução: A massa de ar em 1 minuto será d = m/v  1,2 = m/8  m = 9,8 g. Em 24 horas teremos 24.60.9,6 = 13824 g. Assim Q = m.c.∆ = 13824. 0,24. (37 – 20) = 56401 cal ou  56,4 kcal. (Resposta C) 62) (CEFET – PR) Um pequeno aquecedor elétrico de imersão é usado para aquecer 100 g de água para uma xícara de café instantâneo. O aquecedor está rotulado com “200 W”, o que significa que ele converte energia elétrica em energia térmica com essa taxa. Calcule o tempo necessário para levar toda essa água de 20ºC para 100ºC, ignorando quaisquer perdas. (Considerar cágua = 1cal/gºC e 1 cal = 4,19 J) a) 40 s. b) 20 s. c) 400 s. d) 167,6 s. e) 1676 s. Resolução: Q = m.c.∆ = 100.4,19.(100 – 20) = 33520 J. Assim Pot = Q/∆t  200 = 35520/∆t  ∆t = 167,6 s. (Resposta D) 63) (PUCCAMP – SP) Admita que o corpo humano transfira calor para o meio ambiente na razão de 2,0kcal/min. Se esse calor pudesse ser aproveitado para aquecer água de 20°C até 100°C, a quantidade de calor transferida em 1 hora aqueceria uma quantidade de água, em kg, igual a: (calor específico da água = 1,0kcal/kg °C) Resolução: Em 1 hora teremos Q = 2.60 = 120 kcal. Assim: Q = m.c.∆  120 = m.1.20 m = 6 kg. (Resposta: 6 kg) 64) (MACK – SP) Na festa de seu aniversário, o aniversariante comeu salgadinhos e bebeu refrigerantes, ingerindo o total de 2 000 kcal. Preocupado com o excesso alimentar, ele pensou em perder as “calorias” adquiridas, ingerindo água gelada a 12 ºC, pois, estando o interior de seu organismo a 37 ºC, as “calorias” adquiridas seriam consumidas para aquecer a água. Admitindo que seu raciocínio esteja correto, o volume de água a 12 ºC que deve beber é: a) 60 litros. b) 65 litros. c) 70 litros. d) 80 litros. e) 90 litros. Resolução: Q = m.c.∆  2000000= m.1. (37 – 12)  m = 80000g o que corresponde a 80 L. (Resposta D) 65) (UNESP – SP) Massas iguais de cinco líquidos distintos, cujos calores específicos estão dados na tabela adiante, encontram-se armazenadas, separadamente e à mesma temperatura, dentro de cinco recipientes com boa isolação e capacidade térmica desprezível. Se cada líquido receber a mesma quantidade de calor, suficiente apenas para aquecê-lo, mas sem alcançar seu ponto de ebulição, aquele que apresentará temperatura mais alta, após o aquecimento, será: a) a água. b) o petróleo. c) a glicerina. d) o leite. e) o mercúrio. Resolução: O de menor calor específico sofrerá maior aquecimento já que cada grama necessitará de menos calor para variar sua temperatura em 1 grau. Portanto, em ordem decrescente de temperaturas teremos mercúrio; petróleo; glicerina; leite; água. (Resposta E) 66) (ENEM) A energia geotérmica tem sua origem no núcleo derretido da Terra, onde as temperaturas atingem 4.000 ºC. Essa energia é primeiramente produzida pela decomposição de materiais radiativos dentro do planeta. Em fontes geotérmicas, a água, aprisionada em um reservatório subterrâneo, é aquecida pelas
  • 15. 15 rochas ao redor e fica submetida a altas pressões, podendo atingir temperaturas de até 370 ºC sem entrar em ebulição. Ao ser liberada na superfície, à pressão ambiente, ela se vaporiza e se resfria, formando fontes ou gêiseres. O vapor de poços geotérmicos é separado da água e é utilizado no funcionamento de turbinas para gerar eletricidade. A água quente pode ser utilizada para aquecimento direto ou em usinas de dessalinização. Roger A. Hinrichs e Merlin Kleinbach. Energia e meio ambiente. Ed. ABDR (com adaptações). Depreende-se das informações acima que as usinas geotérmicas a) utilizam a mesma fonte primária de energia que as usinas nucleares, sendo, portanto, semelhantes os riscos decorrentes de ambas. b) funcionam com base na conversão de energia potencial gravitacional em energia térmica. c) podem aproveitar a energia química transformada em térmica no processo de dessalinização. d) assemelham-se às usinas nucleares no que diz respeito à conversão de energia térmica em cinética e, depois, em elétrica. e) transformam inicialmente a energia solar em energia cinética e, depois, em energia térmica. Resolução: A energia térmica resultante das reações nucleares é transformada em cinética e em seguida em elétrica, como nas usinas nucleares. (Resposta D) 67) (FUVEST – SP) Um fogão, alimentado por um botijão de gás, com as características descritas no quadro abaixo, tem em uma de suas bocas um recipiente com um litro de água que leva 10 minutos para passar de 20 ºC a 100 ºC. Para estimar o tempo de duração de um botijão, um fator relevante é a massa de gás consumida por hora. Mantida a taxa de geração de calor das condições acima, e desconsideradas as perdas de calor, a massa de gás consumida por hora, em uma boca de gás desse fogão, é aproximadamente a) 8 g b) 12 g c) 48 g d) 320 g e) 1920 g Resolução: 1L contém 1kg de água. Assim, em 10 minutos temos Q = m.c.∆ = 1.1.(100 – 20) = 80 kJ e como o calor de combustão é 40000 kJ/kg, temos 40000 = 80/m  m = 0,002 kg ou 2 g e em 1 hora fica 2 x 60 min/10 min = 12 g. (Resposta B) 68) (UERJ – RJ) O gráfico abaixo representa o consumo de oxigênio de uma pessoa que se exercita, em condições aeróbicas, numa bicicleta ergométrica. Considere que o organismo libera, em média, 4,8 kcal para cada litro de oxigênio absorvido. A energia liberada no período entre 5 e 15 minutos, em kcal, é: a) 48,0 b) 52,4 c) 67,2 d) 93,6 Resolução: Q = 1,4 (15 – 5) = 14 L x 4,8 = 67,2 kcal. (Resposta C) 69) (UERJ – RJ) O excesso de gordura no organismo é nocivo à saúde. Considere uma pessoa, com massa corporal estável, que deseje perder gordura, sem alterar sua dieta alimentar. Para essa pessoa, um dispêndio energético de 9 kcal em atividades físicas corresponde à perda de 1 g de gordura corporal. Para perder 6,0 kg de gordura, o tempo, em minutos, que ela necessita dedicar a atividades físicas, despendendo, em média, 12 kcal/min, corresponde a: a) 2,0. 10² b) 4,5.10³ c 4 d) 6,0.105 Resolução: A perda de 6000 g de gordura requer um dispêndio energético de 6000.9 = 54000 kcal. Assim, ∆t = 54000/12 = 4500 minutos (Resposta B) 70) (MACK – SP – Modificado) A preocupação com a qualidade e com o tempo de vida leva o homem moderno urbano a manter uma dieta alimentar adequada, acompanhada de exercícios físicos corretos. - Considere que, para Pedro, o valor energético adequado, consumido por almoço, seja de 700 kcal de alimentos. - Considere, ainda, que Pedro, no almoço, ingira 100 g de arroz, 100 g de feijão, 150 g de bife e
  • 16. 16 50 g de batata frita, além de uma lata de refrigerante. Consultando a tabela, pode-se afirmar que, para consumir o excesso energético ingerido, Pedro deve correr, aproximadamente: Alimento Energia kcal/g Arroz 3,6 Feijão 3,4 Bife 3,8 Batata frita 1,4 Considere: 1) Lata de refrigerante 48 kcal 2) Energia consumida em corrida 1080 kcal/hora a) 80 minutos. b) 40 minutos. c) 34 minutos. d) 60 minutos. e) 92 minutos. Resolução: A energia será 100.3,6 + 100.3,4 + 150.3,8 + 50.1,4 + 48 = 360 + 340 + 570 + 48 = 1318 kcal. O excesso é 1318 – 700 = 618 kcal. Assim, ∆t = 618/1080x 60 min  34 min = (Resposta C) 71) (UFF – RJ) Para se resfriar um motor em funcionamento, é necessário acionar seu sistema de refrigeração, podendo-se usar as substâncias ar ou água. A massa de ar m1 e a massa de água m2 sofrem a mesma variação de temperatura e proporcionam a mesma refrigeração ao motor. Neste caso, a razão m1/m2 é: Dados: calor específico da água = 1,0 cal/g o C calor específico do ar = 0,25 cal/g o C a) 4,0 b) 0,67 c) 2,0 d) 0,25 e) 1,0 Resolução: Q1 = Q2  m1.c1.∆ = m2.c2.∆  m1. 1 = m2.0,25  m1. / m2 = 0,25. (Resposta D) 72) (UFPI – PI) Para perfurar um buraco num bloco de cobre, de 1,00 kg de massa, um operário utilizou uma furadeira de potência de 200W durante 200s . Supondo-se que somente 70% da energia envolvida no processo de perfuração tenha sido absorvida pelo bloco na forma de calor, o aumento médio da temperatura do bloco foi de, aproximadamente: Dados: o calor específico do cobre é 0,093cal/go C e 1cal = 4,2 J. a) 30,7 o C. b) 71,7 o C. c) 1,02×10² o C. d) 9,30×10² o C. e) 9,52×10³ o C. Resolução: Pot = Q/∆t  70%.200 = Q/200  Q = 28000 J. Sendo Q = m.c.∆ fica 28000 = 1000. 0,093.4,2.∆  ∆  71,7 ºC. (Resposta B) 73) (UERJ – RJ) Duas barras metálicas A e B, de massas mA=100 g e mB=120 g, inicialmente à temperatura de 0 ºC, são colocadas, durante 20 minutos, em dois fornos. Considere que toda a energia liberada pelas fontes térmicas seja absorvida pelas barras. O gráfico a seguir indica a relação entre as potências térmicas fornecidas a cada barra e o tempo de aquecimento. Após esse período, as barras são retiradas dos fornos e imediatamente introduzidas em um calorímetro ideal. O diagrama abaixo indica a variação da capacidade térmica de cada barra em função de sua massa. A temperatura que corresponde ao equilíbrio térmico entre as barras A e B é, em ºC, aproximadamente igual a: a) 70 b) 66 c) 60 d) 54 Resolução: As quantidades de calor fornecidas a cada barra em 20 min serão dadas através das áreas do 1º diagrama. Assim: QA = 600.20/2 = 6000 cal e QB = 240.20/2 = 2400 cal. As temperaturas iniciais das barras serão, portanto: CA = QA/∆A  80 = 6000/∆A  ∆A = 75 ºC e CB = QB/∆B  48 = 2400/∆B  ∆B = 50 ºC. O equilíbrio térmico se dará a: mA.cA.∆A + mB.cB.∆ B = 0  80( – 75) +
  • 17. 17 48( – 50)   = 65,625 ºC. (Resposta B) 74) (UFF – RJ) Duelo de Gigantes: O rio Amazonas é o maior rio do mundo em volume d’água com uma vazão em sua foz de, aproximadamente, 175 milhões de litros por segundo. A usina hidroelétrica de Itaipu também é a maior do mundo, em operação. A potência instalada da usina é de 12,6.109 W. Suponha que toda essa potência fosse utilizada para aquecer a água que flui pela foz do rio Amazonas, sem que houvesse perdas de energia. Veja, 24/09/ 2003. (Adaptado) Nesse caso, a variação de temperatura dessa água, em grau Celsius, seria da ordem de: Dados: calor específico da água c = 1,0 cal/g o C, densidade da água = 1,0 g/cm³ e 1 cal = 4,2 Joules a) 10–2 b) 10–1 c) 100 d) 101 e) 102 Resolução: Pot = Q/∆t = m.c.∆/∆t  12,6.109 = 175. 109 .1.∆/1  ∆ = 0,072 = 7,2 . 10–2 ºC. ordem de grandeza 10–1 já que 7,2 > 10 (Resposta B) 75) (UFG – GO) O cérebro de um homem típico, saudável e em repouso, consome uma potência de aproximadamente 16 W. Supondo que a energia gasta pelo cérebro em 1 min fosse completamente usada para aquecer 10 ml de água, a variação de temperatura seria de, aproximadamente, Densidade da água: 1,0·10³ kg/m³ Calor específico da água: 4,2·10³ J/kg·o C a) 0,5 o C b) 2 o C c) 11 o C d) 23 o C e) 48 o C Resolução: Pot = Q/∆t  16 = Q/60  Q = 960 J e, sendo Q = m.c.∆, fica 960 = 10.4,2. ∆  ∆ = 22,85 ºC. (Resposta D) 76) (FGV – SP) Os trajes de neopreme, um tecido emborrachado e isolante térmico, são utilizados por mergulhadores para que certa quantidade de água seja mantida próxima ao corpo, aprisionada nos espaços vazios no momento em que o mergulhador entra na água. Essa porção de água em contato com o corpo é por ele aquecida, mantendo assim uma temperatura constante e agradável ao mergulhador. Suponha que, ao entrar na água, um traje retenha 2,5 L de água inicialmente a 21°C. A energia envolvida no processo de aquecimento dessa água até 35°C é: Dados: densidade da água = 1 kg/L e calor específico da água = 1 cal/(g°C) a) 25,5 kcal. b) 35,0 kcal. c) 40,0 kcal. d) 50,5 kcal. e) 70,0 kcal. Resolução: Q = m.c.∆ = 2500. 1. (35 – 21) = 35000cal ou 35 kcal. (Resposta B) 77) (UNIFESP – SP) Dois corpos, A e B, com massas iguais e a temperaturas tA = 50°C e tB = 10°C, são colocados em contato até atingirem a temperatura de equilíbrio. O calor específico de A é o triplo do de B. Se os dois corpos estão isolados termicamente, a temperatura de equilíbrio é: a) 28°C b) 30°C c) 37°C d) 40°C e) 45°C Resolução: QA + QB = 0  mA.cA.∆A + mB.cB.∆B = 0  m. 3 cB.( – 50) + m.cB.( – 10) = 0  3 – 150 +  – 10 = 0   = 40 ºC. (Resposta D) 78) (UFRJ – RJ) Três amostras de um mesmo líquido são introduzidas num calorímetro adiabático de capacidade térmica desprezível: uma de12 g a 25 °C, outra de 18 g a 15 °C e a terceira de 30 g a 5 °C. Calcule a temperatura do líquido quando se estabelecer o equilíbrio térmico no interior do calorímetro. Resolução: QA + QB + QC = 0  mA.cA.∆A + mB.cB.∆B + mC.cC.∆C = 0  12.c.( – 25) + 18.c.( – 15) + 30.c.( – 5) = 0   = 12 ºC. (Resposta: 12 ºC) 79) (FUVEST – SP) Dois recipientes iguais A e B, contendo dois líquidos diferentes, inicialmente a 20°C, são colocados sobre uma placa térmica, da qual recebem aproximadamente a mesma quantidade de calor. Com isso, o líquido em A atinge 40°C, enquanto o líquido em B, 80°C. Se os recipientes forem retirados da placa e seus líquidos misturados, a temperatura final da mistura ficará em torno de:
  • 18. 18 a) 45°C b) 50°C c) 55°C d) 60°C e) 65°C Resolução: A relação entre as capacidades térmicas dos líquidos é: CA/CB = (Q/∆A)/ (Q/∆B)  CA/CB = ∆B/∆A  CA/CB = (80 – 20)/(40 – 20) = 3 ou seja CA = 3CB Na mistura fica QA + QB = 0  mA.cA.∆A + mB.cB.∆B = 0  CA. ( – 40) + CB. ( – 80) = 0  3CB. ( – 40) + CB. ( – 80) = 0  3 – 120 +  – 80 = 0   = 50 ºC. (Resposta B) 80) (UFMG – MG) Numa aula de Física, o Professor Carlos Heitor apresenta a seus alunos esta experiência: dois blocos, um de alumínio e outro de ferro, de mesma massa e, inicialmente, à temperatura ambiente, recebem a mesma quantidade de calor, em determinado processo de aquecimento. O calor específico do alumínio e o do ferro são, respectivamente, 0,90 J/go C e 0,46 J/go C. Questionados quanto ao que ocorreria em seguida, dois dos alunos, Alexandre e Lorena, fazem, cada um deles, um comentário: • Alexandre: “Ao final desse processo de aquecimento, os blocos estarão à mesma temperatura.” • Lorena: “Após esse processo de aquecimento, ao se colocarem os dois blocos em contato, fluirá calor do bloco de ferro para o bloco de alumínio.” Considerando-se essas informações, é correto afirmar que: a) apenas o comentário de Alexandre está certo. b) apenas o comentário de Lorena está certo. c) ambos os comentários estão certos. d) nenhum dos dois comentários está certo. Resolução: Ao receberem iguais quantidades de calor, o bloco de ferro fica mais aquecido pois seu calor específico é menor, sendo que cada grama desse material necessita de menos calor para cada grau de variação em sua temperatura. Assim Alexandre está errado pois o ferro está mais aquecido. Lorena está certa pois o fluxo de calor é maior do ferro para o alumínio. (Resposta B) 81) (INATEL – MG) Calor de combustão é a quantidade de calor liberada na queima de uma unidade de massa do combustível. O calor de combustão do gás de cozinha é 6,0.106 cal/kg. Calcule o volume de água, em litros, que pode ser aquecido de 20°C a 100°C com um botijão de gás de cozinha de 13 kg, admitindo que esse processo tenha uma eficiência de 40%. Dados: calor específico da água c = 1cal/g°C, densidade da água d = 1,0.10³g/L) Resolução: Em 1 botijão temos CC = Q/m  6,0.106 = Q/13  Q = 78. 106 cal. Sendo a eficiência 40% então o calor aproveitado será Q = 40%.78. 106 = 31,2. 106 cal. A massa de água será Q = m.c.∆  31,2. 106 = m.1. (100 – 0)  m = 31,2. 104 g = 312.10³g = 312 kg e sendo a densidade igual a 1 teremos V = 312 L. (Resposta: 312 litros) 82) (FUVEST – SP) O processo de pasteurização do leite consiste em aquecê-lo a altas temperaturas, por alguns segundos, e resfriá-lo em seguida. Para isso, o leite percorre um sistema, em fluxo constante, passando por três etapas: I) O leite entra no sistema (através de A), a 5 ºC, sendo aquecido (no trocador de calor B) pelo leite que já foi pasteurizado e está saindo do sistema. II) Em seguida, completa-se o aquecimento do leite, através da resistência R, até que ele atinja 80 ºC. Com essa temperatura, o leite retorna a B. III) Novamente em B, o leite quente é resfriado pelo leite frio que entra por A, saindo do sistema (através de C), a 20ºC. Em condições de funcionamento estáveis, e supondo que o sistema seja bem isolado termicamente, pode-se afirmar que a temperatura indicada pelo termômetro T, que monitora a temperatura do leite na saída de B, é aproximadamente de a) 20 ºC b) 25 ºC c) 60 ºC d) 65 ºC e) 75 ºC Resolução: QCED = m.c.(20 – 80) = -60mc QREC = m.c. ( – 5) e, sendo QCED +QREC = 0, fica -60mc + m.c. ( – 5) = 0   – 5 = 60 e  = 65 ºC. (Resposta D) 83) (UFPR – PR) Um recipiente termicamente isolado contém 500 g de água na qual se mergulha uma barra metálica homogênea de 250 g. A temperatura inicial da água é 25,0 °C e
  • 19. 19 a da barra 80,0 °C. Considere o calor específico da água igual a 1,00 cal/g.°C, o do metal igual a 0,200 cal/g.°C e despreze a capacidade térmica do recipiente. Com base nesses dados, é correto afirmar que: (01) A temperatura final de equilíbrio térmico é de 52,5 °C. (02) O comprimento da barra permanece constante durante o processo de troca de calor. (04) A temperatura inicial da barra, na escala kelvin, é de 353 K. (08) A quantidade de calor recebida pela água é igual à cedida pela barra. (16) A energia interna final da água, no equilíbrio térmico, é menor que sua energia interna inicial. Soma = ( ) Resolução: 01(F) QA + QB = 0  mA.cA.∆A + mB.cB.∆B = 0  500.1.(  - 25) + 250.0,2. ( - 80) = 0   = 30 ºC. 02(F) Já que a barra diminui sua temperatura então sofrerá uma contração térmica. 04(V) T(K) = C + 273 = 80 + 273 = 353 K 08(V) Já que não ocorrem trocas de calor com o meio externo e tampouco com o recipiente. 16(F) Já que a água experimentou uma elevação em sua temperatura. (Resposta: 12 (04 + 08)) 84) (ITA – SP) Colaborando com a campanha de economia de energia, um grupo de escoteiros construiu um fogão solar, consistindo de um espelho de alumínio curvado que foca a energia térmica incidente sobre uma placa coletora. O espelho tem um diâmetro efetivo de 1,00m e 70% da radiação solar incidente é aproveitada para de fato aquecer certa quantidade de água. Sabemos ainda que o fogão solar demora 18,4 minutos para aquecer 1,00 L de água desde a temperatura de 20 °C até 100 °C, e que 4,186.10³ J é a energia necessária para elevar a temperatura de 1,00 L de água de 1,000 K. Com base nos dados, estime a intensidade irradiada pelo Sol na superfície da Terra, em W/m². .Justifique. Resolução: A potência solar útil no espelho será: Pot = Q/∆t  Pot = m.c.∆/∆t = 1kg.418,6J/(kg.K).80K/18,4.60s = 30,33 J/s = 30,33 W. A potência total será 30,33/70% = 43,33 W. assim a intensidade da radiação é: I = Pot/A = 43,33/,r² = 43,33/.0,5²  55,17 W/m². (Resposta: 55,17 W/m²) 85) (UFPE – PE) Um certo volume de um líquido A, de massa M e que está inicialmente a 20ºC, é despejado no interior de uma garrafa térmica que contém uma massa 2M de um outro líquido, B, na temperatura de 80ºC. Se a temperatura final da mistura líquida resultante for de 40ºC, podemos afirmar que a razão cA/cB entre os calores específicos das substâncias A e B vale: a) 6 b) 4 c) 3 d) ½ e) 1/3 Resolução: QA + QB = 0  mA.cA.∆A + mB.cB.∆B = 0  M. cA.(40 – 20) + 2M. cB.(40 – 80) = 0  20cA – 80.cB = 0  cA/cB = 4. (Resposta B) 86) (ENEM) Nos últimos anos, o gás natural (GNV: gás natural veicular) vem sendo utilizado pela frota de veículos nacional, por ser viável economicamente e menos agressivo do ponto de vista ambiental. O quadro compara algumas características do gás natural e da gasolina em condições ambiente. d (g/cm³) GNV 0,8 Gasolina 738 Apesar das vantagens no uso de GNV, sua utilização implica algumas adaptações técnicas, pois, em condições ambientes, o volume de combustível necessário, em relação ao de gasolina, para produzir a mesma energia, seria: a) muito maior, o que requer um motor muito mais potente. b) muito maior, o que requer que ele seja armazenado a alta pressão. c) igual, mas sua potência será muito menor. d) muito menor, o que o torna o veículo menos eficiente. e) muito menor, o que facilita sua dispersão para a atmosfera. Resolução: A baixa densidade do GNV requer que, para se obter uma massa equivalente a da gasolina, o volume deverá ser bem maior requerendo armazenamento sb alta pressão para reduz[i-lo a ponto de ser armazenado. (Resposta B) Mudanças de estados físicos da matéria 87) (UNESP – SP) Nos quadrinhos da tira, a mãe menciona as fases da água conforme a mudança das estações.
  • 20. 20 Entendendo “boneco de neve” como sendo “boneco de gelo” e que com o termo “evaporou” a mãe se refira à transição água/vapor, pode-se supor que ela imaginou a seqüência gelo/água/vapor/água. As mudanças de estado que ocorrem nessa seqüência são: a) fusão, sublimação e condensação. b) fusão, vaporização e condensação. c) sublimação, vaporização e condensação. d) condensação, vaporização e fusão. Resolução: gelo/água  fusão; água/vapor  vaporização; vapor/água  condensação. (Resposta B) 88) (UFRN – RN - Modificado) A existência da água em seus três estados físicos, sólido, líquido e gasoso, torna nosso Planeta um local peculiar em relação aos outros Planetas do Sistema Solar. Sem tal peculiaridade, a vida em nosso Planeta seria possivelmente inviável. Portanto, conhecer as propriedades físicas da água ajuda a melhor utilizá-la e assim contribuir para a preservação do Planeta. Na superfície da Terra, em altitudes próximas ao nível do mar, os estados físicos da água estão diretamente relacionados à sua temperatura conforme mostrado no gráfico abaixo. Esse gráfico representa o comportamento de uma massa de 1,0 g de gelo a uma temperatura inicial de – 50 o C, colocada em um calorímetro que, ligado a um computador, permite determinar a temperatura da água em função da quantidade de calor que lhe é cedida. Observando-se o gráfico, pode-se concluir que a quantidade de calor necessária para elevar a temperatura do gelo a 0 o C até água a 100 o C é: a) 180 cal. b) 200 cal. c) 240 cal. d) 100 cal Resolução: Q = 205 – 25 = 180 cal. (Resposta A) 89) (IFPE – PE) Uma amostra de determinada substância com massa 30g encontra-se inicialmente no estado liquido, a 60°C. Está representada pelo gráfico ao lado a temperatura dessa substância em função da quantidade de calor por ela cedida. Analisando esse gráfico, é correto afirmar que: a) a temperatura de solidificação da substância é 10°C. b) o calor específico latente de solidificação é - 1,0 cal/g. c) o calor específico sensível no estado líquido é 1/3 cal/g°C. d) o calor específico sensível no estado sólido é 1/45 cal/g°C. e) ao passar do estado líquido a 60°C para o sólido a 10°C a substância perdeu 180 cal. Resolução: a(F) o gráfico mostra que a temperatura se torna constante a 30 ºC. É a temperatura de mudança de estado (solidificação). b(V) L = Q/m = -(60 – 30)/30 = -1 cal/g c(F) c = Q/m.∆ = 30/30.30 = 1/30 cal/g°C d(F) c = Q/m.∆ = 30/30.20 = 1/20 cal/g°C e(F) o gráfico mostra 90 calorias. (Resposta B) 90) (UFG – GO) No gráfico, está representada a variação da temperatura em função do tempo de uma massa de 200 g d’água. Suponha que a fonte forneceu energia térmica a uma potência constante desde o instante t = 0, e que toda essa energia foi utilizada para aquecer a água. Dado: c = 1,0 cal/g°C. Analisando-se o gráfico, pode-se afirmar que 1-( ) no primeiro minuto, não ocorreu absorção de calor.
  • 21. 21 2-( ) a potência da fonte térmica é de 16 kcal/min. 3-( ) a temperatura da água para t = 2 min é 80 °C. 4-( ) de 0 a 9 minutos, ocorreram três transições de fase. Resolução: 1(F) A fonte tinha potência constante. A água encontrava-se a 0 o C e no estado sólido. 2(V) Pot = m.c.∆/ ∆t = 200.1.100/1,25 = 16000 cal/min ou 16 kcal/min 3(V) Pot = m.c.∆/∆t  16000 = 200.1./1   = 80 o C. 4(F) ocorreram 2. Fusão e vaporização. (Resposta: 1-F; 2-V; 3-V; 4-F) 91) (PUC – RS) Muitas pessoas, ao cozinharem, se preocupam com a economia de gás e adotam algumas medidas práticas, como: I. deixar o fogo baixo do início ao fim, pois assim se obtém cozimento mais rápido; II. baixar o fogo quando a água começa a ferver, pois a temperatura permanece constante durante a ebulição; III. deixar o fogo alto do início ao fim, obtendo uma constante elevação de temperatura, mesmo após o início da ebulição. Pela análise das afirmativas, conclui-se que somente a) está correta a I. b) está correta a II. c) está correta a III. d) estão corretas a I e a III. e) estão corretas a II e a III. Resolução: I-F a água demorará mais tempo para chegar a sua temperatura de ebulição e ali estabilizar. II-V a temperatura permanecerá constante dali por diante, independentemente do fluxo do calor, apenas mudando de estado físico. III-F durante a mudança de estado a temperatura permanece constante. (Resposta B) 92) (UFRN – RN) Cotidianamente são usados recipientes de barro (potes, quartinhas, filtros etc.) para esfriar um pouco a água neles contida. Considere um sistema constituído por uma quartinha cheia d´água. Parte da água que chega à superfície externa da quartinha, através de seus poros, evapora, retirando calor do barro e da água que o permeia. Isso implica que também a temperatura da água que está em seu interior diminui nesse processo. Tal processo se explica porque, na água que evapora, são as moléculas de água a) com menor energia cinética média que escapam do líquido, aumentando, assim, a energia cinética média desse sistema. b) que, ao escaparem do líquido, aumentam a pressão atmosférica, diminuindo, assim, a pressão no interior da quartinha. c) com maior energia cinética média que escapam do líquido, diminuindo, assim, a energia cinética média desse sistema. d) que, ao escaparem do líquido, diminuem a pressão atmosférica, aumentando, assim, a pressão no interior da quartinha. Resolução: A água, ao evaporar, absorve energia do sistema, resfriando-o. (Resposta C) 93) (ITA – SP) Um pedaço de gelo flutua em equilíbrio térmico com uma certa quantidade de água depositada em um balde. À medida que o gelo derrete, podemos afirmar que a) o nível da água no balde aumenta, pois haverá uma queda de temperatura da água. b) o nível da água no balde diminui, pois haverá uma queda de temperatura da água. c) o nível da água no balde aumenta, pois a densidade da água é maior que a densidade do gelo. d) o nível da água no balde diminui, pois a densidade da água é maior que a densidade do gelo. e) o nível da água no balde não se altera. Resolução: Ao fundir, a parte submersa do gelo sofre uma contração térmica de mesmo volume que a parte emersa do gelo. Assim o nível da água não se altera com o derretimento do gelo. (Resposta E) 94) (CEFET – PR) Uma estudante colocou em um recipiente 2,0 litros de água cuja densidade é 1 g/cm³, inicialmente a 20 ºC, para ferver. Distraindo-se, esqueceu a água no fogo por um certo tempo e, quando percebeu, metade da água havia evaporado. Curiosa, desejou saber que quantidade de calor a água havia consumido no processo. Sendo o calor específico e o calor de vaporização da água, respectivamente, iguais a 1,0 cal/g. ºC e 540 cal/g, encontrou: a) 700 kcal. b) 620 kcal. c) 160 kcal. d) 540 kcal. e) 80 kcal. Resolução: Q = QS + QL = m.c.∆ + m1.LF = 2000.1.(100 – 20) + 1000.540 = 700000cal ou 700 kcal. (Resposta A) 95) (FUVEST – SP) Um recipiente de isopor, que é um bom isolante térmico, tem em seu interior água e gelo em equilíbrio térmico. Num dia quente, a passagem de calor por suas paredes pode ser estimada, medindo-se a massa de gelo Q presente no interior do isopor, ao
  • 22. 22 longo de algumas horas, como representado no gráfico. Esses dados permitem estimar a transferência de calor pelo isopor, como sendo, aproximadamente, de: (Calor latente de fusão do gelo ≈ 320 kJ/kg) a) 0,5 kJ/h b) 5 kJ/h c) 120 kJ/h d) 160 kJ/h e) 320 kJ/h Resolução: Fluxo = Q/∆t = m.L/∆t = (20 – 8). 320/24 = 160 kJ. (Resposta D) 96) (ACAFE – SC) Patrícia deseja “gelar” um refrigerante que se encontra à temperatura ambiente. Para isso, dispõe de dois recipientes: um com certa massa de água e outro com igual massa de gelo, ambas a uma temperatura de 0ºC. Para conseguir o seu intento, da melhor forma possível, é aconselhável que Patrícia mergulhe o refrigerante: a) no gelo, porque inicialmente ele absorve calor e não aumenta de temperatura. b) no gelo, porque ele tem maior calor específico do que a água. c) na água, porque o equilíbrio térmico se dará a uma temperatura menor. d) na água, porque ela tem maior calor específico que o gelo. e) no gelo, porque ele contém menos calor do que a água. Resolução: Enquanto o gelo estiver derretendo sua temperatura permanece constante, abaixando apenas a temperatura do refrigerante. (Resposta A) 97) (UFES – ES) Quantas calorias são necessárias para vaporizar 1,00 litro de água, se a sua temperatura é, inicialmente, igual a 10,0 °C? Dados: - calor específico da água: 1,00 cal/g°C; - densidade da água: 1,00 g/cm³; - calor latente de vaporização da água: 540 cal/g. a) 5,40 × 104 cal b) 6,30 × 105 cal c) 9,54 × 106 cal d) 5,40 × 101 cal e) 6,30 × 10² cal Resolução: Q = m.c.∆ + m.LV = 1000.1.(100 – 10) + 1000.540 = 90000 + 540000 = 630000cal ou 6,30 × 105 cal. (Resposta B) 98) (UNICAMP – SP) Em um dia quente, um atleta corre dissipando 750 W durante 30 min. Suponha que ele só transfira esta energia para o meio externo através da evaporação do suor e que todo o seu suor seja aproveitado para sua refrigeração. Adote L = 2 500 J/g para o calor latente de evaporação da água na temperatura ambiente. a) Qual é a taxa de perda de água do atleta em kg/min? b) Quantos litros de água ele perde em 30 min de corrida? Resolução: a) Pot = Q/∆t  750 = Q/30.60  Q = 1350000 J. A massa de água Q = m.L  1350000 = m.2500  m = 540 g. A taxa será 540g/30 min = 18g/min = 0,018 kg/min. b) Em 30 min perde 0,018 x 30 = 0,540 g o que corresponde a 0,54 litros. (Respostas: a) 18g/min; b) 0,54 litros) 99) (PUC – RS) Colocam-se 420g de gelo a 0°C num calorímetro com água a 30°C. Após atingida a temperatura de equilíbrio térmico, verifica-se que sobraram 20g de gelo. Sendo de 80cal/g o calor de fusão da água, é correto afirmar que a temperatura final de equilíbrio térmico e a quantidade de calor ganho pelo gelo são, respectivamente, a) 30°C e 50kcal b) 30°C e 45kcal c) 15°C e 40kcal d) 0°C e 38kcal e) 0°C e 32kcal Resolução: O equilíbrio térmico se deu a 0 o C já que no recipiente sobraram ainda 20 g de gelo. A quantidade de calor absorvida pelo gelo é Q = m.L = (420 – 20).80 = 32000 cal ou 32 kcal.. (Resposta E) 100) (UERJ – RJ) Uma menina deseja fazer um chá de camomila, mas só possui 200 g de gelo a 0 o C e um forno de microondas cuja potência máxima é 800 W. Considere que a menina está no nível do mar, o calor latente de fusão do gelo é 80 cal/g , o calor específico da água é 1 cal/go C e que 1 cal vale aproximadamente 4 joules. Usando esse forno sempre na potência máxima, o tempo necessário para a água entrar em ebulição é: a) 45 s. b) 90 s. c) 180 s. d) 360 s.
  • 23. 23 Resolução: Pot = (Q1 + Q2)/∆t  800/4 = (m.L + m.c.∆)/∆t  200 = (200.80 + 200.1.100)/∆t  ∆t = 180 s. (Resposta C) 101) (UERJ – RJ) Uma bolinha de aço a 120º C é colocada sobre um pequeno cubo de gelo a 0º C. Em escala linear, o gráfico que melhor representa a variação, no tempo, das temperaturas da bolinha de aço e do cubo de gelo, até alcançarem um estado de equilíbrio, é: Resolução: Inicialmente, enquanto derrete, a temperatura do gelo se mantém constante. Após isso a água resultante tem sua temperatura elevada até o equilíbrio térmico com a bolinha de aço. (Resposta D) 102) (UERJ – RJ) Quatro esferas metálicas e maciças, E1 , E2 , E3 e E4 , todas com a mesma massa, são colocadas simultaneamente no interior de um recipiente contendo água em ebulição. A tabela abaixo indica o calor específico e a massa específica do metal que constitui cada esfera. Atingido o equilíbrio térmico, essas esferas são retiradas da água e colocadas imediatamente na superfície de um grande bloco de gelo que se encontra na temperatura de fusão. A esfera que fundiu a maior quantidade de gelo e a esfera que produziu a cavidade de menor diâmetro no bloco de gelo é respectivamente: a) E3 ; E4 b) E2 ; E4 c) E1 ; E3 d) E1 ; E2 Resolução: Com as esferas aquecidas a 100 ºC, a de maior calor específico deverá ceder uma quantidade maior de calor ao gelo até o equilíbrio térmico. Assim a esfera que forma a maior cavidade é a E1. Como as massas são iguais, a esfera que produzirá a cavidade de menor volume e, conseqüentemente, de menor diâmetro é a de maior densidade, ou seja, E3. (Resposta C) 103) (UFGO – GO) Em um copo grande, termicamente isolado, contendo água à temperatura ambiente (25ºC), são colocados 2 cubos de gelo a 0ºC. A temperatura da água passa a ser, aproximadamente, de 1ºC. Nas mesmas condições se, em vez de 2, fossem colocados 4 cubos de gelo iguais aos anteriores, ao ser atingido o equilíbrio, haveria no copo: a) apenas água acima de 0ºC b) apenas água a 0ºC c) gelo a 0ºC e água acima de 0ºC d) gelo e água a 0ºC e) apenas gelo a 0ºC Resolução: A quantidade de calor cedida pelo gelo será suficiente para reduzir a temperatura 1ºC ficando o sistema estabilizado a 0 o C com parte sólida e parte líquida, pois a 0 o C os fluxos de calor trocados entre as partes se tornam iguais, (Resposta D) 104) (UFF – RJ) Gelo seco nada mais é que gás carbônico (CO2) solidificado e sua aplicação vai de efeitos especiais em shows à conservação de alimentos. Tal substância é conhecida desde meados do século XIX e recebeu esse nome devido ao fato de não passar pela fusão, quando submetida à pressão atmosférica e à temperatura ambiente, como ocorre com o gelo comum. Considere um cubo de 0,10 kg de gelo seco, a - 78 o C, e um bloco de gelo comum de 1,0 kg, a - 10 o C,colocados em um recipiente. Desprezando a capacidade térmica do recipiente e a troca de calor com o ambiente: a) determine a temperatura de equilíbrio térmico; b) descreva os elementos que comporão o sistema no equilíbrio térmico. Dados: Temperatura de sublimação do gelo seco = - 78 o C Temperatura de fusão do gelo comum = 0 o C Calor latente de vaporização do gelo seco = 134 cal/g Calor específico do vapor de gelo seco = 0,20 cal/g o C Calor específico do gelo comum = 0,50 cal/g o C Resolução:
  • 24. 24 a) Abaixando a temperatura do gelo comum até a do gelo seco, temos: Q = m.c.∆ = 1000.0,5. (- 78 – (-10)) = - 34000 cal. Cedendo pouco a pouco esta quantidade de calor ao sistema, temos, para sublimar o gelo seco: Q = m.LS = 100.134 = 13400cal. Sobram, portanto, 34000 – 13400 = 20600 cal. No recipiente temos vapor de gelo seco e gelo comum, ambos a -78 ºC. Com o calor restante, a temperatura do sistema fica: Q = m.c.∆ + m.c.∆  20600 = 1000.0,5. ( – (-78)) + 100.0,2. ( – (-78))  20600 = 500 + 39000 + 20 + 1560  -19960 = 520   = -38,38 ºC. b) No recipiente temos vapor de gelo seco e gelo comum a -38,38 ºC. (Resposta: a) -38,38 ºC.; b) vapor de gelo seco e gelo comum) 105) (FATEC – SP - Modificado) Um frasco contém 20 g de água a 0 °C. Em seu interior é colocado um objeto de 50 g de alumínio a 80 °C. Os calores específicos da água e do alumínio são respectivamente 1,0 cal/g°C e 0,10 cal/g°C e o calor latente de vaporização da água é 540 cl/g. Supondo não haver trocas de calor com o frasco e com o meio ambiente, após atingida a temperatura de equilíbrio, Qual a quantidade de calor necessária para que toda toda a água do sistema vaporize? a) 12,9 kcal b) 10 kcal c) 5,4 kcal d) 12,48 kcal e) 6,24 kcal. Resolução: QA + QB = 0  mA.cA.∆A + mB.cB.∆B = 0  20.1.( - 0 ) + 50.0,1.( - 80) = 0  20 + 5 - 400 = 0   = 16 ºC. Assim Q = mA.cA.∆A + mA.LV + mB.cB.∆B = 20.1.(100 – 16) + 20.540 + 50.0,1.(100 – 16) = 1680 + 10800 + 420 = 12900 cal ou 12,9 kcal. (Resposta A) 106) (UFPR – PR) Um esquiador desce, com velocidade constante, uma encosta com inclinação de 30° em relação à horizontal. A massa do esquiador e de seu equipamento é 72 kg. Considere que todo o calor gerado pelo atrito no movimento seja gasto na fusão da neve, cujo calor latente de fusão é 3,6 x 105 J/kg, e suponha a aceleração da gravidade igual a 10 m/s2 . Determine a massa de neve fundida após o esquiador descer 90 m na encosta. Expresse o resultado em gramas. Resolução: A energia dissipada durante a descida é Q = m.g.h = 72.10.90.sen 30º = 32400 J. Na descida de 90 metros, a quantidade de gelo fundida será Q = m.LF  32400 = m. 3,6 x 105  m = 0,09 kg = 90 g. (Resposta: 90 g) Complementos de mudanças de fases (estados físicos) 01) Conhecer os processo de mudanças de estados físicos da matéria requer alguns conhecimentos básicos. O primeiro deles é saber qual a denominação de cada uma das mudanças e qual a influência da pressão na temperatura de mudança de estado. Assim, analise as afirmações seguintes e aponte a única incorreta: a) Qualquer passagem da fase líquida para a fase gasosa é denominada evaporação. b) A passagem da fase gasosa para a fase líquida é denominada condensação ou liquefação. c) Fusão é a passagem do estado sólido para o estado líquido. d) A passagem do estado gasoso para o estado sólido recebe o nome de cristalização. e) A evaporação de um líquido ocorre a uma temperatura inferior a denominada temperatura de ebulição. Resolução: Vaporização é a passagem da fase líquida para a gasosa e pode ocorrer de três maneiras distintas; por ebulição, por evaporação ou por calefação. (Resposta A) 02) (ENEM) A adaptação dos integrantes da seleção brasileira de futebol à altitude de La Paz foi muito comentada em 1995, por ocasião de um torneio, como pode ser lido no texto abaixo. “A seleção brasileira embarca hoje para La Paz, capital da Bolívia, situada a 3.700 metros de altitude, onde disputará o torneio Interamérica. A adaptação deverá ocorrer em um prazo de 10 dias, aproximadamente. O organismo humano, em altitudes elevadas, necessita desse tempo para se adaptar, evitando-se, assim, risco de um colapso circulatório.” (Adaptado da revista Placar, edição fev.1995) A adaptação da equipe foi necessária principalmente porque a atmosfera de La Paz, quando comparada à das cidades brasileiras, apresenta: a) menor pressão e menor concentração de oxigênio. b) maior pressão e maior quantidade de oxigênio. c) maior pressão e maior concentração de gás carbônico. d) menor pressão e maior temperatura. e) maior pressão e menor temperatura.
  • 25. 25 Resolução: À medida que a altitude aumenta ocorre uma diminuição na pressão atmosférica e uma conseqüente diminuição do nível de oxigênio. (Resposta A) 03) (PUC – RS) A temperatura de fusão de uma substância depende da pressão que é exercida sobre a mesma substância. O aumento de pressão sobre um corpo ocasiona, na sua temperatura de fusão: a) um acréscimo, se o corpo, ao se fundir, se expande. b) um acréscimo, se o corpo, ao se fundir, se contrai. c) um decréscimo, se o corpo, ao se fundir, se expande. d) um decréscimo para qualquer substância. e) um acréscimo para qualquer substância. Resolução: As substâncias ditas com comportamento normal (se expandem ao fundir) sofrem uma aumento em sua temperatura de fusão e ebulição quando ocorre um aumento de pressão. (Resposta A) 04) Em um passado recente o Paraná foi o maior produtora de café do mundo, cultura essa que promoveu o desenvolvimento acentuado de várias cidades produtoras no estado. A falta de subsídios para as lavouras de café, a insegurança dos agricultores frente às políticas de preços propostas pelo governo e a possibilidade de destruição da lavoura de café frente às geadas levaram a erradicação quase que total das plantações de café, dando lugar a outros tipos de lavouras como a soja, o trigo e o algodão, além da formação de enormes áreas de pastagens. A geada, um tormento para os proprietários de certos tipos de lavouras tem sua formação e ação explicada a seguir de três modos. Distintos: I- O vapor-d'água existente na atmosfera sofre condensação, precipitando-se, suavemente, nas plantas, sob a forma de pequenos cristais de gelo. II- A água existente no interior das plantas congela quando a temperatura ambiente atinge um valor muito pequeno, aumentando seu volume e rompendo células e vasos que compõem a planta. III- O vapor-d'água existente na atmosfera condensa-se em contato com a planta, transformando-se em pequenas gotículas (orvalho). Com a queda de temperatura, o orvalho congela (solidifica) e, ao fato, denominamos geada. a) apenas I é correta; b) apenas II é correta; c) apenas III é correta: d) existem duas afirmações corretas apenas e) as três afirmações estão corretas. Resolução: A condensação dos vapores em contato com a planta e a conseqüente queda de temperatura ocasiona o congelamento das gotículas e também da água existente no interior das plantas. Ao congelar, a água aumenta seu volume rompendo os vasos, resultando na morte da planta. (Resposta D) 05) (PUC-RS) Há uma relação entre a pressão e a temperatura nas quais ocorrem as mudanças de fase. Assim, é correto afirmar que a temperatura de: a) fusão do gelo é superior a 0ºC, quando a pressão é superior a 1atm. b) fusão do gelo é inferior a 0ºC, quando a pressão é superior a 1atm c) fusão do gelo é sempre 0ºC, independente da pressão. c) ebulição da água é inferior a 100ºC, quando a pressão é superior a 1atm. e) ebulição da água é sempre 100ºC, independente da pressão. Resolução: O diagrama representa as fases da água. Observe que sob pressão de 1 atm o ponto de fusão é 0 o C e o de ebulição, 100 o C. O aumento de pressão acarreta uma queda na temperatura de fusão e um aumento da temperatura de ebulição. (Resposta B) 06) (UFPI – PI) A experiência do regelo, feita pela primeira vez pelo físico John Tyndal (1820-1893), pode facilmente ser reproduzida da seguinte forma: - coloque um bloco de gelo a aproximadamente –2 °C, apoiado sobre dois suportes de madeira, de modo que a parte central do bloco fique livre; - prenda pesos nas extremidades de um fio fino de arame e, - coloque este fio transversalmente sobre a parte central do bloco de gelo, deixando os pesos pendentes. Observe que o fio de arame atravessa o bloco de gelo sem que este fique dividido em duas partes. A explicação para essa observação experimental é: a) O arame, estando naturalmente mais aquecido, funde o gelo, que perde calor para o ambiente, deixando o bloco novamente sólido.
  • 26. 26 b) O acréscimo de pressão no contato fio-gelo diminui a temperatura de fusão e produz derretimento do gelo sob o fio; este, por sua vez, desloca-se através da água formada, a qual se congela ao voltar à pressão normal. c) A pressão exercida pelo arame sobre o gelo aumenta seu ponto de fusão que, ao ser atingido, ocorre o degelo. Assim, o fio se desloca através da água formada, a qual se congela depois que o fio a atravessa. d) O ponto de fusão do gelo independe da pressão do arame sobre ele, o degelo ocorre simplesmente por causa do contato de substâncias diferentes, no caso fio-gelo. Cessado o contato, a água volta a congelar. e) O acréscimo de pressão sobre o bloco aumenta a temperatura de fusão e produz derretimento do gelo sob o fio; este se desloca através da água formada, a qual perde calor para o ambiente e congela novamente. Resolução: O aumento de pressão ocasionado pelo arame faz com que o gelo sob ele funda devido a uma diminuição na sua o temperatura de fusão (no caso inferior a -2º C). A água resultante desliza sobre o arame retornando a pressão inicial e congelando imediatamente. (Resposta B) 07) Você acorda num dia frio, prepara-se e vai para a escola ou trabalho caminhando, naturalmente bem agasalhado. Você repara que, ao respirar, “solta” uma pequena nuvem de "fumaça" durante a aspiração do ar. Três possíveis explicações são formuladas para o fato. I- Durante a respiração, seu organismo separa o vapor-d'água do restante dos componentes aproveitáveis do ar atmosférico. Tal vapor, por não ser aproveitado, é expelido, o que dá a impressão da formação de uma pequena “nuvem de fumaça”. II- A respiração nos dias frios exige um grande esforço. A energia necessária para que você respire vem da queima dos alimentos que ingere. Como a queima é muito grande, ocorre a formação de fumaça de vapor cuja quantidade é muito intensa ao ser expelida. III- Quando respiramos, aspiramos o ar atmosférico, expelindo-o em seguida. O vapor- d'água expelido, em contato com o meio ambiente, cuja temperatura é menor, a ele cede calor, condensando-se sob a forma de pequenas gotículas que, agrupadas, parecem uma pequena nuvem. a) apenas I é correta; b) apenas II é correta; c) apenas III é correta: d) existem duas afirmações corretas apenas e) as três afirmações estão corretas. Resolução: A “fumaça” é simplesmente o vapor d’água expelido que condensa sob a forma de pequenas gotículas formando a “nuvem”. (Resposta: C) 08) Uma garrafa de vidro cheia de água pode quebrar, dentro do congelador, por que: a) a água, ao se congelar, sofre aumento de volume, e suas moléculas exercem uma força muito intensa sobre as paredes da garrafa de vidro. c) a água quimicamente destrói as paredes do vidro; d) a garrafa não pode ter temperatura baixa; e) o gelo externo à garrafa exerce pressão muito maior que o gelo interno. Resolução: O congelamento da água ocasiona sua expansão durante a formação do gelo. As moléculas, com isso, exercem forças imensas nas paredes do recipiente, rompendo-as. (Resposta A) 09) A figura seguinte indica os diferentes pontos de ebulição em algumas cidades localizadas em altitudes diferentes. a) Qual a relação entre o ponto de ebulição e a altitude das cidades? b) Em Santos e em Quito, a água a 0º C se encontraria em qual estado físico? Justifique. Resolução: a) À medida que a altitude aumento ocorre uma queda na pressão e com isso uma redução na temperatura de ebulição. b) Em Quito a água está no estado sólido, pois sua temperatura de fusão é maior que 0 o C já que a pressão é menor que a atmosférica. Em Santos a água pode estar no estado sólido, líquido ou em transição pois 0 o C é justamente sua temperatura de fusão. (Respostas: a) Diminui com o aumento da altitude; b) Quito: estado sólido. Santos: estado sólido, líquido ou em transição) 10) A decomposição de inúmeras rochas que se fragmentam em pedaços menores deve-se ao fato de possuírem fendas ou porosidades que permitem a infiltração de água. Nos países de clima muito frio, ou quando ocorre grande queda de temperatura observa-se a fragmentação mais acentuada. Explique fisicamente por que isto ocorre. Resolução: A água infiltrada nas rochas congela e durante a formação do gelo ocorre sua
  • 27. 27 expansão e o rompimento das fendas e poros. (Resposta: Devido ao congelamento da água) 11) (UFPI – PI) Podemos conhecer melhor uma substância através de seu diagrama de fases, que são curvas que delimitam as regiões correspondentes às fases sólida, líquida e gasosa da substância. A figura abaixo mostra o diagrama de fase da água, líquido vital aos seres vivos e que é estudado universalmente. Analise o diagrama de fase mostrado e assinale, nas afirmativas, V (para verdadeiro) ou F (para falso). 1( ) Sob pressão de 4,58 mmHg a à temperatura de 0,01 ºC há a coexistência das três fases da água em equilíbrio: sólida, líquida e vapor. Esse ponto é o ponto triplo. 2( ) A curva AT do diagrama representa a curva de sublimação, cujos pontos representam os estados de equilíbrio entre as fases sólida e liquida. 3( ) À pressão de 2,51 mmHg e à temperatura ambiente de 20 ºC, o diagrama mostra que a água está no estado de vapor. 4( ) A curva TB do diagrama mostra que a fusão é facilitada pelo aumento da pressão. Resolução: 1)(V) No ponto triplo (intersecção das três curvas) as condições de temperatura e pressão determinam a coexistência dos três estados físicos. 2(F) Entre os estados sólido e gasoso (no caso, vapor). 3(V) o ponto (pxV) estará determinado abaixo das curvas AT e TC. 4(V) Já que o aumento de pressão ocasiona uma queda na temperatura de fusão. (Resposta: 1(V); 2(F); 3(V); 4(V)) 12) (UFMS – MS) Uma cozinheira resolve ferver água em uma panela de pressão, para atingir maior temperatura que em panelas abertas. Para isso, coloca água no seu interior, onde todo o sistema, panela e água, está em equilíbrio térmico com o ambiente na temperatura TA e pressão atmosférica pA. Em seguida, fecha a panela e coloca-a sobre a chama de um fogão. A água, no interior da panela, vai aumentando a pressão e a temperatura e, após certo tempo entra em ebulição liberando vapores para o ambiente pela válvula. As figuras abaixo representam diagramas da pressão p x temperatura T da água, onde as linhas mais finas definem as regiões, de temperatura e pressão em que a água pode se apresentar em cada um dos possíveis estados, sólido (S), líquido (L) e vapor (V). Já as linhas mais grossas (negrito) representam processos termodinâmicos. Assinale qual dos diagramas representa corretamente o processo termodinâmico que a água, no interior da panela, sofreu desde o instante em que começou a ser aquecida, quando estava à temperatura TA e pressão pA, até o momento em que entra em ebulição à temperatura TB e Pressão pB. Resolução: Partindo do ponto pA,TA (estado líquido) a água vai gradativamente sofrendo aumento de pressão e temperatura determinando na curva de ebulição o ponto pB,TB conforme o diagrama D. (Resposta D) 13) As afirmações a seguir referem às substâncias que têm comportamento normal, ou seja, aumentam de volume durante a fusão. Analise cada uma delas e indicando se é verdadeira (V) ou falsa (F).